Solution
Solution
Chapter
Chapter Name Page Number
Number
So, the given set in the roster form is ∴ Required set = {1,2,3,4,5}
The given set in the set-builder form is {x: x = 2n , subset of B and B is called a super set of A.
We see that each member in the given set is a 13. Ans (A)
The set {5,25,125,625} in set-builder form is Every natural number is an integer and every
The set {2,4,6, … } in the set-builder form is {x: x = number is a real number.
7. Ans (C) So, the subsets of the set {1, {2}} are ϕ, {1}, {{2}}
∴ A ∩ (B ∩ C) = {2} ≠ ϕ {2,3,5,7,11,13,17,19,23,29}.
31. Ans (C) 39. Ans (B)
X ∩ Y ∩ Z = L.C.M. of (x, 2x, 5x) The smallest set A = {3,5,9}, such that
= 10x, ∀x ∈ N A ∪ {1,2} = {1,2,3,5,9}
32. Ans (B) 40. Ans (C)
In Venn diagram, difference of two sets A and B I. Let, A = {1,2,3,4}
can be represented as B = {x: x is a natural number and
4 ≤ x ≤ 6}
= {4,5,6}
A ∩ B = {4}
Hence, A and B are not disjoint.
II. Let, P = {a, e, i, o, u}
Q = {c, d, e, f}
Hence, Statement I is false. P ∩ Q = {e}
Now, it is clear from the diagram Hence, P and Q are not disjoint.
A − B, A ∩ B and B − A are mutually disjoint sets. III. Let, X = {x: x is an even integer }
Hence, Statement II is true. = {… − 6, −4, −2,2,4,6, … }
33. Ans (B) and Y = {x: x is an odd integer }
Using the set-builder form, we can write the = {… − 5, −3, −1,1,3,5, … }
definition of difference as ∴X∩Y=ϕ
A − B = {x: x ∈ A and x ∉ B} Hence, X and Y are disjoint.
34. Ans (D) 41. Ans (B)
aN = {ax: x ∈ N},3N = {3x: x ∈ N} We know, A − (A − B) = A ∩ B (refer Venn
= {x: x is a positive multiple of 3} diagram)
7N = {7x: x ∈ N}
= {x: x is a positive multiple of 7}
3N ∩ 7N = {x: x is a positive multiple of 21}
= {21x: x ∈ N} = 21N
35. Ans (D)
B − C = {3,4,5} − {4,5,7,8} = {3}
∴ Statement I is false.
A − (B − C) = {1,2,3,4} − {3}
Now, B ′ = {1,2,3,4,5,9,10}
= {1,2,4}
MATHEMATICS Page | 1A. 3
SETS
A ∩ B ′ = {1,5} = A = ϕ ∩ B′ = ϕ
A′ ∩ B ′ ∴ Statement II is true. 48. Ans (C)
42. Ans (C) n[(A ∩ B)′ ∩ A)
Now, A − (A − B) = A − (A ∩ B c ) = n[(A′ ∪ B ′ ) ∩ A]
= A ∩ (A ∩ B c )c [by De Morgan's law]
=A∩ (Ac ∪ B) = n[(A′ ∩ A) ∪ (B ′ ∩ A)]
= (A ∩ Ac ) ∪ (A ∩ B) [Distributive law]
=A∩B = n(ϕ ∪ (A ∩ B ′ )) = n(A ∩ B ′ )
Alternatively, refer venn diagram. = n(A − B) = n(A) − n(A ∩ B)
43. Ans (A) = 8 − 2 = 6.
′ (A′
(A ∪ B) ∪ ∩ B) 49. Ans (B)
= (A′ ∩B ∪ ′) (A′ ∩ B) Clearly, A′ = {1,4,5,6}, B ′ = {1,2,6}.
[by De-Morgan's law] Hence, A′ ∩ B ′ = {1,6}.
= A′ ∩ (B ′ ∪ B) by distribution of ∪ over ∩) Also, A ∪ B = {2,3,4,5}, so that
′ ′
=A ∩U= A (A ∪ B)′ = {1,6} = A′ ∩ B ′
44. Ans (A) It can be shown that the above result is true in
′ )′ (A′ ′
Consider (A ∪ B ∩ ∪ B) general. If A and B are any two subsets of the
= (A′ ∩ B) ∩ (A ∩ B ′)
universal set U, then (A ∪ B)′ = A′ ∩ B ′
= (B − A) ∩ (A − B) = ϕ Similarly, (A ∩ B)′ = A′ ∪ B ′
45. Ans (D) Alternatively, refer De Morgan's law
A = {2,4,6,8, … } 50. Ans (C)
B = {1,3,5,7, … } B ∪ C = {2,3,4,6,7,8}
A∩B=ϕ (B ∪ C)′ = U − (B ∪ C) = {1,5,9,10}
(A ∩ B)′ = U C − A = {4,8}
46. Ans (D) (C − A)′ = {1,2,3,5,6,7,9,10}
A ∪ C = {1,2,3,4,5,6} 51. Ans (C)
(A ∪ C) = U − (A ∪ C) = {7,8,9} Since A ⊆ B,
A ∪ B = {1,2,3,4,6,8} ∴A∪B=B
(A ∪ B) = U − (A ∪ B) = {5,7,9} So, n(A ∪ B) = n(B) = 6
′ ′
∴ (A ∪ C) ≠ (A ∪ B) 52. Ans (D)
We know, (A′ )′ =A The shaded region represents
B − C = {2,8} (P ∩ Q) ∪ (P ∩ R)
′
(B − C) = U − (B − C) 53. Ans (C)
= {1,3,4,5,6,7,9} From the given we have in interval notation A =
′ ′
B − C = (U − B) − (U − C) (0,3) and B = [1,5]
= {1,3,5,7,9} − {1,2,7,8,9} Clearly A − B = (0,1)
= {3,5} = {x ∈ R: 0 < x < 1}
′ ′ ′
∴ (B − C) ≠ B − C and B − A = [3,5]
47. Ans (C) = {x ∈ R: 3 ≤ x ≤ 5}
A ∩ (A ∪ B) = A ∩ (A′ ∩ B ′ )
′
∴ AΔB = (A − B) ∪ (B − A)
′) ′
= (A ∩ A ∩ B = (0,1) ∪ [3,5]
MATHEMATICS Page | 1A. 4
SETS
74. (C)
83. (C)
A = {1,3,5,15}
B = {2,3,5,7} A = {x ∈ R: |x| < 2}
C = {2,4,6,8} B = {x ∈ R: |x − 2| ⩾ 3}
A ∪ C = {1,2,3,4,5,6,8,15} |x| < 2
(A ∪ C) ∩ B = {2,3,5} ⇒ −2 < x < 2
∴ A = (−2,2)
75. (A) |x − 2| ⩾ 3
A ∪ B = {2,3,4,5,8,10,12} ⇒x−2⩾3
or x − 2 ⩽ −3
A ∪ C = {2,3,4,5,6,8,10,12,14}
x⩾5
(A ∪ B) ∩ (A ∪ C) = {2,3,4,5,8,10,12} or x ⩽ −1
76. (D) B = (−∞, −1] ∪ [5, ∞)
A ∪ B = ℝ − [2,5)
S − 1A − B = {7,10} A ∩ B = (−2, −1]
S − 2A − B = {x: x ∈ A, x ∉ B} B − A = (−∞, −2] ∪ [5, ∞)
77. (B) = ℝ − (−2,5)
A − B = [−1,2)
A−B = A − (A ∩ B)
= (0,3) − [1,3)
= (0,1)
78. (A)
X∪Y=X
84. (B)
Y⊂X
Maximum number of elements in A ∩ B
both correct and II Explains I
= minimum of {n(A), n(B)}
= 22
79. (C) 85. (B)
A = {1,2}, B = {3,4,5}, C = {2,3,4,5} S = {3,6,9 … . ,99}
n(S) = 33
(A ∪ B) = (A ∪ C)
P = {2,3,5,7,11,13,17,19}
But B ≠ C n(P) = 8
80. (C) ∴ n(S) + n(P) = 41
15
If elements in B1 , B2 , … , Bn are not repeated, then
total number of elements is 3n but each element is
3n 3n
repeated 9 times, so S = ⇒ 15 = ⇒ n = 45
9 9
103. (B)
2m = 112 + 2n → Verify options 108. (A)
104. (B) By substituting the values of n, we get;
(A ∩ B′)′) ∪ (B ∩ C) = (A′ ∪ B) ∪ (B ∩ C) X = {0, 49, 490, … }, Y = {0, 49, 98, … . }
[ By Demorgan’s Law] ∴ X ⊂ Y.
= A′ ∪ B 109. (C)
1
105. (D) The graph of y = and y = −x do not intersect.
x
Consider the following Venn diagram with 110. (A)
universal set U as the set of all the quadrilaterals in From the Venn diagram, it is clear that: A ∩
plane. (A ∪ B) = A
111. (B)
A′ ∪ [(A ∪ B) ∩ B′)] = A′ ∪ [(A ∩ B′) ∪ (B ∩ B′)]
Clearly, F2 ⊂ F1 , F3 ⊂ F1 , F4 ⊂ F1 = A′ ∪ [(A ∩ B ′ ) ∪ ∅]
∴ F1 = F1 ∪ F2 ∪ F3 ∪ F4 = A′ ∪ (A ∩ B ′ ) = (A′ ∪ A) ∩ (A′ ∪ B ′ )
⇒0<x+5<7 2. (A)
115. (D) The power set of a set containing n elements has 2n
Since x = −1 and x = 1 are roots elements.
∴ (x + 1) and (x − 1) are factors of equation. Clearly, 2n cannot be equal to 26
∴ (x + 1)(x − 1) = x 2 − 1 = 0 is required 3. (C)
equation. We have, A ⊃ B ⊃ C
∴ Set builder form of given set A is ∴ A ∪ B ∪ C = A and A ∩ B ∩ C = C
A = {x: x is a root of the equation x 2 = 1} ⇒ (A ∪ B ∪ C) − (A ∩ B ∩ C)
116. (A) = A−C
Given, A ⊂ B ⇒ A ∩ B = A 4. (C)
∴ n(A ∪ B) = n(A) + n(B) − n(A ∩ B) A ∩ B = {x: x a multiple of 3}
= n(A) + n(B) − n(A) = n(B) and {x: x is a multiple of 5}
117. (B) = {x: x is a multiple of 15}
We have, n(A) = 6 = {15, 30, 45, … … … . }
Total number of subsets of A = 26 5. (C)
∴ Number of subsets of A which contains atleast We have, A ⊂ B and B ⊂ C
6
two elements = 2 − 7 ∴ A ∪ B = B and B ∩ C = B
[∵ Subsets of A having none or 1 element are ⇒ A∪B= B∩C
= 64 − 7 = 57 6. (D)
118. Ans (D) We have,
2
x + 1 ≠ 0, ∀x ∈ R A ∩ (A ∩ B)c = A ∩ (Ac ∪ B c )
⇒ A ∩ (A ∩ B)c
119. Ans (C)
= (A ∩ Ac ) ∪ (A ∩ B c )
𝑚 𝑛
2 = 56 + 2
⇒ A ∩ (A ∩ B)c = ϕ ∪ (A ∩ B c )
120. Ans (B)
= A ∩ Bc
𝐴 = {−3, 3}, 𝐵 = {2,3,4}, 𝐶 = {2,3}
7. (C)
𝑁𝑜𝑤 𝑒𝑣𝑎𝑙𝑢𝑎𝑡𝑒 (𝐵 − 𝐶) ∪ 𝐴
It is given that
121. Ans (D)
A1 ⊂ A2 ⊂ A3 … ⊂ A99
• Use TSR min [n(A ∪ B)] = min {n(A), n(B)} 999
⇒ n (⋃ Ai ) = n(A99 ) = 99 + 1 = 100
i=1
∴ A − B = ϕ ⇒ n(A − B) = 0 9. (A)
n(B) − n(A) = 6 − 5 = 1
∴ ⋃ An = A10 = {2,3,5,7,11,13,17,19,23,29}
n=2
22. (A)
The set of negative real numbers is denoted by
(A ∪ B) ∪′ (A′ ′
∩ B) = A (−∞, 0).
15. (C) 23. (C)
5 4 3 2
U = {x: x + 6x + 11x − 6x = 0}
sin θ − cos θ = √2 cos θ
= {0, 1, 2, 3}
⇔ sin θ = (√2 + 1) cos θ
A = {x: x 2 − 5x + 6 = 0} = {2, 3}
⇔ (√2 − 1) sin θ = cos θ
And B = {x: x 2 − 3x + 2 = 0} = {2, 1}
⇔ sin θ + cos θ = √2 sin θ
∴ (A ∩ B)′ = U − (A ∩ B)
∴P=Q
= {0, 1, 2, 3} − {2} = {0, 1, 3}
24. (A)
MATHEMATICS Page | 1A. 10
SETS
A, B ⊂ S A′ ∩ B ′ ∴ Statement II is true.
⇒ A∪B⊂ S 29. Ans (C)
Now, A − (A − B) = A − (A ∩ B c )
25. (C) = A ∩ (A ∩ B c )c
p(A) = {ϕ, {ϕ}, {{ϕ}}, A} = A ∩ (Ac ∪ B)
26. (B) = (A ∩ Ac ) ∪ (A ∩ B)
A = {x | |x| < 3, x ∈ z} =A∩B
−3 < x < 3 Alternatively, refer venn diagram.
∴ A = {−2, −1,0,1,2}
R = {(x, y)/y = |x|, x ≠ 1} 30. Ans (A)
R = {(−2,2), (0,0), (1,1), (2,2)}
{(A − B) (B − C) (C − A)}
Therefore, the number of elements in the power
=ABC=U
set of R, 24 = 16
27. Ans (C)
I. Let, A = {1,2,3,4}
B = {x: x is a natural number and
4 ≤ x ≤ 6}
= {4,5,6}
A ∩ B = {4}
Hence, A and B are not disjoint.
II. Let, P = {a, e, i, o, u}
Q = {c, d, e, f}
P ∩ Q = {e}
Hence, P and Q are not disjoint.
III. Let, X = {x: x is an even integer }
= {… − 6, −4, −2,2,4,6, … }
and Y = {x: x is an odd integer }
= {… − 5, −3, −1,1,3,5, … }
∴X∩Y=ϕ
Hence, X and Y are disjoint.
28. Ans (B)
We know, A − (A − B) = A ∩ B (refer Venn
diagram)
∴ Statement I is false.
Now, B ′ = {1,2,3,4,5,9,10}
A ∩ B ′ = {1,5} = A
3x − 6 10 − 5x
⇒ ≤
5 3
⇒ 9x − 18 ≤ 50 − 25x
⇒ 9x + 25x ≤ 50 + 18 ∴ Solution set is (−1,7).
68 5. Ans (A)
⇒ 34x ≤ 68 ⇒ x ≤ ⇒x≤2
34 x (5x−2) (7x−3)
We have, < −
∴ Solution set is (−∞, 2]. 4 3 5
x 5(5x−2)−3(7x−3)
1 3x 1 ⇒ <
II. We have, ( + 4) ≥ (x − 6) 4 15
2 5 3
1 3x 4 1
⇒ ( + ) ≥ (x − 6)
2 5 1 3
Taking LCM in LHS,
1 3x + 20 1 ∴ Solution set is (4, ∞).
( ) ≥ (x − 6)
2 5 3 ⇒ 15x < 4[(25x − 10) − (21x − 9)]
3x + 20 x − 6
⇒ ≥ ⇒ 15x < 4[(25x − 10 − 21x + 9]
10 3
⇒ 15x < 4[4x − 1]
⇒ 3(3x + 20) ≥ 10(x − 6)
⇒ 15x < 16x − 4
⇒ 9x + 60 ≥ 10x − 60
⇒ 15x − 16x < −4 ⇒ −x < −4
⇒ 9x − 10x ≥ −60 − 60
6. Ans (A)
⇒ −x ≥ −120
x x
⇒ x ≤ 120 I. We have, x + + < 11
2 3
∴ Solution set is (−∞, −6). Hence, there exists only one pair of even integers
x
Transferring the term to LHS, (10,12).
2
x x 11. Ans (B)
⇒ − >1
3 2 We have, Profit = Revenue - Cost
2x − 3x −x
⇒ >1⇒ >1 = (60x + 2000) − (20x + 4000)
6 6
= 40x − 2000
Multiplying both sides by 6 ,
−x To earn some profit, 40x − 2000 > 0
6× >1×6
6 ⇒ x > 50
−x > 6 ⇒ x < −6 Hence, the manufacturer must sell more than 50
7. Ans (A) items to realize some profit.
We have, 4x + 3 < 5x + 7 12. Ans (B)
⇒ 4x − 5x < 7 − 3 ⇒ −x < 4 We have, 5x − 3 < 7
⇒ x > −4 Adding 3 on both sides,
∴ Solution set is (−4, ∞) 5x − 3 + 3 < 7 + 3
8. Ans (B) ⇒ 5x < 10
We have, 4x + 3 < 6x + 7. Dividing both sides by 5,
or 4x − 6x < 7 − 3 5x 10
< ⇒x<2
or −2x < 4 5 5
or x > −2 I. When x is an integer, the solution of the given
⇒ 3x > −6 as x < 2.
3x −6 I. Given, x ≥ −3
⇒ > ⇒ x > −2
3 3 Adding 5 on both sides,
(i) When x is an integer, the solution of the given x + 5 ≥ −3 + 5
inequality is {−1,0,1,2 … }. ⇒ x+5≥ 2
(ii) When x is a real number, the solution of the II. We have, −x ≤ −4
given inequality is (−2, ∞). i.e., all the numbers 2x ≥ 8
lying between -2 and ∞ but -2 and ∞ are not 1
III. Given, <0
x−2
included.
⇒ x−2< 0
10. Ans (A)
⇒x<2
Let x be the smaller of two consecutive even
14. Ans (A)
positive integers. Then, the other even integer is
Let x be the marks obtained by student in the
x + 2.
annual examination. Then,
Given, x > 8, x + 2 > 8 and x + x + 2 < 25
62 + 48 + x
⇒ x > 8 and 2x + 2 < 25 ≥ 60
3
⇒ x > 8 and 2x < 23 or 110 + x ≥ 180
23
⇒ x > 8 and x < ⇒ x = 10 or x ≥ 70
2
or −10 ≤ 5 − 3x ≤ 16 ⇒ |x − 2| ≥ 1 and |x − 2| ≤ 3
or −15 ≤ −3x ≤ 11 ⇒ (x − 2 ≤ −1 or x − 2 ≥ 1)
11 and (−3 ≤ x − 2 ≤ 3)
or 5 ≥ x ≥ −
3
−11
⇒ (x ≤ 1 or x ≥ 3) and
or ≤x≤5
3 (−1 ≤ x ≤ 5)
−11
∴ x∈[ , 5] ⇒ x ∈ (−∞, 1] ∪ [3, ∞)
3
and x ∈ [−1,5]
24. Ans (C)
Combining the solutions of two
The given inequality
inequalities, we have
7x
−3 ≤ 4 − ≤ 18 x ∈ [−1,1] ∪ [3,5]
2
Adding ( -4 ) to each term, 29. Ans (C)
7x |3x + 2| < 1 ⇔ −1 < 3x + 2 < 1
−3 − 4 ≤ 4 − − 4 ≤ 18 − 4
2 ⇔ −3 < 3x < −1
−7x 1
⇒ −7 ≤ ≤ 14 ⇔ −1 <x <− .
2 3
−2
Multiplying by ( ) to each term, 30. Ans (D)
7
2 7x 2 |3 − x| = 3 − x is true only
−7 × (− ) ≥ − × (− )
7 2 7 when 3 − x ≥ 0
2 ⇒ x ≤ 3.
≥ 14 × (− )
7
31. Ans (B)
⇒ 2 ≥ x ≥ −4 ⇒ −4 ≤ x ≤ 2
(x − 1)2 is always positive except when x = 1
∴ Solution set is [−4,2].
(and then it is 0 )
25. Ans (D)
∴ Solution is when x + 4 < 0
|3x − 5| ≤ 2
and x ≠ 1
⇒ −2 ≤ 3x − 5 ≤ 2
i.e. x < −4, x ≠ 1 ∴ x ∈ (−∞, −4).
⇒ 3 ≤ 3x ≤ 7
32. Ans (C)
7
⇒1≤x≤ We have, |x + 3| ≥ 10
3
26. Ans (C) ⇒ x + 3 ≤ −10 or x + 3 ≥ 10
2x − 1 = |x + 7| ⇒2≤x−3<4
⇒ −1 < x < 5
48. Ans (B)
Given −3 < 2x − 1 < 19
⇒ −2 < 2x < 20
4
So, xϵ(−∞, 0) ∪ ( , ∞)
⇒ −1 < x < 10 … (1) 5
So x= {-1,0,1,2,3……..} x2 + 7 x2 + 7
⇒ + 1 ⩽ 0( or ) −1⩾0
53. Ans(A) 8x 8x
x 2 + 8x + 7 x 2 − 8x + 7
Refer the fig, (x + 5)(x − 2) ≤ 0 and x ≠ 0 ⇒ ⩽ 0( or ) ⩾0
8x 8x
⇒ critical points for wavy curve are
⇒ x(x + 1)(x + 7) ⩽ 0 → (1)( or )
x=-5 & x=2
x(x − 1)(x − 7) > 0, x ≠ 0 → (2)
Draw all the critical points of (1) &(2) in number
line to apply wavy curve method
So, x ∈ [−5,2)
54. Ans(B)
2x − 1
−2 < <2
x−1
2x − 1 2x − 1
+ 2 > 0 and −2 <0
x−1 x−1
4x − 3 1
> 0 and <0
x−1 x−1
(4x − 3)(x − 1) > 0 and x − 1 < 0
So Set is (−∞, −7] ∪ [−1,0) ∪ (0,1] ∪ [7, ∞)
3
x ∈ (−∞, ) ∪ (1, ∞)&x < 1 58. Ans(B)
4
1 1
3 x+ > 2 ( or ) x + < −2
⇒ x ∈ (−∞, ) x x
4
⇒ x ≠ 1,0, −1
55. Ans(B)
∴ x ∈ ℝ − {−1,0,1}
2(3x − 4) ⩾ x + 1 − 4
59. Ans(B)
6x − 8 ⩾ x − 3
−9 ⩽ x + 2 ⩽ 9
5x ≥ 5 ⇒ x ⩾ 1
−11 ⩽ x ⩽ 7
56. Ans(A)
|x − 4| x ∈ [−11,7]
< 0, x ≠ 4
x−4 60. Ans(A)
(i)x − 4 < 0 7
x ∈ (−∞, )
−(x − 4) 2
<0 61. Ans(A)
(x − 4)
−1 < 0( true ) (i)−5 < x − 4 < 5 ⇒ −1 < x < 9
∴x<4 (ii)2x + 5 < −7 or 2x + 5 > 7
(ii)x − 4 > 0 ⇒ x < −6 or x > 1 − (2)
x−4 from (i) & (ii) x ∈ (1,9)
( )<0
x−4 62. Ans(C)
1 < 0( not true )
(x − 3)(x + 4) > 0
∴ only x < 4
⇒ x ∈ (−∞, −4) ∪ (3, ∞)
∴ x ∈ (−∞, 4)
63. Ans(A)
57. Ans(B)
5x − 6 ⩽ −8 or 5x − 6 ⩾ 8
x2 + 7
| |⩾1 5x ⩽ −2 or 5x ⩾ 14
8x −2 14
x⩽ or x ⩾
x2 + 7 x2 + 7 5 5
⇒ ⩽ −1( or ) ⩾1 −2 14
8x 8x (−∞, ] ∪ [ , ∞)
5 5
MATHEMATICS Page |2A. 7
LINEAR INEQUALITIES
9
–2x < – 8 x > 4 i.e., x (4, )
Solution set is (− , −2) ∪ (3, ∞) 2x−3 2+4x
2
+6≥ →(2)
4 3
77. Ans (B)
2x−3 2+4x
+6≥
We have, x2 – 5x + 4 < 0 4 3
(x – 4) (x – 1) < 0 6x + 63 8 – 63
Since, x Z, x = 2, 3 55 55
⇒x≤ i. e. , x ∈ (−∞, ]
10 10
Thus, there are 2 integral solutions.
Solution set is given by
78. Ans (A)
55 55
Let x and x + 2 be two even integers (−∞, ] ∩ (4, ∞) = (4, ]
10 10
such that x < 12, x < 10 and sum 2x + 2 > 14 83. Ans (C)
x < 10 and x > 6 2
Case (i): when x > 0, < 3 ⇒ 2 < 3x
x
6 < x < 12x=8 2 2
⇒ < x or x >
Thus, the pair of numbers is (8, 10). 3 3
2
79. Ans (C) Case (ii): when x < 0, < 3 ⇒ 2 > 3x
x
x+1 1 4x + 4 − x 2 − 2 2
⇒ >x⇒x .
2
> ⇒ >0 3 3
x2 + 2 4 4(x 2 + 2)
Which is satisfied when x < 0
x 2 − 4x − 2
⇒ <0 2
4(x 2 + 2) ∴ x ∈ ( , ∞) ∪ (−∞, 0)
3
x2 – 4x – 2 < 0(∵ 4(x 2 + 2) > 0) 84. Ans (C)
4 ± √16 + 8 10x − 1
⇒x= = 2 ± √6 4x − 3 ≥
2 3
⇒ 2 − √6 < x < 2 + √6 ⇒ 12x − 9 ≥ 10x − 1
x = 0, 1, 2, 3, 4. Thus 5 solutions. 12x – 10x + 9 – 1 2x 8
80. Ans (A) x 4 x [4, )
12x 12|x| 85. Ans (A)
| |≥ 1⇒ ≥ 1(∵ 4x 2 + 9 > 0)
4x2 +9 4x2 +9
We have, –3x + 17 < – 13
⇒ 4x 2 − 12|x| + 9 ≤ 0
MATHEMATICS Page |2A. 9
LINEAR INEQUALITIES
We have, 3x − 7 > 5x − 1 5 − 2x x 5 − 2x x − 30
≤ −5⇒ ≤
3 6 3 6
Transferring the term 5x to L.H.S. and the term −7
x − 30
to R.H.S. ⇒ 5 − 2x ≤ ⇒ 10 − 4x ≤ x − 30
2
Dividing both sides by 2 ⇒ 40 ≤ 5x
3x − 5x > −1 + 7 ⇒ −2x > 6 ⇒ 8 ≤ x ⇒ x ∈ [8, ∞)
2x 6 91. Ans (C)
⇒ < − ⇒ x < −3
2 2
If x cm is the breadth, then: 2(3x + x) ≥ 160
With the help of number line, we can easily look
⇒ x ≥ 20
for the numbers less than −3.
92. Ans (B)
Let Ravi got x marks in third unit test.
∴ Average marks obtained by Ravi
∴ Solution set is (−∞, −3), i.e., all the numbers Sum of marks in all tests
=
lying between −∞ and −3 but −∞ and −3 are not Number of tests
70 + 75 + x 145 + x
included as x < −3. = =
3 3
87. Ans (C)
Now, it is given that he wants to obtain an average
We have, 37 − (3x + 5) ≥ 9x − 8(x − 3)
of at least 60 marks.
(37 − 3x − 5) ≥ 9x − 8x + 24
At least 60 marks means that the marks should be
⇒ 32 − 3x ≥ x + 24
greater than or equal to 60.
Transferring the term 24 to L.H.S. and the term 145 + x
≥ 60 ⇒ 145 + x ≥ 60 × 3
(−3x) to R.H.S. 3
32 − 24 ≥ x + 3x ⇒ 8 ≥ 4x ⇒ 4x ≤ 8 ⇒ 145 + x ≥ 180
Dividing both sides by 4, Now, transferring the term 145 to R. H. S..
4x 8 x ≥ 180 − 145 ⇒ x ≥ 35
⇒ ≤ ⇒x≤2
4 4 i.e., Ravi should get greater than or equal to 35
marks in third unit test to get an average of at
least 60 marks.
∴ Solution set is (−∞, 2]. ∴ Minimum marks Ravi should get = 35.
88. Ans (A) 93. Ans (B)
We have, 3x − 2 < 2x + 1 We have, a < b and c < 0 Dividing both sides of
Transferring the term 2x to L.H.S. and the term (- a < b by c. Since, c is a negative number, sign at
2) to R.H.S. a b
inequality will get reversed. Hence, >
c c
Transferring the term (−2x) to L.H.S. and the term ⇒ 90 < 5 F − 160 < 180
6 to ⇒ 90 + 160 < 5 F < 180 + 160
R.H.S.; −x + 2x > 11 − 6 ⇒ x > 5 250 340
⇒ 250 < 5 F < 340 ⇒ <F<
5 5
Draw the graph of inequations (i) and (ii) on the
⇒ 50 < F < 68
number line,
102. Ans (C)
The given system of inequalities is x + 2 > 11…….
(i)
Hence, solution set of the equations are real 2x ≤ 20…...(ii)
numbers, x lying on greater than 5 excluding 5.i.e., Now, x + 2 > 11 ⇒ x > 11 − 2 ⇒ x > 9 ⇒ x ∈
x>5 (9, ∞)
∴ Solution set is (5, ∞) ∴ Solution of inequality (i) is x > 9……(iii)
97. Ans (D) and 2x ≤ 20 ⇒ x ≤ 10 ⇒ x: ∈ (−∞, 10]
Given inequality is 5x + 1 > −24 ∴ Solution of inequality (ii) is x ≤ 10……(iv)
⇒ 5x > −25 ⇒ x > −5
Also, 5x − 1 < 24 ⇒ 5x < 25 ⇒ x < 5
Hence, −5 < x < 5 ⇒ x ∈ (−5,5)
98. Ans (B) Clearly the common values of x satisfying (iii) and
2x − 7 < 11 ⇒ 2x < 18 ⇒ x < 9 (iv) lie between 9 and 10 .
3x + 4 < −5 ⇒ 3x < −9 ⇒ x < −3 Hence, the solution of the given system is given by
Hence, common solution is x < −3. 9 < x ≤ 10 ⇒ x ∈ (9,10]
So, x ∈ (−∞, −3)
99. Ans (C)
Case I :
2 2 2 103. Ans (C)
When x > 0, < 3 ⇒ 2 < 3x ⇒ < x or x >
x 3 3
Conceptual
Case II:
104. Ans (B)
2 2 2
When x < 0, < 3 ⇒ 2 > 3x ⇒ > x or x < ,
x 3 3 We have 3x − 7 > x + 1
which is satisfied when x < 0. ⇒ 2x > 8 ⇒ x > 4,So x ∈ (4, ∞)
2
∴ x ∈ (−∞, 0) ∪ ( , ∞)
3
105. Ans (D)
5 113. Ans(C)
x−
⇒ 6 < 0 ⇒ x ∈ (− 1 , 5) If x < 5 then −x > −5
5 4 6
x+ 114. Ans(C)
4
109. Ans (C) Given that x < y, b < 0
We have
3
<1 x y
x−2 ⇒ > ,b < 0
b b
3
⇒ −1<0 115. Ans(A)
x−2
3 − (x − 2) Given that −3x + 17 < −13
⇒ <0
x−2 ⇒ −3x < −17 − 13 ⇒ −3x < −30
5−x 3x > 30 ⇒ x > 10
⇒ < 0 ⇒ x ∈ (−∞, 2) ∪ (5, ∞)
x−2
⇒ x ∈ (10, ∞)
110. Ans (A)
116. Ans(B)
A= {m ∈ 𝐑: x2-(m+1)x+m+4=0 has real roots }
Given that |x| < 3 ⇒ − 3 < x < 3
D ≥ 0 ⇒ (m + 1)2 − 4(m + 4) ≥ 0
117. Ans(D)
⇒ 𝑚2 + 2𝑚 + 1 − 4𝑚 − 16 ≥ 0
Given that |x| > b, b > 0
⇒ 𝑚2 − 2𝑚 − 15 ≥ 0
⇒ x < −b or x > b
⇒ (m − 5)(m + 3) ≥ 0
⇒ x ∈ (−∞, −b) ∪ (b, ∞)
⇒ m€(−∞, −3] ∪ [5, ∞)
118. Ans(C)
So, A=(−∞, −3] ∪ [5, ∞) & B[-3,5)
Given that |x − 1| > 5
Now, check the options
⇒ (x − 1) < −5 or (x − 1) > 5
A-B=(−∞, −3] ∪ [5, ∞)
⇒ x < −5 + 1 or x > 5 + 1
AB= {-3}
⇒ x < −4 or x > 6
B-A=(-3,-5)
⇒ x ∈ (−∞, −4) ∪ (6, ∞)
A∪B=R
119. Ans(B)
Hence option (A) is the correct answer.
Given that |x + 2| ≤ 9
111. Ans (B)
⇒-9≤ x + 2 ≤ 9
Given,
⇒-7≤ x ≤ 7
A=x: xϵ (-2,2) and B=x: xϵ (-∞, -1]∪[5, ∞)
⇒ x ∈ [−11,7]
(i)A ∩ B = (−2, −1]
120. Ans(A)
MATHEMATICS Page |2A. 12
LINEAR INEQUALITIES
7
The given graph represents ⇒3≤3x≤7⇒1≤x≤
3
x > −5& x < 5
130. Ans A
Combine to get |x| < 5
R(x) − c(x) > 0;
121. Ans(D)
60x + 2000 − 20x − 4000 > 0 & 𝑥 > 50
The gives graph represents all values of x greater
131. Ans (C)
than 5 including 5 on the real number line. So, x ∈
3𝑥=6⇒𝑥=2⇒4𝑥−𝑦=−1
(5, ∞)
⇒8−𝑦=−1⇒9=𝑦
122. Ans(B)
132. Ans (𝐂)
The given graph has all real values of x greater
a<b
9
than and equal to . a b
2
> (x < 0)
9 9 x x
So, x ≥ ⇒ x ∈ [ , ∞)
2 2
−1 9. Ans(A)
⇒ x ∈ (−∞, −2) ∪ (−2, ) ∪ (1, ∞)
2 x+5
5. Ans(D) ⩽0
x−2
(x − 4)(x − 3) ⇒ (x + 5)(x − 2) ⩽ 0 and x ≠ 2
>0
2x 2 + 4x + 5 ⇒ x ∈ [−5,2)
(x − 4)(x − 3) > 0
10. Ans(C)
∵ 2x 2 + 4x + 5 > 0, ∀x ∈ R
x+1 1 4(x + 1) − (x 2 + 2)
(Δ = 16 − 40 < 0) − > 0 ⇒ >0
x2 + 2 4 4(x 2 + 2)
x ∈ (−∞, 3) ∪ (4, ∞) 4x + 2 − x 2
⇒ > 0 ⇒ 4x + 2 − x 2 > 0
6. Ans(A) 4(x 2 + 2)
1 3 ⇒ x 2 − 4x − 2 < 0 ⇒ x ∈ (2 − √6, 2 + √6)
− <0
x+2 x−3
⇒ x ∈ (2 − 2.4 , 2 + 2.4)
x − 3 − 3x − 6
⇒ <0 ⇒ x ∈ (−0.4 , 4.4)
(x + 2)(x − 3)
−2x − 9 So, the integral values of x= {0,1,2,3,4}
⇒ <0
(x + 2)(x − 3) 11. Ans(D)
(2x + 9) x−4
⇒ >0 −2≥0
(x + 2)(x − 3) x+1
x − 4 − 2x − 2
⇒ critical points for wavy curve are ≥0
−9
x+1
x= , -2 & 3 −x − 6 x+6
2
≥0⇒ ⩽0
x+1 x+1
⇒ (x + 6)(x + 1) ⩽ 0 and x ≠ −1
⇒ x ∈ [−6, −1] and x ≠ −1 ∴ x ∈ [−6, −1)
12. Ans(A)
9
So x ∈ (− , −2) ∪ (3, ∞)
2 5x − 6 ≥ 8, ( or )5x − 6 ≤ −8
7. Ans(D) x≥
14
(or) x ≤
−2
5 5
7 + 9(x − 2) + (x − 2)(x − 3)
<0 2 14
(x − 2)(x − 3) ⇒ x ∈ (−∞, − ] ∪ [ , ∞)
5 5
(x + 5)(x − 1)
<0 13. Ans(C)
(x − 2)(x − 3)
x − 1 < −5( or )x − 1 > 5
⇒ critical points for wavy curve are
x < −4( or )x > 6
x=-5,1,2 & 3
x ∈ (−∞, −4) ∪ (6, ∞)
14. Ans (D)
2x − 3
−3≥0
3x − 5
xϵ (-5,1)∪ (2,3) 2x − 3 − 9x + 15
≥0
8. Ans(A) 3x − 5
x 2 + 2 ⩽ 3x and 3x ⩽ 2x 2 − 5 12 − 7x
≥0
3x − 5
x 2 − 3x + 2 ⩽ 0 and 2x 2 − 3x − 5 ⩾ 0
⇒ (12 − 7x)(3x − 5) ⩾ 0, x ≠ 5/3
(x − 1)(x − 2) ⩽ 0 and 2x 2 − 5x + 2x − 5 ⩾ 0
5 12
x ∈ [1,2] and (x + 1)(2x − 5) ⩾ 0 ⇒x∈( , ]
3 7
5 15. Ans (D)
x ∈ (−∞, −1] ∪ [ , ∞)
2
Solution set x = ϕ
MATHEMATICS Page |2A. 14
LINEAR INEQUALITIES
x2 – x + 3 0 for all x R
(∵ a > 0 and b2 – 4ac < 0)
x2 − x + 3
1 ∴ ≤0
x ∈ (−∞, −2) ∪ ( , 1) ∪ (4, ∞) x(x − 3)
4
⇒ x(x − 3) < 0 ⇒ 0 < x < 3
So, x ∉ (1,4)
⇒ x ∈ (0,3)
Solution set is (0, 3).
16. Ans (D)
21. Ans (B)
|4x – 3| < 27 –27 < (4x – 3) < 27
The given inequality
–24 < 4x < 30
6 ≤ −3(2x − 4) < 12
– 6 < x < 15/2
6 ≤ −6x + 12 < 12
15
Solution set is (−6, ) Adding ( -12) to each term,
2
Recall that a|b stands for a divides b. For the ∴ Total number of non-empty relations from A to
elements of the given sets A and B, B = 28p − 1
We find 2|6,2|10, 3|3,3|6,5|10 20. Ans (B)
R={(2,6), (2,10), (3,3), (3,6), (5,10)} 6
Given, R = {(x, y): y = x + ,
x
∴ R−1 is given by
where x, y ∈ N and x < 6}
= {(6,2), (10,2), (3,3), (6,3), (10,5)} 6
For, x = 1, y = 1 + = 7
1
15. Ans (C)
6
The given relation in Roster form is, x = 2, y = 2 + =5
2
R = {(3,5), (4,6), (5,7), (6,8), 6
x = 3, y = 3 + = 5
(7,9), (8,10), (9,11)} 3
∴ Domain of R = {3,4,5,6,7,8,9} 6 11
x = 4, y = 4 + = ∉N
4 2
So, A is true.
6 31
Range of R = {5,6,7,8,9,10,11} x = 5, y = 5 + = ∉N
5 5
So, R is false. ∴ The given relation in Roster form is
16. Ans (A) R = {(1,7), (2,5), (3,5)}
We have, y = x + 5, x < 4, x, y ∈ N Hence, domain of R = {1,2,3}
For, x = 1, y = 6 Range of R = {7,5}
x = 2, y = 7 21. Ans (C)
x = 3, y = 8 Prime numbers less than 10 are 2,3,5,7.
∴ In Roster form R = {(1,6), (2,7), (3,8)} Hence, relation R in Roster form is
Domain of R = {1,2,3} R = {(2,8), (3,27), (5,125), (7,343)}
Range of R = {6,7,8} 22. Ans (B)
17. Ans (A) The difference of two integers is also an integer.
Clearly, the difference between the elements in P ∴ Domain of R = Z
and Q is 2. So, in Set-builder form, Range of R = Z
Relation R = {(x, y): x − y = 2, x ∈ P and y ∈ Q} 23. Ans (C)
In Roster form, R = {(5,3), (6,4), (7,5)} ∵ 3 + 7 = 10,3 + 11 = 14,
Domain of R = {5,6,7} 5 + 7 = 12,5 + 11 = 16.
Range of R = {3,4,5} ∴ There is no element that belongs to R. Hence,
18. Ans (B) R = {}
Let the number of elements in set B be x. 24. Ans (A)
Number of elements in set A = 3 Given, A = {1,2,3,4,5,
Number of relations from A to B = 4096 R: A → A and R = {(x, y): y = 2x − 1}
3x
⇒2 = 4096 ∴ R = {1,1), (2,3), (3.5)}
3x 12
⇒2 =2 Hence, domain of R = {1,2,3}
On comparing, we get 3x = 12 25. Ans (C)
⇒x=4 Given, S = {(a, b): b = |a − 1|, a ∈ Z
So, number of elements in set B = 4 and |a| < 3}z → set of integers.
19. Ans (D) ∴ S = {(−2,3), (−1,2), (0,1), (1,0), (2,1)}
Given n(A) = 8 and n(B) = p ∴ Range of set (S) = {0,1,2,3}
8p
∴ Total number of relations from A to B = 2 (∵ |a| < 3 ⇒ −3 < a < 3)
KCET MATHEMATICS Page |3A. 2
RELATIONS AND FUNCTIONS
Given, f(x) = 2x − 5 1 1
1 + x2 + + 1 = 2 + x2 + 2
x2 x
Put x = 0, f(0) = 2(0) − 5 = −5
1 2
x = 7, f(7) = 2(7) − 5 = 9 = (x + )
x
∴ f(0) ≠ f(7) 49. Ans (C)
x = −3, f(−3) = 2(−3) − 5 f(x + y) = f(x) ⋅ f(y) ∀x, y ∈ N,
= −6 − 5 = −11 f(1) = 3
44. Ans (C)
f(2) = f 2 (1) = 32
2
f(x) = 2x + bx + c,
f(3) = f(1)f(2) = 33
f(0) = 3 ⇒ 3 = c
f(4) = 34
and f(2) = 1
f(k) = 3k
⇒ 1 = 8 + 2b + c
∑nk=1 f(k) = 3279
⇒ 2b + 3 = −7
f(1) + f(2) + f(3) + ⋯ … … + f(k) = 3279
⇒ b = −5
3 + 32 + 33 + ⋯ … 3k = 3279
2
∴ f(1) = 2 × 1 + (−5) × 1 + 3
3(3k − 1)
= 2 − 5 + 3 = 0. = 3279
3−1
KCET MATHEMATICS Page |3A. 4
RELATIONS AND FUNCTIONS
3k − 1 = 23(2) = 26 = 64
= 1093
2 55. Ans (B)
3k − 1 = 2186
2n(A)n(B) = 64 = 26 ⇒ n(A) ⋅ n(B) = 6
k
3 = 2187
And 2n(B)⋅n(C) = 512 = 29
k=7
⇒ n(B) ⋅ n(C) = 9
50. Ans (D)
n(A) ⋅ n(B) = 6 = 2 × 3(or)6 × 1
In the given function,
n(B) ⋅ n(C) = 9 = 3 × 3( or )1 × 9
1−x
f(x) = log ( ) n(B) = 3(or)1
1+x
2x Required sum of values of n(B)
Putting in place of x we get,
1+x2
=3+1
2x
2x 1+ 56. Ans (A)
f( ) = log ( 1 + x2 )
1+x 2 2x n(A) = n
1−
1 + x2
2
1 + x 2 + 2x 2n(A×A) = 2n
= log ( )
1 + x 2 − 2x 57. Ans (D)
1−x 2 1+x A = {1,2,3,4,5}
= log ( ) = 2log ( )
1+x 1−x verify the options
= 2f(x) 58. Ans (A)
A = {1,2,3,4,5,6,7,8}
51. Ans (B) x + 2y = 9
A = {1,2,3,4,5,6, … . ,10} if, x = 1, ⇒ y = 4
R = {(2,4), (4,3), (6,2)(8,1)} x = 3, ⇒ y = 3
∴ Domain of R is {2,4,6,8} x = 5, ⇒ y = 2
52. Ans (C) x = 7, ⇒ y = 1
x2 + y2 ⩽ 4 ∴ R = {(1,4), (3,3), (5,2), (7,1)}
2 2
y ⩽4−x R−1 = {(4,1), (3,3), (2,5), (1,7)}
y ⩽ √4 − x 2 domain of R−1 = {4,3,2,1}
4 − x2 ⩾ 0 59. Ans (B)
⇒ −2 ⩽ x ⩽ 2 R 2 is a function
Therefore the domain R
= {−2, −1,0,1,2}
53. Ans (C)
A = {1,2,3}, B = {1,4,6,9}. R ⊆ A × B
R = {(x, y): x > y, x ∈ A, y ∈ B}
R = {(2,1)(3,1)} 60. Ans (A)
Range = {1} f(x)=4x-x 2 =x(4-x)
54. Ans (C) f (a + 1) − f (a − 1) = (a + 1)(4 − a −
A = {a, b, c} ⇒ n(A) = 3 1) − (a − 1)(4 − a + 1)
B = {1,2} ⇒ n(B) = 2 = (a + 1) (3 − a) − (a − 1)(5 − a)
Therefore, Number of relations from A to B= = 3a – a2 +3-a-5a+ a2 +5-a
2n(A)⋅n(B) =8-4a
−1 − √3 ⩽ x ⩽ √3 − 1 Hence x ∈ (−∞, 0)
⩾1 0, x∈ℤ
f(x) = [2x] − 2[x] = {
1, x∉ℤ
Range = [1, ∞)
Range = {0,1}
104. Ans (A)
111. Ans (A)
1
f(x) = 1 − f: [2,3] → ℝ
1 + x2
0 ⩽ f(x) < 1 f(x) = x 3 + 3x − 2 is monotonically increasing
105. Ans (B) ⇒fmax and fmin exist at end point of given interval.
1 f(2) = 23 + 6 − 2 = 12
2
x + 2⩾2
x f(3) = 27 + 9 − 2 = 34
x4 + 1 Range = [12,34]
⩾2
x2
112. Ans (A)
x2 1
0⩽ ⩽ 1
1 + x4 2 f(x) = x 2 +
1 + x2
1
Range = [0, ] 1
2
= (x 2 + 1) + −1⩾2−1
106. Ans (C) (1 + x 2 )
0 ⩽ |sin x| ⩽ 1 ⇒ x 2 − a2 ≤ 0 ⇒ x 2 ≤ a2
0 ⩽ 3|sin x| ⩽ 3 → (2) ⇒ x ≤ ±a ⇒ −a ≤ x ≤ a
∴ n( A × B) = n( A) ⋅ n( B) = mn ⇒ a−b=5 ---(1)
4 − x ≥ 0 and x 2 − 1 > 0 x
f(x) =
x2 +1
−x ≥ −4 and (x − 1)(x + 1) > 0
2
x ≤ 4 and x < −1 and x > 1 f(2) =
5
∴ Domain of f(x) is (−∞, −1) ∪ (1,4] 2 2/5 10
f(f(2)) = f ( ) = =
[∵ by taking intersection of above intervals using 5 4 29
1+
25
number line]
129. Ans (C)
123. Ans(D)
f(x) = 2x 2
4−x
Given that: f(x) = f(b) − f(a) 2b2 − 2a2
x−4
=
We know that f(x) is defined if b−a b−a
= 2(b + a)
x−4≠0⇒x≠4
∴ required = 2(3.8 + 4) = 2 × 7.8 = 15.6
So, the domain of f(x) is = R − {4}
4−x 130. Ans (A)
Let f(x) = y = = −1
x−4
Domain of f(x) = (−1,1)
⇒ Range of f(x) = −1
3 + 4x − 4x 2 ≥ 0
124. Ans (D)
4x 2 − 4x − 3 ≤ 0
Given that: f(x) = √x − 1
(2x + 1)(2x − 3) ≤ 0
f(x) is defined if x − 1 ≥ 0 ⇒ x ≥ 1 −1 3
⇒x∈[ , ]
∴ Domain of f(x) = [1, ∞) 2 2
−1 3
Let f(x) = y = √x − 1 ⇒ Domain of g(x) = [ , ]
2 2
If x ∈ [1, ∞), then x − 1 ∈ [0, ∞) −1 3
Domain of f(x) + g(x) = (−1,1) ∩ [ , ]
2 2
⇒ √x − 1 ∈ [0, ∞)
−1
=[ , 1]
∴ Range of f(x) = [0, ∞) 2
We have, f(x) =
3
+ log10 (x 3 − x). √a2 − x 2 is defined for a2 – x2 0
4−x2
3 Now, a2 – x2 0 a2 x2 x2 a2
is defined for 4 – x2 0 i. e ., x 2
4−x2
KCET MATHEMATICS Page |3A. 12
RELATIONS AND FUNCTIONS
–a x a ⇒ x∈ [ – a, a] ⇒ x2 y − x + y = 0
12. Ans (B) For x to be real
We have, f (x) = x2 – 4x + 5 1 − 4y 2 ≥ 0
x2 – 4x + 4 + 1 (x – 2)2 + 1 ⇒ (1 − 2y)(1 + 2y) ≥ 0
We know that (x – 2)2 0 (x – 2)2 + 1 1 1 1
⇒ ( − y) ( + y) ≥ 0
2 2
Range: [1, )
1 1
13. Ans (C) ⇒ (y + ) (y − ) ≤ 0
2 2
In the given options only option (C) satisfies the 1 1
⇒ − ≤y≤
condition of a function. 2 2
1 1
Hence, option (C) is a function. ∴ y = f(x) ∈ [− , ]
2 2
14. Ans (A)
18. Ans (A)
For x ∈ (π, 3 π/2), we have
For f(x) to be defined
−1 < sin x < 0
x − 1 > 0 and 2x − 1 > 0 and 2x − 1 ≠ 1
⇒ 0 < 1 + sin x < 1 and 1 < (2 + sin x) < 2 1
⇒ x > 1, x > and x ≠ 1
2
∴ [sin x] = −1, [1 + sin x] = 0 and [2 + sin x] = 1
⇒x>1
⇒ f(x) = [sin x] + [1 + sin x] + [2 + sin x]
Hence, domain is (1, ∞).
= −1 + 0 + 1 = 0
19. Ans (D)
For x = π, we have
We have,
[sin x] = 0, [1 + sin x] = 1 and [2 + sin x] = 2
1 1 1
∴ f(x) = 0 + 1 + 2 = 3 f(x 2 ) = x 2 − 2
= (x − ) (x + )
x x x
3π
For x = , we have 1
2 = (x + ) f(x)
x
[sin x] = −1, [1 + sin x] = 0 and
20. Ans (D)
[2 + sin x] = 1
Given, f(x) = √x
∴ f(x) = −1 + 0 + 1 = 0
Hence, range of f(x) = {0, 3} f(25) √25 5
∴ = = =1
f(16) + f(1) √16 + √1 4+1
15. Ans (C)
21. Ans (B)
f(a) = a
For domain of f(x), 2 − 2x − x 2 ≥ 0
αa2
⇒ =a ⇒ x 2 + 2x − 2 ≤ 0
α+1
1 ⇒ −1 − √3 ≤ x < −1 + √3
⇒ αa2 = a2 + a ⇒ α = 1 + (∵ a ≠ 0)
a 22. Ans (C)
16. Ans (C)
We have,
1
Given, f(x) = 1 1
√4−x2
f(x)f ( ) = f(x) + f ( ) ⇒ f(x) = x n + 1
x x
For domain of f(x),
Now,
⇒ 4 − x2 > 0
f(10) = 1001 ⇒ 10n + 1 = 1001 ⇒ n = 3
⇒ x2 < 4
∴ f(x) = x 3 + 1 ⇒ f(20) = 203 + 1 = 8001
⇒ −2 < x < 2
23. Ans (A)
∴ Domain= (−2, 2)
Given, y = x − 3 ⇒ x − y = 3
17. Ans (C)
x ∴ R = {(11, 8), (13, 10)}
Let y =
1+x2 ⇒ R−1 = {(8, 11), (10, 13)}
KCET MATHEMATICS Page |3A. 13
RELATIONS AND FUNCTIONS
f(x) + 1
⇒x=
1 − f(x)
f(x) + 1
2 x − 1 2 {1 − f(x)} − 1 3 f(x) + 1
∴ f(2 x) = = =
2x+1 f(x) + 1 f(x) + 3
2{ }+1
1 − f(x)
29. Ans (A)
Given a set containing 10 distinct elements and
f: A → A Now, every element of a set A can make
image in 10 ways.
∴ Total number of ways in which each element
makes images = 1010 .
30. Ans (D)
since, f(x + y) = f(x) + f(y) and f(1)=7,
take f(x)=7x
Now, ∑ f(r)=∑ 7r
= 7(1 + 2 + 3+. . . +n)
7n(n + 1)
=
2
= 75 cm (sin θ + cos θ)
=
Length of arc (l) = 21 cm sin θ ⋅ cos θ
l 21 7
1 1
Now, θ = = = radian = + = sec θ + cosec θ
r 75 25 cos θ sin θ
13. Ans (D)
In II quadrant, only sin θ and cosec θ are positive.
∴ cos 2 θ = (1 − sin2 θ)
9 16
= (1 − )=
25 25
16 −4
10. Ans (D) ⇒ cos θ = −√ =
25 5
Since, diameter of the circle = 40 cm
−5
∴ Radius of the circle (r) = 20 cm ∴ sec θ =
4
and length of the chord (l) = 20 cm cos θ −4 5 −4
and cot θ = = × =
sin θ 5 3 3
From the shown figure, it is clear that ∠AOB =
(−5) (−4)
π
60∘ = radian ∴ (2sec θ − 3cot θ) = {2 × −3× }
3 4 3
∴ Length of minor arc of the chord (l) = radius of −5 3
=( + 4) =
2 2
circle (r) × angle subtended in radian by the
14. Ans (D)
minor arc (θ)
In III quadrant, only tan θ and cot θ are positive.
π 20π
= 20 × = cm sin2 θ = (1 − cos 2 θ)
3 3
9 16
= (1 − )=
25 25
16 4
⇒ sin θ = ±√ =±
25 5
−4
⇒ sin θ =
5
11. Ans (D) −4 5 4
∴ tan θ = ( × )=
tan 10∘ tan 20∘ tan 30∘ tan 40∘ 5 −3 3
3 −5
tan 50∘ tan 60∘ tan 70∘ tan 80∘ and cot θ = ⇒ cosec θ =
4 4
= tan (90 − 80)∘ tan (90 − 70)∘ −5
and sec θ =
3
tan (90 − 60)∘ tan (90 − 50)∘
−5 3
tan 50∘ tan 60∘ tan 70∘ tan 80∘ (cosec θ + cot θ) ( 4 + 4)
∴ =
(sec θ − tan θ) −5 4
= tan 80∘ × cot 80∘ × tan 70∘ × cot 70∘ ( − )
3 3
× tan 60∘ × cot 60∘ × tan 50∘ −2
( ) −2 3 1
× cot 50∘ = 1 = 4 = × =
−9 4 −9 6
( )
12. Ans (A) 3
(sin θ + cos θ)(tan θ + cot θ) 15. Ans (D)
π π
sin θ cos θ Given, f(x) = sin [x], − < x <
= (sin θ + cos θ) [ + ] 4 4
cos θ sin θ
Clearly, sin 0 = 0
sin2 θ + cos 2 θ
= (sin θ + cos θ) ( ) π 3.14
sin θ ⋅ cos θ and [ ] = [ ]=0
4 4
(∵ sin2 θ + cos 2 θ = 1) π
∴ ∀x ∈ [ 0, ) , sin [x] = 0
4
MATHEMATICS Page | 4A. 2
TRIGONOMETRY
π (∵ cos 2 θ + sin2 θ = 1)
∀x ∈ [− , 0) , [x] = −1
4
21. Ans (D)
∴ sin [x] = sin (−1) = −sin 1
3 π
We have: sin A = , where 0 < A <
So, the range of f(x) is {0, −sin 1}. 5 2
[∵ tan β is negative in II nd
Hence, minimum value is 7 quadrant ]
19. Ans (A) tan α + tan β
∴ tan(α + β) =
1 − tan α ⋅ tan β
π π 2π
(1 + cos ) (1 + cos ) (1 + cos ) (1 4
6 3 3 2+(−3) 2
= −4 = .
1−(2)( ) 11
7π 3
+ cos )
6 23. Ans (D)
√3 1 1 √3 1
= (1 + ) (1 + ) (1 − ) (1 − ) √3
2 2 2 2 −
sin 10∘ cos 10∘
3 1 1 3 3 [cos 10∘ − √3sin 10∘ ]
= (1 − ) (1 − ) = × = =
4 4 4 4 16 sin 10∘ cos 10∘
20. Ans (B) 1 √3
2 [ cos 10∘ − sin 10∘ ]
Given equations can be rewritten as 2 2
=
a b sin 10∘ cos 10∘
cos θ = , sin θ = 2[sin 30∘ cos 10∘ − cos 30∘ sin 10∘ ]
x−h y−k
=
a2 b2
sin 10∘ cos 10∘
Now, + =1 2[sin (30∘ −10∘ )] 2⋅2sin (30∘ −10∘ )
(x−h)2 (y−k)2
= = = 4.
sin 10∘ cos 10∘ 2sin 10∘ cos 10∘
1 47. (C)
= 2 × cos 15∘ = cos 15∘
2 Area of circle = πr2=9 π ⇒r=3cm
∘ ∘
⇒ sin θ = sin (90 − 15) ⇒ θ = 75 π
l = rθ=3. =π
3
39. Ans (D)
So Req. perimeter of sector=l+2r=π + 6
sin 55∘ − sin 35∘
sin 10∘ 48. (C)
2cos 45∘ ⋅ sin 10∘ For 1min=120 revolutions
= = √2
sin 10∘ ⇒2 sec=4 revolutions
[∵ sin C − sin D
Now, for 1 revolution angle turned is
C+D C−D
= 2cos sin ] 4× 2π=8π revolutions.
2 2
54. (D)
⇒ 1 + cosec 2 α + 2(1) + sec 2 α
A⋅M⩾ G⋅M
1 +2(1) = k + tan2 α + cot 2 α
cos θ = x + ⩾ 2
x ⇒ 5 + 1 + cot 2 α + 1 + tan2 α
but maximum value of cos θ is 1
= k + tan2 α + cot 2 α
⇒k=7
60. (D)
55. (C)
sin θ1 + sin θ2 + sin θ3 = 3
2P6 − 3P4 + 1 ⇒ sin θ1 = sin θ2 = sin θ3 = 1
= 2(cos 6 θ + sin6 θ) − 3(cos 4 θ + sin4 θ) + 1 π
⇒ θ1 = θ2 = θ3 =
2
= 2[1 − 3sin2 αcos 2 α] − 3[1 − 2 cos 2 α sin2 α] + 1 ∴ cos θ1 + cos θ2 + cos θ3 = 0
= 2−3+1 =0 61. (A)
56. (B) sin θ + cosec θ = 2
3 ⇒ sin θ = 1 = cos ecθ
sec θ + tan θ = ∴ sinn θ + cos ec n θ = 1 + 1 = 2
2
2 62. (A)
sec θ − tan θ =
3 AM ≥ GM
3 2 5 Tan2 α + cot 2 α ≥ 2
2tan θ = − =
2 3 6 63. (C)
5
tan θ =
12
Q ∈ I quadrant −4
tan θ =
sec θ + tan θ = p 3
1 ⇒ θ ∈ IIQ(or)IVQ
sec θ − tan θ =
p 4 −4
1 ⇒ sin θ = (or)
2sec θ = p + 5 5
p
p2 +1 72. (A)
sec θ =
2p
We can write,
64. (B)
Sin190=sin(180+10)= -sin10
acos θ + bsin θ = p → (1)
asin θ − bcos θ = q → (2) Sin200=sin(180+20)= -sin20
(1)2 + (2)2 ⇒ a2 + b2 = p2 + q2 ...... .......... .............
65. (A) ...... .......... .............
A+B= =
2 sin2 x + cos 2 x − sin2 xcos 2 x
⇒ cot A ⋅ cot B = 1 1 − sin2 xcos 2 x
=1
Req. Value = 1 1 − sin2 xcos 2 x
f(x) = 1
68. (D) f(2023) = 1
sin x = cos 2 x
74. (A)
now, cos 2 x + cos 4 x
= cos 2 x + sin2 x = 1 cos θ = 1
sec θ = 1
69. (B) cos 999 θ − sec 999 θ = 1 − 1 = 0.
π
A+B= ⇒ sin2 A + sin2 B = 1 75. (A)
2
sin A 1 + cot A
∴ K = sin2 50 + sin2 100 + ⋯ + sin2 80∘ [ ]
sin A 1 − cot A
+ sin2 850 3
1+
1 1 = 4 = 7.
= 8+ =8 3
2 2 1−
4
70. (A) 76. (C)
cot α1 ⋅ cot α2 … … ⋅ cot αn = 1 sin (6(180∘ ) + 60∘ )
π
⇒ α1 = α2 … … αn = √3
4 = sin 60∘ =
∴ max of 2
cos α1 ⋅ cos α2 … … ⋅ cos αn 77. (C)
1 1 1
= ⋅ …⋯⋅ (n times ) cos 4 x + cos 2 x = 1
√2 √2 √2 (cos 4 x + cos 2 x)2 = 1
1 cos 8 x + 2cos 6 x + cos 4 x = 1
= n/2
2
78. (D)
71. (B)
Cos 1> cos2
cos x is decreasing in (0, π)
82. (B)
86. (A)
cos αcos β 2
=
sin αsin β 1
Using Componendo and Dividendo.
cos (α − β) 2 + 1
= =3
cos (α + β) 2 − 1
87. (A)
1 1 2tan β/2
sin Acos B = tan α = , tan β =
4 3 1 − tan2 β/2
3sin Acos B = sin Bcos A 1 4
3 = =
sin Bcos A = 1 3
4 1−
4
1 3 tan α + tan β
sin (A + B) = + = 1 tan (α + β) =
4 4 1 − tan α ⋅ tan β
π
A+B= 1 4
2 + 5×3
=3 3= =3
88. (B) 4 5
1−
9
√3sin θ + cos θ 93. (B)
√3 1 π
= 2 [ sin θ + cos θ] = 2sin (θ + ) > 0 sin 120 ⋅ sin 480 ⋅ sin 540
2 2 6
sin 120 sin (600 − 120 )sin (60∘ + 12)sin 540
π =
0<θ+ <π sin 720
6
−π 5π 1 sin 360 cos 360 1
<θ< . = =
6 6 4 sin 720 8
101. (B)
108. (A)
⇒ y= 8−5 =3
132. (A) So, the minimum value of the given expression is 3 .
tan 75∘ − cot 75∘ 136. (A)
∘ ∘)
= tan 75 − cot (90 − 15 The given expression is
sin 75∘ sin 15∘
= tan 75∘ − tan 15∘ = − tan 3A − tan 2A − tan A
cos 75∘ cos 15∘
sin 75 cos 15 − cos 75 sin 15∘
∘ ∘ ∘ tan 3 A = tan (2 A + A)
=
cos 75∘ cos 15∘ tan 2 A + tan A
sin (75∘ − 15∘ ) ⇒ tan 3 A =
= 1 − tan 2 Atan A
1
× 2cos 75∘ cos 15∘ ⇒ tan 3 A(1 − tan 2 Atan A)
2
2sin 60∘ = tan 2 A + tan A
=
cos (75∘ + 15∘ ) + cos (75∘ − 15∘ ) ⇒ tan 3 A − tan 3 Atan 2 Atan A
√3 = tan 2 A + tan A
2× √3
= 2 = = 2√3
cos 90∘ + cos 60∘ 0 + 1 ⇒ tan 3 A − tan 2 A − tan A
2
= tan 3 Atan 2 Atan A
133. (B)
137. (D)
We know that if θ increases then the value of sin θ
sin (45∘ + θ) = sin 45∘ cos θ + cos 45∘ sin θ
also increases
1 1
π = cos θ + sin θ
So sin 1∘ < sin 1 [∵ 1 radian = sin 1]
180 √2 √2
134. (D) cos (45∘ − θ) = cos 45∘ cos θ + sin 45∘ sin θ
−3 θ
⇒ = 2cos 2 − 1
5 2
θ 3 2
⇒ 2cos 2 = 1 − =
2 5 5
θ 2 1 θ 1
⇒ cos 2 = = ⇒ cos = ±
2 5×2 5 2 √5
θ 1 π θ 3π
⇒ cos =− [∵ < < ]
2 √5 2 2 4
147. (C) 4 −3
And sin A = and cot A =
Given equation is tan x + sec x = 2cos x 5 4
5π π 5π π 2π π 1 2 1
+ − + 1 − tan2 α 1 − (7) 1−
= 2sin ( 18 18 ) ⋅ cos (18 18) + 2sin (9 9) cos 2α = = = 49
2 2 2 1 + tan2 α 1 2 1+ 1
1+( ) 49
7
2π π 48 24
−
⋅ cos ( 9 9) = =
50 25
2
2tan β
Now tan 2β =
π π π π 1−tan2 β
= 2sin ⋅ cos + 2sin ⋅ cos
6 9 6 18 1 2
2×
1 π 1 π π π = 3= 3 =2×9=3
= 2 × cos + 2 × cos = cos + cos 1 8 3 8 4
2 9 2 18 9 18 1−
9 9
π π π π 7π 8π 3
= sin ( − ) + sin ( − ) = sin + sin ∴ tan 2β =
2 9 2 18 18 18 4
7π 4π 2tan 2β
= sin + sin sin 4β =
18 9 1 + tan2 2β
149. (B)
Given that: 3tan A + 4 = 0, A lies in second 3 3
2× 3 16 24
= 4 = 2 = × =
quadrant 3 2 1+ 9 2 25 25
1+( ) 16
−4 −3 4
∴ tan A = & cos A =
3 5 24
cos 2α = sin 4β =
[A lies in second quadrant] 25
𝐍𝐨𝐭𝐞: option (B) would have been the correct 159. (A)
answer if ‘and’ is replaced by ‘or’. sin1° + sin2°+. . . . . + sin359°
⇒ sinθ =
2√3 +3
×
2√3 − 4
=
√3
⇒θ=
π tan θ ⋅ cot θ = 1
2√3 +4 2√3 −4 2 3
⇒ θ = √5 – 2
171. (D)
By graph
163. (B)
Consider √3 cosec 20° − sec20° 172. (B)
=
√3
−
1
=
√3cos20°− sin20° log ⋅ tan 1∘ tan 2∘ … . . tan 89∘
sin20° cos20° sin20°cos20°
√3 1
log 1 = 0
2( 2 cos20°− 2sin20°)
= ⇒ e0 = 1
sin20°cos20°
2(sin60°cos20°−cos20° sin20°) 173. (D)
=
sin20°cos20°
174. (C)
2×2sin(60°−20°) 4sin40°
= = =4
2sin20°cos20° sin40° 175. (A)
164. (D) Note that
2 2
We have, sin 51° + sin 39° sin[𝜋 2 ]𝑥 = sin 9𝑥 and sin [−𝜋 2 ]𝑥 = − sin 10𝑥
2 2
= sin 51° + sin (90° − 51°) 176. (B)
2 2
= sin 51° + cos 51° = 1 177. (C)
165. (D)
We have, tanA + cotA = 2
MOCK TEST SOLUTIONS:
Squaring both sides,
1. (C)
then we get tan2 A + cot 2 A + 2 = 4
1
We have,
(∵ tanA = ) ⇒ tan2 A + cot 2 A = 2
cot A cos(θ + ϕ) = m cos(θ − ϕ)
Again squaring both sides we get, 1 cos(θ − ϕ)
⇒ =
tan4 A + cot 4 A + 2 tan2 A. cot 2 A = 4 m cos(θ + ϕ)
⇒ tan4 A + cot 4 A = 2 1 + m 2 cos θ cos ϕ
⇒ =
166. (A) 1 − m 2 sin θ sin ϕ
1−m 1−m
cos (3 × 360∘ + 120∘ ) + tan (4 × 360∘ + 45∘ ) = ⇒ tan θ tan ϕ = ⇒ tan θ = cot ϕ
1/2 1+m 1+m
2. (D)
MATHEMATICS Page | 4A. 17
TRIGONOMETRY
1 1 sin(80 − 10) . cos70
Given that, sin A = and sin B = =
√10 √5
cos80cos10sin70
We know that, cos80cos10 + sin80sin10
=
sin(A + B) = sin A cos B + sin B cos A cos80cos10
= 1 + tan80. tan10
1 1 1 1
= √1 − + √1 − = 1 + tan(90 − 10) tan10
√10 5 √5 10
= 1 + cot10tan10
1
4 1 9 = 1+1= 2
= √ + √
√10 5 √5 10
6. (B)
1 5 1 π 2π 32π
= (2 + 3) = √ = cos cos … cos
√50 √50 √2 65 65 65
π π 2π 25 π
⇒ sin(A + B) = sin = cos . cos … cos
4 65 65 65
π 26 π 65π
⇒ A+B= sin sin
= 65 65
4 π = π
26 sin 64 sin
3. (B) 65 65
1 π
sin (π − ) 1
1 + | cos x| + cos 2 x + | cos 3 x|+. . . ∞ = = 65
1 − | cos x| π = 64
64 sin
65
1
∴ = 43 7. (B)
81−| cos x|
1
3 Given, cos x + sin x =
⇒ = 26 ⇒ 1 = 2 − 2| cos x| 2
21−| cos x|
1
1 ⇒ 1 + sin 2x =
⇒ | cos x| = 4
2
2 tan x −3
1 ⇒ =
⇒ cos x = ± 1 + tan x 4
2
⇒ 8 tan x = −3 − 3 tan2 x
π π 2π 2π
⇒ x = ,− , ,− ⇒ 3 tan2 x + 8 tan x + 3 = 0
3 3 3 3
∴ Number of solutions =4 −8 ± √64 − 36 −8 ± 2√7
⇒ tan x = =
4. (A) 6 6
∵ sin 2x + cos 4x = 2 4 ± √7
⇒ = −( )
3
It is possible only when
8. (B)
sin 2x = 1 and cos 4x = 1
π We have,
⇒ 2x = 2nπ + and 2x = 2mπ 12 3
2 cos(α + β) = and sin(α − β) =
π 13 5
∴ x = nπ + and x = mπ, n ∈ I 5 4
4 ⇒ sin(α + β) = and cos(α − β) =
13 5
Then, solution,
Now,
π
= (nπ + , n ∈ I) ∩ (mπ, m ∈ I) = ϕ sin 2α = sin{(α + β) + (α − β)}
4
5. (C) ⇒ sin 2α = sin(α + β) cos(α − β)
sin80 sin10 + cos(α + β) sin(α − β)
−
= cos80 cos10 5 4 12 3 56
sin70 ⇒ sin 2α = × + × =
cos70 13 5 13 5 65
sin80cos10 − sin10cos80 9. (D)
= cos80cos10 We have, sin10 2x = 1 + cos10 x
sin80
cos80
MATHEMATICS Page | 4A. 18
TRIGONOMETRY
√3 π 2π 16 3
⇒ sin x = ⇒ x = or, ⇒ cos θ − √1 − =−
2 3 3 25 5
21. (A)
3
θ 1+cos θ 1− 1
We have, Now, cos = ±√ = ±√ 5
= ±√
2 2 2 5
2 2 2 2
1 + 8 sin x cos x = 1 + 2 θ 1
But we take cos = −
2 2 )2 2 )2 2 √5
(2 + sin x cos x = 1 + 2(sin 2x
θ
= 1 + 2 sin2 2x 2 = 1 + (1 − cos 4x 2 ) Since, if θ lies in IIIrd quadrant then
2
2 2 will be in IInd quadrant
= 2 − cos 4x Now, −1 ≤ cos 4x ≤ 1
θ 1
⇒ 1 ≤ 2 − cos 4x 2 ≤ 3 Hence, cos = −
2 √5
⇒ 1 ≤ 1 + 8 sin2 x 2 cos x 2 ≤ 3 26. (D)
Hence, the required maximum value is 3
We have,
22. (D)
sin x + sin2 x = 1 ⇒ sin x = 1 − sin2 x
Since sin θ, cos θ are the roots of
⇒ sin x = cos 2 x
ax 2 − bx + c = 0
∴ cos 8 x + 2 cos 6 x + cos 4 x
b c
∴ sin θ + cos θ = and sin θ cos θ = = sin4 x + 2 sin3 x + sin2 x
a a
√2 + √2 + √2 + 2cos 8θ
= √2 + √2 + √2(2cos 2 4θ)
= √2 + √2(2cos 2 2θ)
= √2(2cos 2 θ) = 2cos θ
We have,
(3−2i)(2+3i) i4n+1 − i4n−1 i − i−1 2i
(1+2i)(2−i) = = =i
2 2 2
6 + 9i − 4i + 6
= 48. Ans (B)
2 − i + 4i + 2
in=i4m+3=i4mi3=1. (−i) = −i
12 + 5i 4 − 3i
= × 49. Ans (C)
4 + 3i 4 − 3i
48 − 36i + 20i + 15 63 − 16i 1+i 1−i
= = = i, = −i
16 + 9 25 1−i 1+i
63 16 1+i 3 1−i 3
= − i ( ) −( ) = i3 − (−i)3 = 2i3 = −2i
25 25 1−i 1+i
63 16
∴ required conjugate = + i 50. Ans (B)
25 25
We know that, if two complex numbers are equal, (1 + i)2n + (1 – i)2n = a purely real number
(x – 1)3 + 8 = 0 = in + in = 2in
1 ∴ Radius= |
k(α−β)
|=|
5(i+i)
|=
5
=ω+ = ω + ω2 = −1 1−k2 1−25 12
ω
Or, For z = −ω2 , we have 78. Ans (B)
33
z100 + z −100 = (−ω2 )100 + (−ω2 )−100 Given, (√5 + √3i) = 249 z
1 Let √5 = r cos θ,√3 = r sin θ
= ω200 +
ω200
∴ r 2 = 5 + 3 ⇒ r = 2√2
1
⇒ z100 + z −100 = ω2 + 2 ∴ (r cos θ + ir sin θ)33 = 249 z
ω
= ω2 + ω = −1 ⇒ |r 33 (cos 33θ + i sin 33θ)| = |249 z|
33
74. Ans (C) ⇒ (2√2) | cos 33θ + i sin 33θ| = 249 |z |
3+2i sin θ 99
Let z =
1−2i sin θ ⇒ 2 2 (1) = 249 |z| ⇒ |z| = √2
3 + 2i sin θ (1 + 2i sin θ) 79. Ans (D)
⇒z= ×
1 − 2i sin θ 1 + 2i sin θ
We have,
3 − 4 sin2 θ + 8i sin θ
= (1 − i)n = 2n
1 + 4 sin2 θ
For purely imaginary of z, put Re(z) = 0 ⇒ |1 − i|n = |2|n
n n n
3 − 4 sin2 θ ⇒ (√2) = 2n ⇒ 22 = 2n ⇒ 22 = 1
ie, =0
1 + 4 sin2 θ
⇒n=0
√3
⇒ sin θ = ± So, there is no non-zero integral solution of the
2
π π given equation
⇒ θ = nπ + (−1)n (+ ) = nπ ±
3 3 80. Ans (B)
75. Ans (B) z = 0 is the only complex number which satisfies
We know that, sum of any four consecutive powers the given relations
of i is zero 81. Ans (B)
13
The coordinates of the points representing 1 +
∴ ∑(in + in+1 )
n=1 i, i − 1 and 2i are (1,1), (−1,1) and (0,2)
= (i + i2 +. . . . +i13 ) + (i2 + i3 +. . . +i14 ) respectively
= i13 + i14 ∴ Requiredarea = 1sq. unit.
=i−1 82. Ans (D)
76. Ans (C) −1+√3i
We know, ω =
2
We have, 1000
1 √3i
√x + iy = ±(a + ib) ∴ (− + ) = (ω)1000 = ω
2 2
⇒ x + iy = a2 − b2 + 2iab [∵ ω3 = 1]
2 2
⇒ x = a − b , y = 2ab 83. Ans (D)
∴ √−x − iy = √−(a2 − b 2 ) − 2iab The vertices of the triangle are
⇒ √−x − iy = √b 2 − a2 − 2iab 1 √3 1 √3
A(0,1), B (− , ) andC (− , − )
2 2 2 2
= √(b − ia)2 = ±(b − ia)
0 1 1
77. Ans (B) 1 √3
z−α
1 |− 1|
On comparing the given circle with | | = k, we ∴ Areaof∆ABC = 2 2
z−β 2| |
1 √3
get − − 1
2 2
α = i,β = −i, k=5 ⇒ Areaof∆ABC
MATHEMATICS Page |5A. 7
COMPLEX NUMBERS
1 1 1 √3 √3 ∴ x 2 − 3x + [x] = 0
= [− (− + ) + 1 ( + )]
2 2 2 4 4 ⇒ x 2 − 3x + 2 = 0 ⇒ x = 1, 2 ⇒ x = 2
√3 Hence, the given equation has two solutions only
⇒ Areaof∆ABC = sq. units
4 88. Ans (C)
84. Ans (A) 100 100
−1 + √−3 −1 − √−3
√3 1 ( ) +( )
+i 2 2
= 2 2
1 √3 = ω100 + ω200 = ω + ω2 = −1
−i
2 2
89. Ans (B)
√3 + i
= We have,
1 − i√3
π π
√3 + i 1 + i√3 1 + i = √2 (cos + i sin )
= × 4 4
1 − i√3 1 + i√3 π π
and, 1 − i = √2 (cos + i sin )
4i 4 4
= =i
4 ∴ (1 + i)8 + (1 − i)8
85. Ans (A) π π 8 π π 8
= 24 (cos + i sin ) + 24 (cos − i sin )
(2 − ω)(2 − ω 2 )(2
−ω 10 )(2
−ω ) 11 4 4 4 4
4 (cos 4
= (2 − ω)(2 − ω2 )(2 − ω)(2 − ω2 ) =2 2π + i sin 2π) + 2 (cos 2π − i sin 2π)
∴ x 3 − 3x + [x] = 0 ⇒ x 2 − 3x + 1 = 0 =|
1
+
1
+. . . +
1
| [using Eq. (i)]
z1 z2 zn
3 ± √5
⇒x= 92. Ans (C)
2
(1+i)2
Clearly, these values of x do not belong to Given, = x + iy
2−i
4i − 2 z1 z̅1 1
⇒ = x + iy For collinear points |z2 z̅2 1| = 0
5
2 4 z3 z̅3 1
⇒ x + iy = − + i 1 + 2i 1 − 2i 1
5 5
∴ |2 + 3i 2 − 3i 1|
2 4 2
∴ x+y =− + = 3 + 4i 3 − 4i 1
5 5 5
4i 1 − 2i 1
93. Ans (A) = |6i 2 − 3i 1| [C1 → C1 − C2 ]
1 𝓏 8i 3 − 4i 1
𝓏3 + 𝓏2 − + 1 = 0
i i −2i −1 + i 0
= |−2i −1 + i 0| = 0
⇒ 𝓏 3 − i𝓏 2 + i𝓏 + 1 = 0
8i 3 − 4i 1
⇒ 𝓏 2 (𝓏 − i) + i(𝓏 − i) = 0 [R1 → R1 − R 2 ,R 2 → R 2 − R 3 ]
2
⇒ (𝓏 − i)(𝓏 + i) ⇒ |𝓏| = 1 Alternatively, use distance formula.
94. Ans (A) 101. Ans (A)
The number z=-4+5i is in second quadrant. verifytheoptionsbysquaring
After rotation in counter clock wise direction at
Hence, a = √2i = ±(1 + i)
angle 180°, it comes to the 4th quadrant
102. Ans (C)
∴ option A is correct
We have,
95. Ans (A) z − 12 5 z−4
| | = and | |=1
Mid point of vertices=origin z − 8i 3 z−8
Only option A satisfies Let z = x + iy. Then,
96. Ans (D) z − 12 5
| |=
verifytheoptionsbysquaring z − 8i 3
⇒ 3|z − 12| = 5|z − 8i|
97. Ans (D)
⇒ 3|(x − 12) + iy| = 5|x + (y − 8)i|
Given, z1/3 = p + iq
⇒ 9(x − 12)2 + 9y 2 = 25x 2 + 25(y − 8)2
⇒ (x − iy) = (p + iq)3 [put z = x − iy]
…(i)
⇒ (x − iy) = p3 — iq3 + 3p2 qi − 3pq2
z−4
⇒ (x − iy) = (p3 − 3pq2 ) + i(3p2 q − q3 ) and, | |=1
z−8
⇒ x = (p3 − 3pq2 ) and −y = 3p2 q − q3 ⇒ |z − 4| = |z − 8|
x 2 2 y 2 2
⇒ = (p − 3q ) and = (q − 3p ) ⇒ |x − 4 + iy| = |x − 8 + iy|
p q
x y ⇒ (x − 4)2 + y 2 = (x − 8)2 + y 2
∴ + = −2p2 − 2q2
p q ⇒x=6
x y
+ Putting x = 6 in (i), we get
p q
⇒ 2 = −2
(p + q2 ) y 2 − 25y − 136 = 0 ⇒ y = 17, 8
98. Ans (D) Hence, z = 6 + 17i or z = 6 + 8i
4 103. Ans (C)
z̅ =
1+i
Let z = x + iy, then z̅ = x − iy
99. Ans (D)
∴ z + z̅ = 2x and z − z̅ = 2iy
1
Let z =
i−1 Given, (3 + i)(z + z̅) − (2 + i)(z − z̅) + 14i = 0
̅̅̅̅̅̅̅
1 1 1
⇒ (3 + i)2x − (2 + i)2iy + 14i = 0
Then, z̅ = ( ) = =−
i−1 −i−1 i+1
x 4 − 3x 2 = 4 ….. (i) and 2x − 3y = −5 Alternatively, take z=1, solve and verify the
1
Form (i) and (ii), we get x = ±2 and y = 3, . options
3
1 118. Ans (A)
Trick: Put x = 2, y = 3 and then x = −2, y = ,
3 z−5i
| | = 1 ⇒ z would lie on the right bisector of the
we see that they both satisfy the given equation. z+5i
113. Ans (C) line segment connecting the points 5i and −5i.
Here z + z‾ = (x + iy) + (x − iy) = 2x (Real) and Thus z would lie on the x-axis
2 − 3i (2 − 3i)(4 + i) Let x = 3 + 2i
=
4−i (4 − i)(4 + i) ⇒ (x − 3)2 = 4i2 = −4
8 + 3 − 12i + 2i 11 − 10i ⇒ x 2 − 6x + 13 = 0
= =
16 + 1 17
Thus, x 4 − 8x 3 + 4x 2 + 4x + 39
11+10i
⇒ Conjugate = . = (x 2 − 6x + 13)(x 2 − 2x − 21) − 96x + 312
17
115. Ans (C) ∴ f(3 + 2i) = 0(x 2 − 2x − 21) − 96(3 + 2i) + 312
1 1 3 − √7i = 24 − 192i = a + ib
= ⋅
3 + √7i 3 + √7i 3 − √7i ∴ a = 24 and b = −192
3 − √7i ∴ =
a 24
=− .
1
= b −192 8
9+7
3−√7i 3 √7
120. Ans (A)
= = − i.
16 16 16 Taking modulus on both sides x 2 + y 2
116. Ans (C)
|u + iv| u2 + v 2
(1 + i)(2 + i) 1 + 3i 3 − i = = =1
|u − iv| u2 + v 2
z= = ×
(3 + i) 3+i 3−i
121. Ans (D)
3 + 4i
= (1 + i)2 −2 + 6i 1 3
5 = =− + i
3−i 10 5 5
⇒ |z| = 1
122. Ans (C)
|z1 ||z2 | √2⋅√5
Trick : |z| = |z3 |
= =1 Verifyforn=1,2,…
√10
Applying transformation x 2 + y 2 + 6x − 8y = 0
On putting z = x + iy z‾ x − iy x − iy x − iy
= = ×
we get x 2 − y 2 + 2ixy + x − iy = 0 z x + iy x + iy x − iy
x 2 + i2 y 2 − 2xyi
⇒ x 2 − y 2 + x = 0, y(2x − 1) = 0 =
1
x 2 − i2 y 2
⇒ x = or y = 0
2 x 2 − y 2 − 2xyi
=
When x = , y = ±
1 √3 x2 + y2
2 2
x2 − y2 2xy
When y = 0, x = 0 or -1 = − 2 i
2
x +y 2 x + y2
⇒ System has four solutions. z‾
When z lies in third quadrant then will also be lie
z
7−z −2i 48
Now f(z) = ⇒( ) = a + ib
1−z2 2i
7 − (1 + 2i) 7 − 1 − 2i 1 = a + ib ⇒ a = 1, b = 0
= =
1 − (1 + 2i)2 1 − 1 − 4i2 − 4i So, (a, b) = (1,0)
6 − 2i 3−i 3 − i 2 + 2i
= = = × 148. (C)
4 − 4i 2 − 2i 2 − 2i 2 + 2i
1 We have, z = x + iy
=1+ i
2 Also, |z + 1| = |z − 1|
1 2 ⇒ |(x + 1) + (iy)| = |(x − 1) + iy|
So,|f(z)| = √(1)2 + ( )
2
⇒ (x + 1)2 + y 2 = (x − 1)2 + y 2
1 √5 |z| ⇒ x 2 + 1 + 2x = x 2 + 1 − 2x
= √1 + = =
4 2 2 ⇒ 4x = 0 ⇒ x = 0
MATHEMATICS Page |5A. 13
COMPLEX NUMBERS
n
i. e. , y-axis
= ∑(r + 1)[r 2 + (ω + ω2 )r + ω3 ]
149. Ans (D)
r=1
1+i x n
( ) = 1 ⇒ ix = 1
1−i = ∑(r + 1)(r 2 − r + 1)
r=1
150. Ans (C) n 2
n(n + 1)
3x = 6 ⇒ x = 2and4x − y = −1 = ∑(r 3 + 1) = [ ] +n
2
⇒ 8 − y = −1 ⇒ 9 = y r=1
1+ω ω2 −ω And 2x − 4y + 14 = 0
∆= | 1 + ω2 ω −ω2 |
On solving, we get x = −1, y = 3
ω2 + ω ω −ω2
2
−ω2 ω2 −ω ∴ zz̅ = |z|2 = (√(−1)2 + (3)2 ) = 10
= | −ω ω −ω2 |
−1 ω −ω2 14. Ans (A)
ω2 ω2 ω ω2 ω 1 We have,
⇒ ∆= | ω 2 | = ω2 |
ω ω ω 1 ω|
(1 + i)2n − (1 − i)2n
1 ω ω2 1 1 ω
(1 + ω4 − ω2 )(1 − ω4 + ω2 )
⇒ ∆= ω2 {ω2 (ω − ω) − ω(ω2 − ω) + (ω − 1)}
{(1 + i)2 }n − {(1 − i)2 }n
⇒ ∆= ω2 {0 − ω3 + ω2 + ω − 1} = −3ω2 =
(1 + ω4 − ω2 )(1 − ω4 + ω2 )
10. Ans (A)
(2i)n − (−2i)n
1+i x
(1 + i)(1 + i)
x =
( ) =[ ] (1 + ω − ω2 )(1 − ω + ω2 )
1−i (1 − i)(1 + i)
(2i)n − (−2i)n
x x =
(1 + i) 1 − 1 + 2i (−2ω2 )(−2ω)
=[ 2
] =[ ]
1−i 2 = 2n−2 {in − (−i)n }
1+i x
⇒( ) = (i)x = 1 [given] 0, ifniseven
1−i ={
2n−1 in ,ifnisodd
∴ x = 4n
15. Ans (B)
11. (A) 3
(n+1)2 x + iy =
Given, an = i 2 + cos θ + i sin θ
2
Here, a1 = i2 = 1, a 2 = i3 = i,
2 3(2 + cos θ − i sin θ)
=
2 2
(2 + cos θ)2 + sin2 θ
a 3 = i4 = 1, a 4 = i5 = i,
6 + 3 cos θ − 3i sin θ
2 =
a 5 = i6 = 1, … 4 + cos 2 θ + 4 cos θ + sin2 θ
Here, we see that for all odd values of n, we get the 6 + 3 cos θ −3 sin θ
=[ ] + i[ ]
5 + 4 cos θ 5 + 4 cos θ
value of a n is 1
On equating the real and imaginary parts on both
∴ a1 + a3 + a 5 + ⋯ + a 25 =
sides, we get
3(2 + cos θ) −3 sin θ
x= ,y =
(5 + 4 cos θ) 5 + 4 cos θ
12. Ans (D) ∴ x 2 + y 2
9
We have, =(5+4 [4 + cos 2 θ + 4 cos θ + sin2 θ]
cos θ)2
n n
1 − i√3 −1 − i√3 9 6 + 3 cos θ
( ) +( ) = = 4[ ]−3
2 2 5 + 4 cos θ 5 + 4 cos θ
= ωn + (ω2 )n = ω6k + ω12k = 4x − 3
1 1 1 −1 + i√3
⇒ |z1 + z2 + ⋯ + zn | = | + + ⋯+ | = 28 ( ) ⇒ |z| = 28 (1) = 28
z1 z2 zn 2
17. Ans (C) ∴ Modulus= 28
1−i 4i
Let z = + 22. Ans (C)
3+i 5
5 − 5i + 12i − 4 1 + 7i
= = If n = 100, ∑100 k
k=1 i = 0
5(3 + i) 5(3 + i)
(1 + 7i)(3 − i) 10 + 20i 1 + 2i Now ∑99 k 96 k
k=1 i = ∑k=1 i + i
97
+ i98 + i99
= = =
5(9 + 1) 50 5 = 0 + i + i2 + i3 = i – 1 – i = –1
99
2 2
1 2 1 √5 1 + ∑ ik = 1 − 1 = 0
∴ |z| = √( ) + ( ) = √1 + 4 =
5 5 5 5 k=1
5 −10 ⇒ x = a2 − b2 , y = 2ab
= 1 (3 − ) + 1(2) + 1 ( )
3 3 ∴ √−x − iy = √−(a2 − b 2 ) − 2iab
4 10
= +2− ⇒ √−x − iy = √b 2 − a2 − 2iab
3 3
4 + 6 − 10 = √(b − ia)2 = ±(b − ia)
= =0
3
25. Ans (B)
Hence, area of triangle is zero, therefore points are
z−α
On comparing the given circle with | | = k, we
collinear. z−β
x 2 + i2 y 2 − 2xyi ⇒ |x + iy − i| = 1
=
x 2 − i2 y 2 ⇒ |x + (y − 1)i| = 1
2 2
x − y − 2xyi ⇒ x 2 + (y − 1)2 = 1
=
x2 + y2
Also, |z + 1| = 1
x2 − y2 2xy
= 2 2
− 2 i ⇒ |x + iy + 1| = 1
x +y x + y2
z‾
⇒ |(x + 1) + iy| = 1
When z lies in third quadrant then will also be lie
z ⇒ (x + 1)2 + y 2 = 1
in third
From Eqs. (i) and (ii), we get
x2 − y2 −2xy
∴ 2 < 0 & 2 <0 x = y = 0 and x = −1, y = 1
x +y 2 x + y2
∴ z = 0, −1 + i
x 2 − y 2 < 0&−2xy < 0 as x 2 + y 2 > 0
30. Ans (C)
⇒ x 2 < y 2 & xy > 0
Let z1 = 3 + pi
So, x < y < 0. 3
and z2 = + pi
27. Ans (A) 4
= |z + 3|2 ⇒ √9 + p2
28. Ans (C) 9
2√ + P2
|cos θ + isin θ| = |cos θ − isin θ| = 1 16
|1 − i√3| 2 1 On squaring both sides, we get
∴ |z| = = =
2|1 − i| 2√2 √2 9
9 + p2 = 4 ( + p2 )
29. Ans (A) 16
We have, |z − i| = |z + 1| = 1 9
⇒ 4P 2 − P 2 = 9 −
Let, z = x + iy 4
27 9 3
∴ |z − i| = 1 ⇒ 3P 2 = ⇒ P2 = ⇒ P = ±
4 4 2
1. Ans ( C) ⇒ x1 = −9, y1 = 9
1 and
x2 +3
= 9,
y2 +3
=6
× 1(k − 1) = ±1 2 2
2
⇒ x2 = 15, y2 = 9
k − 1 = ±2 ⇒ k = 3 & k = −1
−9+15+3 9+9+3
Now, centroid = ( , ) = (3,7)
3 3
Now, mid-point of AC
−2 + 3 −1 + 3 1 2 1
=( , ) = ( , ) = ( , 1)
2 2 2 2 2
and mid-point of BD
4−3 0+2 1
=( , ) = ( , 1)
2 2 2
⇒ ∠PAX = 180∘ − 60∘ = 120∘
⇒ Mid-point of AC = Mid-point of BD
∴ Slope of line AQ = m = tan 120∘
Hence, Statement II is true.
= tan (180∘ − 60∘ )
8. Ans (C)
Let slope of one line is m, then slope of another line = −tan 60∘ = −√3
is 2m. Given, the tangent of the angle between them 11. Ans (B)
1 Since, points are A(x, −1), B(2,1) and C(4,5) are
is tan θ = .
3
collinear.
m1 − m2 1
∴| |= Here, x1 = x, y1 = −1, x2 = 2, y2 = 1, x3 = 4 and
1 + m1 m2 3
1 m − 2m y3 = 5
⇒ =| |
3 1 + m × 2m Slope of AB = Slope of BC
1 −m y2 − y1 y3 − y2
⇒ =| | ⇒ =
3 1 + 2m2 x2 − x1 x3 − x2
⇒ (1 + 2m2 ) = 3m ⇒
1+1
=
5−1
(∵ slope of line =
y2 −y1
)
2−x 4−2 x2 −x1
2
⇒ 2m − 3m + 1 = 0
2 4
Factorize it by splitting the middle term ⇒ = ⇒2−x=1
2−x 2
⇒ 2m2 − 2m − m + 1 = 0 ⇒x=2−1=1
⇒ 2m(m − 1) − 1(m − 1) = 0 12. Ans (B)
⇒ (2m − 1)(m − 1) = 0 Let the incident ray strike X-axis at the point A
⇒ 2m − 1 = 0 whose coordinates be (x, 0). From the figure, the
1
or m − 1 = 0 ⇒ m = , m = 1 slope of the reflected ray is given by
2
3
9. Ans (D) tan θ = …. (i)
5−x
The inclination of the line (θ) lies within 0∘ to 180∘ Again, the slope of the incident ray is given by
∘ ∘
i.e., 0 ≤ θ ≤ 180 . −2
tan (π − θ) =
10. Ans (C) x−1
−2
Given, ∠YPQ = 30∘ or −tan θ =
x−1
To find slope of line AQ. Solving Eqs. (i) and (ii), we get
Here, ∠YPQ = ∠OPA
(vertically opposite angles)
∵ ∠OPA + ∠POA + ∠PAO = 180∘
(∵ sum of all angles of a triangle is 180∘ )
⇒ 30∘ + 90∘ + ∠PAO = 180∘
⇒ ∠PAO = 180∘ − 120∘ = 60∘ 3 2
=
5−x x−1
13
⇒x =
5
13. Ans (D) Since, the points A(1,6), B(3, −4) and C(x, y) are
Let A(x1 , y1 ) = (p + 1,1), B(x2 , y2 ) = (2p + 1,3) collinear.
and C(x3 , y3 ) = (2p + 2,2p) 1 6 1
∴ |3 −4 1| = 0
Points A, B and C are collincar if, x y 1
Slope of AB = Slope of BC ⇒1(-4-y)-6(3-x) +1(3y+4x) =0
3−1 2p−3
i.e., = ⇒ 10x + 2y − 22 = 0
(2p+1)−(p+1) (2p+2)−(2p+1)
2 2p − 3 ⇒ 5x + y − 11 = 0
⇒ =
p 1 17. Ans (C)
2
⇒ 2p − 3p − 2 = 0 Slope of the line 3x + y = 3 is -3.
⇒ (2p + 1)(p − 2) = 0 Slope of any line perpendicular to the line 3x + y =
−1 −1 1
⇒p= ,2 3 is =
2 −3 3
1
14. Ans (A) Now, equation of line with slope and passing
3
We know that the acute angle θ between two lines through (2,2) is y − 2 = (x − 2)
1
3
with slopes m1 and m2 is given by
⇒ x − 3y + 4 = 0
m2 − m1
tan θ = | | x y
1 + m1 m2 ⇒ + =1
−4 4
1 3
Let m1 = ,
2 4
Hence, its y-intercept is .
m2 = m and 3
θ= .
π 18. Ans (B)
4
Any line parallel to 2x − 3y = 4 is
Now, putting these values in (i), we get
2x − 3y + λ = 0
1
π m− λ λ
tan = | 2|
4 1 A ≡ [− , 0] , B ≡ [0, ]
1+ m 2 3
2
1 λ λ
2m−1 area of △ AOB = |( ) ( )|
⇒1=| |, 2 2 3
2+m
2
2m−1 2m−1 = 12λ = 144
which gives = 1 or = −1
2+m 2+m
⇒ λ = 12, −12
1
Therefore m = 3 or m = − line is 2x-3y+12=0 and 2x-3y-12=0
3
1
Hence, slope of the other line is 3 or − .
3
λ−3 1
=− ⇒λ=4
−2 2
27. Ans (D)
co-ordinates of point D are A-B+C= (4,7)
⇒ line AD is 5x − 3y + 1 = 0
30. Ans (B)
28. Ans (C)
1−0
Equation of line in intercept form is The slope of the line AB is = 1 or tan 45∘ . After
3−2
x y rotation of the line through 15∘ , the slope of the line
+ =1
a b
AC in new position is tan 60∘ = √3.
Given, a + b = 9
Therefore, the equation of the new line AC is
and line (i) passes through the point (2,2), it will
y − 0 = √3(x − 2) or y − √3x + 2√3 = 0
satisfy the line (i)
i.e., put x = 2, y = 2 in Eq. (i)
2 2
⇒ + =1
a b
Solve Eqs. (ii) and (iii), to find the values of a and b.
From Eq. (ii), b = 9 − a put in Eq. (iii), we get
2 2
+ =1
a 9−a
⇒ 2(9 − a) + 2a = a(9 − a) 31. Ans (B)
⇒ a2 − 9a + 18 = 0 x − 7y + 5 = 0
⇒ a(a − 6) − 3(a − 6) = 0 −5 15
x= ,y =
22 22
⇒ (a − 6)(a − 3) = 0 5 15
Hence, the intersection point is (− , ).
⇒ a = 6 or 3 ⇒ b = 3 or 6 22 22
1 a2 c2
∴ m = tan 90∘ = ) and y = − x− …. (ii)
b2 b2
0
a1
⇒ 22x + 5 = 0 Slopes of the lines (i) and (ii) are m1 = − and
b1
a
32. Ans (A) m2 = − 2 , respectively.
b2
−a
Slope of the line ax + by = c is , Now,
b
−a′ I. Lines are parallel, if m1 = m2 , which gives
Also, slope of the line a′ x + b′ y = c ′ is .
b′
a1 a2 a1 a2
The lines are perpendicular if − =− or =
b1 b2 b1 b2
satisfy the equation (i) so that Consider the given equation of lines,
h+1 k+2 x − (t + α) = 0
− 3( ) + 4 = 0 or h − 3k = −3
2 2
y + 16 = 0
Solving (ii) and (iii),
6 7
and −αx + y = 0
we get h = and k = .
5 5 Since, these lines are concurrent, therefore the
Hence, the image of the point (1,2) in the line (i) is system of equations is consistent.
6 7
( , ). 1 0 −(t + α)
5 5
Now, | 0 1 16 | = 0
Alternatively, use T.S.R −α 1 0
34. Ans (B) ⇒ 1(0 − 16) − (t + α)(0 + α) = 0
3×1+3−9 3
Ratio = − = (Internally) ⇒ −16 − α(t + α) = 0
3×2+7−9 4
By seeing the equation of the sides, we can easily Here, A = 3, B = −4, C1 = 7, and C2 = 5. Therefore,
y = 0 in 3x − 4y = 0, ⇒ 12x + 72 = 5y − 10
So, line 3x − 4y = 0 passes through orthocentre. Hence, distance from the point (−1,1) is
This point lies on the given line, so we have 44. Ans (A)
h+4 k − 13 △ ABC is an equilateral triangle. AD is altitude.
5( )+ +6=0
2 2 Equation of BC is 2x − y = 1
or 5h + k + 19 = 0 So perpendicular distance,
Again, the slope of the line joining points (h, k) and 2(−1) − 2 − 1
k+13
AD = | |
(4,13) is given by . This line is perpendicular to √4 + 1
h−4
k+13 = | − √5| = √5
the given line and hence (−5) ( ) = −1
h−4
This gives 5k + 65 = h − 4
or h − 5k − 69 = 0 … ( ii )
On solving (i) and (ii), we get h = −1 and k = −14.
Thus, the point (−1, −14) is the reflection of the
given point.
41. Ans (D)
AD
The given lines are In △ ABD, = sin 60∘
AB
x − 3y + 5 = 0 √5 √3
⇒ =
3x + 4y = 0 AB 2
5x − 2y = 0 2√5 20
∴ AB = =√ units
√3 3
20 15
Solving (i) and (ii), we get (− , ) as their
13 13 45. Ans (A)
intersection point.
MATHEMATICS Page | 6A. 7
STRAIGHT LINE
Given line is 4x − y = 0. 5
4−
d= 2 = 3 = 3√2 > 1
In order to find the distance of the line from the 4
√1 + 12 2√2
2
point P(4,1) along another line, we have to find the Hence, no point lies in it
point of intersection of both the lines. 48. Ans (D)
For this purpose, we will first find the equation of k
+1
∘ According to the question, |2 | = 2
the second line. Slope of second line is tan 135 = 13
⇒ 4x + 3y − 12 = 0
Now, distance from origin is
4×0+3×0−12 12
| |= unit
√32 +4 2 5
or x + y =
5 ⇒ y + 3 = ±6
2
⇒ y = 3( or ) − 9
The distance between two parallel lines,
∴ y1 + y2 = 3 − 9 = −6
3 2 √10
52. Ans (A) AB = √(1 − 0)2 + (3 − ) =
8 3
√a2 (cos θ − cos ϕ)2 + a2 (sin θ − sin ϕ)2 = 2a
BC = √(82 − 1)2 + (30 − 3)2 = 27√10
2 (cos 2 2 2
⇒a θ + cos ϕ − 2cos θcos ϕ + sin θ
+ sin2 ϕ − 2sin θsin ϕ) = 4a2 8 2 82√10
CA = √(82 − 0)2 + (30 − ) =
3 3
⇒ 1 + 1 − 2cos θcos ϕ − 2sin θsin ϕ = 4
Clearly AB + BC = CA
⇒ 2 = −2(sin θsin ϕ + cos θcos ϕ)
∴ A, B, C are collinear
⇒ −1 = cos (θ − ϕ)
59. Ans (C)
⇒ θ − ϕ = 2nπ ± π
1 x1 − x 3 x2 − x4
⇒ θ = 2nπ ± π + ϕ Area of Quadrilateral = |y − y y2 − y4 |=18
2 1 3
53. Ans (D) 60. Ans (D)
A = (0, b) on y − axis x1 +x2 +x3 y1 +y2 +y3 19
Centroid G = ( , ) = (5, )
3 3 3
B = (4,5)
61. Ans (B)
AB = C
O = (0,0), A = (a, 0), B = (0, b)
√16 + (b − 5)2 = c
O, A, B are form a right angle triangle
16 + (b − 5)2 = c 2
Right angle at O and circumcentre is midpoint of AB
(b − 5)2 = c − 16 < 0 not possible a b
Thus, circumcentre S = ( , ) and orthocentre is
54. Ans (D) 2 2
0=(0, 0)
A = (x1 , y1 ) = (−7,1), B = (x2 , y2 ) = (3,6)
P = (x, y) = (−1,4) a2 b 2 1 2
∴ OS = √ + = √a + b 2
P divides ̅̅̅̅
AB in the ratio x1 − x: x − x2 4 4 2
= −7 + 1: −1 − 3 62. Ans (C)
= −6: −4 = 3: 2 Given D = (−3, −5) and E = (3,3)
55. Ans (B) ∴ BC = 2DE
Let A(2, −3), P(10, y) = 2√36 + 64 = 20
Given AP = 10
A = (9,5)
⇒ xa + xb + ya − yb = xa − xb + ya + yb
⇒ 2xb = 2yb ⇒ x = y ⇒ x − y = 0
73. Ans (B) (ii) Translation 3 units along the negative y-axis =
PA = PB (−2,0)
⇒ PA2 = PB 2
⇒ [x − (a + b)]2 + [y − (a − b)]2
= [x − (a − b)]2 + [y − (a + b)]2
⇒ x 2 + (a + b)2 − 2x(a + b) + y 2 + (a − b)2 − 2y(a
− b)
= x 2 + (a − b)2 − 2x(a − b) + y 2 + (a + b)2 + 2y(a
+ b)
⇒ −2x(a + b) − 2y(a − b)
= −2x(a − b) + 2y(a + b) 78. Ans (B)
m = −1 ⇒ tan θ = −1 ⇒ 3x + 2y = 0
π 3π
⇒θ=π− ⇒θ=
4 4
82. Ans (B)
A = (0, b), B = (0,0), C = (a, 0)
a a b
D = ( , 0) , E = ( , )
2 2 2 86. Ans (D)
̅̅̅̅ ̅̅̅̅
AD ⊥ BE P=(2,3), A=(-5,6) and B = (−6,5)
̅̅̅̅ × slope of BE
Slope of AD ̅̅̅̅ = −1 1
Slope of ̅̅̅̅
AB = = 1
1
b b/2
⇒ × = −1 ̅̅̅̅ = −1
Slope of the line perpendicular to AB
(−a/2) a/2
−2b2 Required Equation, y − 3 = −1(x − 2)
⇒ = −1
a2 ⇒x+y=5
2 2
⇒ a = 2b 87. Ans (C)
⇒ a = ±√2b mid point of ̅̅̅̅
AB = P(2,2)
−1 −1
Slope of ̅̅̅̅
CD = = = −3
̅̅̅̅ 1/3
Slope of AB
∴ eqn. of ̅̅̅̅
CD y − 2 = −3(x − 2)
⇒ y − 2 = −3x + 6
⇒ 3x + y = 8 ⇒ 3x + y − 8 = 0
⇒ a = 2b ⇒ x1 + y1 = 2
equation of the line in the intercept form is
x y
+ =1
a b
x y
⇒ + =1
2b b
⇒ x + 2y = 2b
4 − 2 = 2b
b=1
100. Ans (B)
required line x + 2y = 2 4
Required line = y − 2 = 11 (x − )
97. Ans (D) 3
a + b = −1 11
⇒y−2= (3x − 4)
3
The equation of the straight lines in the intercept
⇒ 3y − 6 = 33x − 44
form is
x y ⇒ 33x − 3y − 38 = 0
+ =1
a b
4 3
+ =1
a b
⇒ 4b + 3a = ab 101. Ans (A)
4(−1 − a) + 3a = a(−1 − a) P = (3,2)
−4 − 4a + 3a = a = a2 ̅̅̅̅ is y − y1 = m(x − x1 )
Equation of BD
a2 = 4 ⇒ y − 2 = 2(x − 3)
a = ±2 ⇒ 2x − y − 4 = 0
case (i): a = 2, b = −3
x y
− =1
2 3
case (ii): a = −2, b = 1
x y
+ =1
−2 1
98. Ans (C) 102. Ans (B)
A = (a, 0), B = (0, b) x-y=a
Let P = (x1 , y1 ) is midpoint of ̅̅̅̅
AB (3, -1)⇒ a = 4
a b ∴ Equation of the line line x − y = 4
⇒ (x1 , y1 ) = ( , )
2 2
103. Ans (A)
⇒ a = 2x, b = 2y
1−0
∴ a + b = 10 Slope of ̅̅̅̅
AB = =1
3−2
⇒ x1 + y1 = 5 ⇒ tan θ = 1 ⇒ θ = 45∘
Locus of P is x + y = 5 ∴ Slope of ̅̅̅̅
AC = tan 60∘ = √3
99. Ans (C) and eqn. of ̅̅̅̅
AC is y − 0 = √3(x − 2)
̅̅̅̅. The locus of P is
Let P(x1 , y1 ) is the midpoint of AB
y = √3x − 2√3
x y
+ =2
x1 y1 ⇒ √3x − y = 2√3
⇒ a = 2x1 , b = 2y1
⇒a+b =4
⇒ 2x1 + 2y1 = 4
MATHEMATICS Page | 6A. 14
STRAIGHT LINE
x
The equation of the line having intercepts a, b is +
a
y
=1
b
x y
⇒ + =1
8 9
⇒ 9x + 8y = 72
104. Ans (C) 108. Ans (C)
−π A = (3,3), B = (7,6)
P = 10, α =
4
Equation of ̅̅̅̅
AB is 3x − 4y = −3
eqn. of line xcos α + ysin α = P x y
x y ⇒ + =1
− = 10 −1 (3/4)
√2 √2
9 5
⇒ x − y = 10√2 ∴ AB = √1 + =
16 4
105. Ans (B)
A line meets x-axis at P and y-axis at Q
Let OP = a, OQ = b
⇒ P = (a, 0), Q = (0, b)
A divide ̅̅̅̅
PQ in the ratio 1: 2
2a b
⇒ A = (4,3) = ( , )
3 3
12 109. Ans (C)
⇒a= , b = 9, ⇒ a = 6, b = 9
2 −x − y = √2
x y
The equation of PQ is + =1 −1 −1
a b
⇒ ( )x + ( )y = 1
x y √2 √2
+ = 1 ⇒ 3x + 2y = 18
6 9 5π 5π
⇒ xcos + ysin =1
4 4
110. Ans (D)
2x − 3y = −6
x y
⇒ + =1
−3 2
x y
from given line + =1
−8 −8
( ) ( )
a b
106. Ans (A)
−8 −8
−√3x + y = 8 = 3, = −2
a b
divide √3 + 1 = 2 −8
⇒a= ,b = 4
3
−√3x 1
+ y=4 111. Ans (C)
2 2
cos (150∘ )x + sin 150∘ y = 4 (cos θ + sin θ)x + (sin 2θ)y = 1
1 ⇒ k 2 = 10 × 36
= − [1 + ] = −1.574
√3 ⇒ k = ±6√10
⇒ [K] = −2
⇒required number of lines = 2
112. Ans (A)
√3x + y = 1 → (1)
Slope of (1) is m1 = −√3
Let slope of the required line m2 = m
m1 −m2
Thus, tan θ = | |
1+m1 m2
−√3 − m m + √3
√3 = | | =| |
1 − √3m 1 − m√3 115. Ans (B)
2x + 6y − 7 = 0 → (1) ⇒ x + 7y = 31
The equation of the line parallel to (1) is
2x + 6y = k
k
x-intercept =
2
k
y-intercept =
6
⇒ 9y + 9 = −2x + 2 or 2x + 9y + 7 = 0
̅̅̅̅ m2 = m
Let slope of AB
∠BAC = 450
m1 − m2
⇒ tan 45∘ = | |
1 + m1 m2
1
−m 123. Ans (D)
⇒ 1 = |3 m|
1+ Equation of AB is x. cos α + y. sin α=p
3
x cosα y sinα
⇒ 3 + m = ±(1 − 3m) ⇒ + =1
p p
⇒ 3 + m = 1 − 3m (or) 3 + m = −1 + 3 m
x y sinα
⇒ 4 m = −2 (or) 2 m = 4 ⇒ p + p =1
( ) ( )
cosα sinα
−1
⇒m= (or)m = 2 p p
2 So co-ordinates A and B are ( , 0) and (0, );
cos α sin α
The required equation of the line passing through ' A So, coordinates of midpoint of AB are
' p p
( , ) = (x1 , y1 )
−1 2cos α 2sin α
y−3 = (x − 1)(or)y − 3 = 2(x − 1) p p
2 Let x1 = , y1 =
2cos α 2sin α
⇒ x + 2y − 7 = 0(or)2x − y + 1 = 0
1−6
126. Ans (A) ∴ equation of BC is y − 6 = (x − 6)
2−6
129. Ans (C) The equation of the diagonal not passing through
Required distance A, B is
y − 2 = −3(x − 2)
0 + 0 + 25 25
OP = | |=| |=5 ⇒ 3x + y − 8 = 0 → (1)
√32 + 42 5
The perpendicular distance from (1,2) to (1)
By Pythagoras theorem AP = PB = 12 |3 + 2 − 8| 3
= =
1 √10 √10
So, Area of △ OAB = × 24 × 5 = 60
2
133. Ans (B)
x y
Given line equation is + = 1, passing through
5 b
(13, 32)
13 32
⇒ + =1
5 b
we get b = −20
x y
thus, line L is − = 1 ⇒ 4x − y = 20 ... (1)
5 20
130. Ans (C) ⇒ Slope = 4.
3x + 4y + 7 = 0 x y
Now, the line K: + = 1
c 3
⇒ 6x + 8y + 14 = 0 → (1) −3
Its slope is
and 6x + 8y + k = 0 → (2) c
−3 −3
The distance between (1) and (2) = 4 ∴ =4⇒c=
c 4
|14 − k|
⇒ =4 Thus K: 4x − y = −3
√36 + 64
The lines are
⇒ |14 − k| = 40
4x − y = 20
⇒ 14 − k = ±40
4x − y = −3
⇒ k = 54 (or ) − 26
|−20−3| 23
131. Ans (A) Required distance = =
√16+1 √17
2x + 3y + 4 = 0 → (1) y − y1 = m(x − x1 )
A = (1,2) = (x1 , y1 ) 3
⇒ y+3= (x − 0)
5
B = (α, β) is the image of a with respective (1)
⇒ 5y + 15 = 3x
α − 1 β − 2 −2(ax1 + by1 + c)
= = ⇒ 5y − 3x + 15 = 0
2 3 a2 + b 2
α − 1 β − 2 −2(12) −24 162. Ans (A)
= = =
2 3 13 13 Intercept form of a line is
48 −72 x y
α= + 1, β = +2 + =1
13 13 a b
−120 + 39 −81 x y
α+β= = + =1 [∵ a = b
a a
13 13
⇒ x+y= a
⇒ y = −x + a
∴ Slope is −1
163. Ans (B)
Equation of line ' l ' is given by
y − y1 = m(x − x1 ).
Since ′l′ passing through the point P(3,2).
159. Ans (B) ∴ y − 2 = m(x − 3)
P = (4, −1) ⇒ y = mx + 2 − 3m
y = x → (1) Since it is given that lines y = x and ' l ' are
PQ = 2 × ( The perpendicular from P to perpendicular to each other,
x − y = 0) ∴ m × 1 = −1[∵ m1 × m2 = −1]
|4 + 1| Put m = −1 in eqn. (i), we get
= 2( ) = 5√2
√2 y = −x + 2 − 3(−1)
y = −x + 5
x+y=5
√3x + y + 1 = 0 y = mx + c1 and y = mx + c2
⇒ y = −√3x − 1 Since slopes of eq. (i) and eq. (ii) are same, they are
parallel lines.
∴ Slope of this line, m1 = −√3
MATHEMATICS Page | 6A. 24
STRAIGHT LINE
From figure, we get that OABC is square having ∵ Distance between two
[ parallel lines = | c1 − c2 |]
corners O(0,0), A(1,0), B(1,1) and C(0,1) Equation of
√a2 + b 2
diagonal AC: Distance between parallel lines (ii) and (iii) we get
1−0
y-0= (x − 1) ⇒ y + x = 1 −5 − 2 7
0−1
| |=
1−0 2
√(3) + (4)2 5
Equation of diagonal OB is y= (x − 0) ⇒y=x
1−0
3 7
∴ Ratio between the distances = : = 3: 7
175. Ans(B) 5 5
Since, they are perpendicular to each other Given 3x-4y+2=0 and P =(-2,3)
y1 The equation of the line parallel to above line and
∴ − 1 × = −1 ⇒ y1 = x1
x1 passing through P is 3x-4y=-6-12
From eq. (i) and (ii) we get x1 + 2a=0 ⇒3x-4y+18=0
⇒ 2a = −x1 187. Ans (A)
y1 + 2b = 0 ⇒ 2b = −y1 x + y = a ⇒ y = −x + a
From eq. (iv) we get,a + b − 2 = 0 Thus, m=-1⇒ θ = 135°
a+a−2=0
a = 1 and b = 1 [∵ a = b]
∴ x1 = −2 × 1 = −2
y1 = −2 × 1 = −2
181. Ans (D)
A = (5,0), B = (0,3) 188. Ans (D)
x y m1 m2 = −1
Equation of ̅̅̅̅̅
AB is + = 1
5 3 17 − β 2
3x + 5y = 15 → (1) × = −1 ⇒ β = 5
−8 3
The ⊥er distance from (4,4) to (1) 189. Ans (D)
|12 + 20 − 15| 17 Since the given lines are perpendicular.
= =
√9 + 25 √34 ∴ Product of their slopes = −1
17 −l −l′
=√ ⇒ × = −1 ⇒ ll′ = −mm′
2 m m′
⇒ ll′ + mm′ = 0
182. Ans (C)
190. Ans (B)
Given P = (2,2)
Put θ = 45° and solve using point slope form
3x + y − 3 = 0 → (1)
191. Ans (B)
The eq. of the line perpendicular to (1) passing
x − 3y = −4
through P is x − 3y = −4
⇒ x = 0 ⇒ y = 4/3
MATHEMATICS Page | 6A. 26
STRAIGHT LINE
Note that, for given two lines = m1 × m2 = −1 The coordinates of the point dividing the line joining
6 + 7 −1 + 3 13 ∴ Equation of line is
( , ) = ( , 1)
2 2 2 x − 3y = −4 …(i)
2−1 2 −4 4
∴ Slope of PS = =− Hence, y-intercept = =
13 9 −3 3
2−
2
22. Ans (C)
Given equation of lines are
5x + 3y − 7 = 0 …(i)
and 15x + 9y + 14 = 0
14
⇒ 5x + 3y + = 0 …(ii)
3
c1 − c2 35
d=| |=
√a2 + b 2 3√34
The required equation which is 23. Ans (B)
AB = √22 + 1 = √5 diameter.
2. Ans (B)
5. Ans (A)
We have the equation of circle
The point of intersection of
x 2 + y 2 + 2gx + 2fy + c = 0
3x + y − 14 = 0 and 2x + 5y − 18 = 0 are
But it passes through (0,0)
x = 4, y = 2, i.e., the point (4,2)
and (2,1), then c = 0.
Therefore, the radius is = √9 + 16 = 5 and
5 + 4g + 2f = 0
hence the equation of the circle is given by
As it touches y-axis
(x − 1)2 + (y + 2)2 = 25
or x 2 + y 2 − 2x + 4y − 20 = 0
6. Ans (C)
Let (h, k) be the centre of the circle. Then k =
h − 1.
Therefore, the equation of the circle is given by
thus 2√f 2 − c = 0[∵ c = 0] (x − h)2 + [y − (h − 1)]2 = 9
⇒ f = 0, Given that the circle passes through the point
∴g=−
5 (7,3) and hence we get
4
(7 − h)2 + (3 − (h − 1))2 = 9
3. Ans (B)
or (7 − h)2 + (4 − h)2 = 9
Centre (2,2), r = 2 so touches both axes
or h2 − 11h + 28 = 0
4. Ans (B)
which gives (h − 7)(h − 4) = 0
Given equation is
⇒ h = 4 or h = 7
x 2 + y 2 − 3x + 5y − 4 = 0.
Therefore, the required equations of the circles
It can be easily seen that (i)
are
represent a circle with centre
3 5
x 2 + y 2 − 8x − 6y + 16 = 0
C ( , − ).
2 2 (Substituting h = 4,7 in (i) )
or x 2 + y 2 − 14x − 12y + 76 = 0
MATHEMATICS Page |7A. 1
CONIC SECTIONS
the circle is (1, −2) and radius is √13. 13. Ans (A)
with a = 2. Hence the required equation is y 2 = x-coordinate is zero. Hence, equation of ellipse
x2 y2
4(2)x = 8x. will be of the form + =1
b2 a2
24. Ans (A) Given that, length of minor axis 2b = 16 and c =
Given, vertex = (0,0) 6 ⇒ b = 8 and c = 6
Point = (2,3) and axis = X-axis ∵ c 2 = a2 − b2 ⇒ (6)2 = a2 − (8)2
Since, point (2,3) lies in first quadrant and axis is ⇒ a2 = 36 + 64 = 100
X-axis. Hence, equation of parabola will be of the Put the values of a2 = 100 and b2 = 64 in Eq. (i),
form y 2 = 4ax, which passes through (2,3) i.e., x2 y2
we get + =1
64 100
Put x = 2, y = 3 in y 2 = 4ax
29. Ans (A)
∴ (3)2 = 4a × (2)
Since, foci (±3,0) lie on X-axis, as y-coordinate is
9
⇒a= zero.
8
Hence, required equation of parabola is Hence, equation of ellipse will be of the form
9 9 x2 y2
y2 = 4 ( ) x ⇒ y2 = x + =1
a2 b2
8 2
Given that, foci (±c, 0) = (±3,0)
25. Ans (B)
c = 3 and a = 4
(2)2 − 4(3) < 0
∵ c 2 = a2 − b2 ⇒ (3)2 = (4)2 − b2
26. Ans (C)
⇒ 9 = 16 − b2 ⇒ b2 = 16 − 9 ⇒ b2 = 7
9x 2 + 4y 2 = 1
Put the values of a2 = 16 and b2 = 7 in Eq. (i),
x y2
⇒ + =1 x2 y2
1/9 1/4 we get + =1
16 7
2a2 4
⇒ Length of latus rectum = = 30. Ans (B)
b 9
2b2
27. Ans (B) According to given condition, = 2ae
a
x2 y2
+ =1 b2
r−2 5−r ⇒ b2 = a2 e or e =
a2
1 √3
As, e1 e2 = ⇒ e2 = b2
2 2
∴ e = √1 +
3 b2 a2
Now, = 1 − ⇒b=2
4 16
5 2 b2 5
34. Ans (A) ⇒ ( ) = (1 + 2 ) [∵ e = ]
4 a 4
We have d = F1 F2 = 2ae
b2 9 b 3
d2 = 4a2 e2 = 4(a2 − b2 ) ⇒ = ⇒ =
a2 16 a 4
= 4(4 − 1) = 12 2b2 3
We have, =9⇒ 2× b =9
a 4
⇒ d = 2√3
⇒ b = 6 and a = 8
35. Ans (A)
x2 y2
∴ Equation of hyperbola is − =1
I. Since, ends of major axis (±3,0) are along X- 64 36
axis and minor axis (0, ±2) are along Y-axis. 38. Ans (D)
x2 y2
Hence, equation of ellipse will be of the torm Given that the hyperbola − − 1 is passing
a2 b2
x2 y2
+ = 1, where, a = 3 through the points (3,0) and (3√2, 2), so we get
a2 b2
and b = 2 a2 = 9 and b2 = 4.
x2 y2 Again, we know that b2 = a2 (e2 − 1). This gives
Hence, Eq. (i) becomes + =1
32 22
13
x2 y2
4 = 9(e2 − 1) or e2 =
9
or + =1
9 4
√13
or e =
II. Since, ends of major axis (0, ±√5) are along Y- 3
axis and minor axis (±1,0) are along X-axis. 39. Ans (C)
3 9
ae = 3, e = , b2 = 4 ( − 1) , b2 = 5
2 4
MATHEMATICS Page |7A. 5
CONIC SECTIONS
x2 y2 x2 y2
− =1 − + =1
4 5 4 3
The eccentricity of this hyperbola is given by
a2 4 7
e = √1 + 2
= √1 + = √
b 3 3
Since foci are (0, ±12), it follows that Now, for an ellipse, ae = 3 or a2 e2 = 9
c = 12. Now, b2 = a2 (1 − e2 )
2b2 ⇒ b2 = a2 − a2 e2 = 16 − 9 = 7
Length of the latus rectum = = 36
a
46. Ans(A)
2
⇒ b = 18a.
x 2 + y 2 − 2x + 3y + k = 0
2 2 2
Therefore, c = a + b gives 5
We have r=√g 2 + f 2 − c =
2 2 2
144 = a + 18a i.e., a + 18a − 144 = 0
9 25 13 25
⇒ a = −24,6. ⇒1+ −k= ⇒ −k=
4 4 4 4
Since, a > 0 we take a = 6 and so ⇒ k = −3
b2 = 108. 47. Ans(C)
Therefore, the equation of the required
hyperbola is
y2 x2
− = 1 ⇒ 3y 2 − x 2 = 108
36 108
42. Ans (D)
144 9 7
e12 = 1 − × = B = 2C − A = (4,2) − (3,4) = (1, −2)
16 144 16
44 9 25 48. Ans(A)
e22 = 1 + × =
16 144 16 Circle with ̅̅̅̅
AB as diameter
⇒ e12 + e22 =2 x 2 + y 2 − 4x + 3y = 0
43. Ans (D)
2b = 5 and 2ae = 13
25 169
b2 = a2 (e2 − 1) ⇒ = − a2
4 4
13
⇒a=6⇒e=
12
44. Ans (A)
The given equation can be written as 49. Ans(A)
A = (4,0), B = (0,3)
MATHEMATICS Page |7A. 6
CONIC SECTIONS
̅̅̅̅ as diameter is
Required Circle with AB
x 2 + y 2 − 4x − 3y = 0
C = (±5,3), r = 5
∴ Equation of circle is
(x ± 5)2 + (y − 3)2 = 25
⇒ x 2 + y 2 ± 10x − 6y = 0
Given C1 = (2,2) , r1 =2 56. Ans(D)
and C2 (6,5) , r2 =? πr 2 = 4π ⇒ r = 2
By the given data C1 C2 =r1 + r2 we have (x − r)2 + (y − r)2 = r 2
⇒ √(16 + 9)=2+r2 ⇒2+r2 =5⇒ r2 =3 ⇒ (x − 2)2 + (y − 2)2 = 4
∴ Centre C2 =(6,5) & r2 =3 57. Ans(C)
2 2
⇒ x + y − 12x − 10y + 52 = 0 3x − 2y − 8 = 0 → (1)
53. Ans(A) 2x − y − 5 = 0 → (2)
2 −1
after solving, center C = ( , ) = (2, −1)
1 1
⇒r=1
58. Ans(B)
Equation of circle passing through the points
(0,0), (4,0) and (0,5) is x 2 + y 2 − 2ay = 0
5 2 25 C = (a, 0)
(x − 2)2 +(y − ) =4+
2 4
x + 2y = 4 → (1)
x 2 +y 2 -4x-5y=0
required line is ⊥er to (1) passing through C
2x − y = 2a
63. Ans(D)
S ≡ x 2 + y 2 − 2x − 4y + 4 = 0
P = (1,2)
S(1,2) = 1 + 4 − 2 − 8 + 4 = −1 < 0
C (-2, -3)⇒ x 2 + y 2 + 4x − 2y + k = 4 + 9 − 8 + s1 ≡ x 2 + y 2 − 6x − 8y + c = 0
6+k = 0 c1 = (3,4), r1 = √9 + 16 − c = √25 − c
⇒ 11 + k = 0 ⇒ k = −11 Also given s2 ≡ x 2 + y 2 = 9
∴ option B is correct c2 = (0,0), r2 = 3
60. Ans(B) Two circles touch externally,
x 2 + y 2 = K/5 so, c1 c2 = r1 + r2
K ⇒ 5 = √25 − c + 3 ⇒ 4 = 25 − c ⇒ c = 21
C = (0,0), r = √
5 65. Ans(D)
x + 2y + 3 = 0 → (1) Given S1 ≡ x 2 + y 2 − 2x − 4y − 20 = 0
C2 = (12,5), r2 = √144 + 25 − a2
= √169 − a2
For exactly two tangents
|r1 − r2 | < c1 c2 < r1 + r2
Given circle is x 2 + y 2 − 4x + 6y − 7 = 0
C = (2, −3), r = √4 + 9 + 7 = √20
Image of centre C(2, -3) w.r.t X- axis is
C ′ = (2,3), r ′ = √20 74. Ans(C)
2 2
∴ 𝑟𝑒𝑞𝑢𝑖𝑟𝑒𝑑 𝑐𝑖𝑟𝑐𝑙𝑒: (x − 2) + (y − 3) = 20 −9
2 2 x2 = y
⇒ x + y − 4x − 6y − 7 = 0 2
69. Ans(B) 4a = −9/2
C = (−8, −9) and r = √64 + 81 − 85 = √60 And equation of directrix is y=-a⇒ y = 9/8
directrix L ≡ 3x − 4y = 2 x2
= 4ax ⇒ x = 12a ⇒ y = 4√3𝑎
3
2a = The ⊥er dis tance from F to L = 0
∴ PQ: 2y = 8√3𝑎
|9 − 12 − 2|
⇒ 2a = =1
5
∴ The length of latus rectum = 4a = 2
77. Ans(B)
option B satisfies both tangent and parabola
78. Ans(D)
Given y 2 = 8x ⇒ a = 2
Let P = (x1 , y1 ) be a point on the parabola 82. Ans(B)
⇒ |x1 + a| = 8 L. L. R = 4a
⇒ x1 = 6, −10 = 4|5| = 20
S ≡ y 2 = 4x y 2 = −4ax
SP = PM 91. Ans(C)
⇒ x 2 + (y + 3)2 = (y − 3)2 Focus S = (3,0) = (a, 0)
⇒ x 2 + 12y = 0 directrix x = −3 ⇒ a = 3
∴ Equation of the parabola y 2 = 4ax
⇒ y 2 = 12x
92. Ans(B)
S ≡ x 2 = 4y ⇒ a = 1
Equation of latus rectum y = −a
86. Ans(B)
L≡y+1=0
Given Focus S(−4,0) and directrix x = 4
P = (2a, a) lies inside S = 0
⇒ a = −4
S(2a,a) < 0
We have y 2 = −4ax
4a2 − 4a < 0 ⇒ a(a − 1) < 0
∴ y 2 = −16x
⇒ a ∈ (0,1)
87. Ans(B)
P, O lies on same side of L = 0
a + 1 > 0 ⇒ a > −1
⇒0<a<1
93. Ans(C)
S ≡ y 2 = 12x
95. Ans(C)
8−6 1
S ≡ y 2 = 24x e=√ =
8 2
⇒a=6
foci = (0, ±be)
P = (6,12) = (a, 2a)
2√2
Q = (6, −12) = (a, −2a) = (0, ± ) = (0, ±√2)
2
are ends of latus rectum of " S=0
102. Ans(A)
⇒ Angle b/w the normal =π/2
x2 y2
96. Ans(C) + =1
cot 2 α cos 2 α
Focus S(−1,0) 1
LLR =
2
m = −√3
2cos 2 α 1
y − 0 = −√3(x + 1) =
cot α 2
⇒y + √3(x + 1) = 0 1
⇒2cos 2 αtanα =
2
97. Ans(C) 1 1
⇒2sinαcosα = ⇒sin 2α =
2 2
x + y = 0 ⇒ y = −x
π π
⇒2α = ⇒α=
x 2 + x 2 − 4x = 0 6 12
⇒2x(x − 2) = 0
(0,0)(2, −2) 103. Ans(B)
x2 y2 3 5
+ =1 (l, m) = ( , )
36 9 32 32
SP + S ′ P = 2a(a > b) = 12 ∴ 3x 2 + 5y 2 = 32
2b = a 4x 2 + 16y 2 = 64
x2 y2
a2 −b2 + =1
We have e = √ 16 4
a2
SP + S ′ P = 2a(a > b) = 8
4b 2 − b 2 √3
=√ = 114. Ans(B)
4b 2 2
F = (4,0) = (ae, 0)
108. Ans(D)
4
e=
5
ae = 4 ⇒ a = 5
we have b2 = a2 (1 − e2 )
= 25 − 16 = 9
x2 y2
∴ + =1
25 9
ae 115. Ans(D)
Tan 30∘ =
b case (i) a < b
1 a̸
⇒ = 2a = 6 and 2be = 8
√3 a̸√1−e2
⇒a = 3 and be = 4
⇒1 − e2 = 3e2
1 We have a2 = b2 (1 − e2 )
⇒4e2 = 1 ⇒ e =
2
⇒9 = b2 − 16 ⇒ b2 = 25
109. Ans(B)
x2 y2
∴ + =1
2 a2 − b 2 2 9 25
e= ⇒√ =
3 a2 3 case (ii)a > b
b 2 4
2b = 6 ⇒ b = 3
⇒1 − ( ) =
a 9 ⇒2ae = 8 ⇒ ae = 4
5 b 2 a 3
⇒ =( ) ⇒ = We have b2 = a2 (1 − e2 )
9 a b √5
110. Ans(C) ⇒9 = a2 − 16 ⇒ a2 = 25
x2 y2
x2 y2 ⇒ + =1
+ =1 25 9
25 16
116. Ans(B)
⇒a = 5, b = 4
PS1 + PS2 = 2a = 2(5) = 10
111. Ans(A)
x2 y2
+ =1
25 16
Here a = 5, b = 4
PF1 + PF2 = 2a = 2(5) = 10
121. Ans(A)
Focus = (c, 0) = (4,0) ⇒ c = 4
4
eccentricity =
5
c 4 4 4
ae = ⇒ = ⇒a=5
From the figure Tan 450 = a 5 a 5
b
Now, c = a − b22 2
b = ae
x2 y2
∴Equation of ellipse is + =1
we have a√1 − e2 = ae 25 9
⇒e = =
c 3 ⇒a = 5
a 5
S = (4,0), S ′ = (−4,0)
120. Ans(C)
c = ae = 4
MATHEMATICS Page |7A. 14
CONIC SECTIONS
we have b2 = a2 (1 − e2 ) = 25 − 16 = 9 We have b2 = a2 e2 − a2
∴Equation of ellipse ⇒ 16 = 52 − a2
x2 y2 ⇒ a2 = 52 − 16
+ =1
a2 b 2 ⇒ a2 = 36
x2 y2
⇒ + =1 ⇒a=6
25 9
2 is an ellipse.
127. Ans(C)
18 − 4 14 7 7
e1 = √ =√ =√ =√ b2
18 18 9 3
tan45° =
a(a + ae)
9 + 4 √13 𝑤𝑒 ℎ𝑎𝑣𝑒 b2 = a2 (1 + e)
e12 = √ =
9 3
⇒ a2 (e2 − 1) = a2 (1 + e)
2e12 + e22 = 3 ⇒ e2 − 1 = 1 + e
128. Ans(C) ⇒ e2 = e + 2
x2 y2
For an ellipse + =1 ⇒ e2 − e − 2 = 0
16 b2
⇒ (e − 2)(e + 1) = 0
Foci S (±√16 − b 2 , 0)
⇒e=2
x2 y2
For a hyperbola 144 − 81 = 1, foci S(±3,0) 133. Ans(C)
25 25
Given 2ae = 18 ⇒ ae = 9
√16 − b 2 = 3 ⇒ 16 − b2 = 9
2a a
⇒ b2 = 16 − 9 and =8⇒ =4
e e
⇒ b2 = 7 a
thus, ae × = 36
129. Ans(D) e
⇒ a2 = 36
SE = (±√12, 0), SH = (±√a2 + 3, 0)
c 9 3
By given data 12 = a2 + 3 e= = ⇒e=
a 6 2
⇒ a2 = 9 ⇒ a = 3 b2 = a2 e2 − a2 = 81 − 36
130. Ans(D) ⇒ b2 = 45
x2 y2 x2 y2
Given hyperbola − =1 ∴ Equation of hyperbola is − =1
9 4 a2 b 2
x2 y2
9 + 4 √13 ⇒ − =1
e=√ = 36 45
9 3
134. Ans(C)
131. Ans(A)
2b = 8 ⇒ b = 4
2𝑐 = 2ae = 4√13 P = (3, −2)
⇒ ae = 2√13 x2 y2
Equation of Hyperbola − =1
a2 b2
⇒ 2b = 8 ⇒ b = 4
MATHEMATICS Page |7A. 15
CONIC SECTIONS
9 4 27
− =1 2 2 × 27
a2 16 = 2 = = 2 × 3√3
36 3√3 3√3
a2 = 2
5
∴ LLR = 6√3
5x 2 y 2
− =1 140. Ans(A)
36 16
135. Ans(B) Since e = √2 ⇒ a = b
2 2
x y ⇒ 2ae = 24
2
− 2=1
a b ⇒ ae = 12
Passes through (3,0)
⇒ a√2 = 6 × √2√2
x2 y2
⇒ − 2=1 ⇒ a = 6√2
9 b
It also passes through (3√2, 2) ⇒ b = 6√2
9×2 4 x 2 − y 2 = a2
⇒ − 2=1
9 b ⇒ x 2 − y 2 = 72
4
⇒ 1 = 2 ⇒ b2 = 4 141. Ans(C)
b
2b2
√a2 +b2 √13 L𝐿𝑅 = 8 =
We have e = = a
a 3
9 SS1 = 2ae
ae =
2
64 + 36
3√3 =2×8×√
a= 64
2
b2 = a2 (e2 − 1) = 2 × 10 = 20
27 27 144. Ans(B)
b2 = (2) =
4 2
25 − 16
2b2 e1 = √
LL1 = 25
a
MATHEMATICS Page |7A. 16
CONIC SECTIONS
b>a 2
⇒ 3e2 = 4 ⇒ e =
3 √3
e=
5 149. Ans(D)
e1 e2 = 1 x2 y2
+ =1
5 9 5
e2 ( Hyperbola ) =
3 1 2
e=√ =
Foci (0, ±3) 9−5 3
9
x2 y2
− =1 x2 y2
a2 b 2 − =1
9 45/4
⇒b=3
x2 y2 1 81 9 3
− =1 e′ = √ √9 = √ = =
a2 9 45 36 6 2
9+
16 4
a2 = b2 2 3
9 ee′ = ⋅ =1
3 2
⇒ a2 = 16
150. Ans(A)
x2 y2
− =1 4
16 9 ae = 2 ⇒ a =
3
145. Ans(A)
b2 = a2 (e2 − 1)
a = 3 b ⇒ a2 = 9b2
16 5 20
a2 = 9a2 (e2 − 1) = ( )=
9 4 9
√10 9x 2 9y 2
9e2 = 10 ⇒ e = Equation of Hyperbola − =1
3 16 20
146. Ans(C) x2 y2 4
⇒ − =
2a = 7, P = (5, −2) 4 5 9
x2 y2 151. Ans(C)
− =1
a2 b 2 Given that the center of the circle is (1,2)
2
4 2 y Radius of the circle
x − 2=1
49 b
= √(4 − 1)2 + (6 − 2)2 = √9 + 16 = 5
4 4 196
× 25 − 2 = 1 ⇒ b2 = So, the area of the circle = πr 2 = π × (5)2 = 25π
49 b 51
4x 2 51y 2 152. Ans(C)
Equation of Hyperbola − =1
49 196
147. Ans(C)
x2 y2
− =1
4−r 4+r
a2 = 4 − r > 0 ⇒ 4 > r ⇒ r < 4 → (1)
b2 = 4 + r > 0 ⇒ r > −4 → (2)
from (1) and (2) −4 < r < 4
148. Ans(C) Let the required circle touch the axes at (a, 0)
By the given data 2 b = ae and (0, a)
2
2b ∴ Centre is (a, a) and r = a
=8
a So, the equation of the circle is
⇒ b2 = 4a
(x − a)2 + (y − a)2 = a2
2 2 2
4b = a e
If it passes through a point P(3,6), then
⇒ 4a2 (e2 − 1) = a2 e2
MATHEMATICS Page |7A. 17
CONIC SECTIONS
⇒ √x 2 + y 2 + 9 + 6y = |y − 3|
Squaring both sides, we have x 2 + y 2 + 9 + 6y =
y 2 + 9 − 6y
⇒ x 2 = −12y
Let the equation of the circle be 156. Ans(B)
(x − h)2 + (y − k)2 = r 2 Given parabola is y 2 = 4ax
Let the center be (0, a) If the parabola is passing through (3,2)
1
∴ Radius r = a Then, (2)2 = 4a × 3 ⇒ 4 = 12a ⇒ a =
3
So, the equation of the circle is Now length of the latus rectum = 4a
2 2 2
(x − 0) + (y − a) = a 1 4
=4 × =
2 2 3 3
⇒ x + y − 2ay = 0
157. Ans(A)
Now CP = r
⇒ √13 + a2 − 6a = a
⇒ 13 + a2 − 6a = a2
13
a= , Putting the value of a in eq. (i) we get
6
13
x2 + y2 − 2 ( ) y = 0
6
⇒ 3x 2 + 3y 2 − 13y = 0
154. Ans(C)
∴Focus F = (−1,0)
MATHEMATICS Page |7A. 18
CONIC SECTIONS
(x + 1)2 + y 2 = (x + 5)2 ⇒ b2 = 4a
y 2 = 8(x + 3) 1 ae
∴ (2ae) = 2b ⇒ ae = 2b ⇒ b =
2 2
158. Ans(A)
a2 e2 a2 e2
⇒ b2 = ⇒ 4a = [from eq. (i)]
Given that focus of the ellipse is (1, −1) and the 4 4
9x 2 9y 2 x2 y2 1 Given y 2 = x
− =1⇒ − =
16 20 16 20 9 Differentiating both sides w.r.t. x
x2 y2 4 dy dy 1
⇒ − = 2y =1⇒ =
4 5 9 dx dx 2y
164. Ans(A) π
Also, tangent makes an angle of with x axis
4
c = (−a, −b), r = √a2 − b 2
dy π
The equation of the circle is ⇒ = tan = 1
dx 4
(x + a)2 + (y + b)2 = a2 − b2 1 1
∴ =1⇒y=
⇒ x 2 + y 2 + 2ax + 2by + a2 + b2 2y 2
170. Ans(A)
2ae = 16, ae = 8, e = √2
b2 = a2 (e2 − 1) = 32
x2 y2
Equation of Hyperbola − =1
32 32
⇒ x 2 − y 2 = 32 (or) ae = 8, e = √2
a = 4√2, (∵ e = √2 ⇒The hyperbola is
S = (0, a) = (0,3)
rectangular hyperbola)
OS = 3, PQ = L. L. R = 4a = 12
So, a = b, ⇒ b = 4√2
1 1
Area of △ OPQ = (OS)(PQ) = × a × 4a 171. Ans(C)
2 2
1 x2 y2
= × 3 × 12 = 18 + =1
2 25 9
166. Ans(B)
a2 − b 2
5 e=√
Given e =
4
a2
1 257 ⇒ 13 − 4k = 25 ⇒ k = −3
⇒ x 2 + y 2 − x + 2y − =0
2 6 9. Ans (A)
1 257
g=− , f = 1, C=− Clearly, C = (1, 1) and radius = 1 unit
4 16
The equation is x2 + y2 – 2x – 2y +1 = 0
1 257 274
radius = √ + 1 + =√ 10. Ans (B)
16 16 16
y2 = 16 x 4a = 16 a = 4
2. Ans (D)
focus = (a, 0) = (4, 0)
For the equation of a circle
11. Ans (D)
Coeff. x2 = co-eff. y2 p = 7
y2 + 8x = 0 y2 = – 8x 4a = 8 a = 2
3. Ans (B)
directrix is x = a i.e., x = 2
Clearly circle passes through (4, 0) and
12. Ans (A)
(0, 5). Also, it passes through (0, 0). Thus, the
y2 = 4ax passes through (2, –6)
equation of the circle is x2 + y2 – 4x – 5y = 0
9
4. Ans (B) ⇒ 36 = 8a ⇒ a =
2
Equation of the circle concentric with L.L.R = 4a = 18
x2 + y2 + 4x – 2y + 3 = 0 is 13. Ans (C)
x2 + y2 + 4x – 2y + c = 0 Clearly the axis of the parabola is x – axis.
since this circle passes through (–2, –3) Now the point of intersection of the axis and the
4 + 9 – 8 + 6 + c = 0 11 + c = 0 directrix is Z = (0, 0), V = (4, 0).
c = – 11 Let S = (a, 0). The vertex is the midpoint of ZS.
The circle is, x2 + y2 + 4x – 2y – 11 = 0 a
∴ (4,0) = ( , 0) ⇒ a = 8 ∴ S = (8,0)
2
5. Ans (D)
14. Ans (B)
Area = 4 r2 = 4 r = 2 The vertex V = (0, 0) directrix is x = –5.
centre (2, 2) and r = 2 a = distance of V from the directrix a = 5
Equation is, (x – 2)2 + (y – 2)2 = 22 Length of the latus rectum = 4a = 20
6. Ans (B) 15. Ans (D)
(3, 4) C: 9 + 25 + 12 − 32 − 29 < 0 y2 + 2y + x = 0 y2 + 2y + 1 = – x+ 1
point lies inside circle (y + 1)2 = – (x – 1)
7. Ans (D)
V = (1, –1) which lies in fourth quadrant
Equation of the circle is
16. Ans (B)
3 5 8
x2 + y2 − x + y − = 0 Focus = (– 4, 0) directrix : x = 4
7 7 7
3 5 Focus is to the left of directrix
g = − ,f =
14 14 Also vertex = (0, 0), a = 4
∴ C = ( ,− )
3 5
equation is y2 = – 16 x
14 14
3. Ans (D) The point (−3,1,2) lies in second octant and the
Since M is the foot of perpendicular from A on the point (−3,1, −2) lies in sixth octant.
YZ-plane and equation of the plane x = 1 Hence, coordinates of L are (3,4,0). Similarly, we
represents the plane parallel to YZ-plane at a can find the coordinates of M(0,4,5) and N(3,0,5)
7. Ans (A)
Since L is the foot of perpendicular from point
(3,5,6) on the x-axis so, its y and z-coordinates are
zero. Hence, the coordinates of L are (3,0,0).
(b) The equation of the plane y = 0 represents the 8. Ans (B)
XZ-plane and the equation of the plane y = 3 I. Through the point P in the space, we draw three
represents the plane parallel to X∠-plane at a planes parallel to the coordinate planes, meeting
distance 3 units above XZ-plane. the X-axis, Y-axis and Z-axis in the points A, B and
C, respectively. We observe that, OA = x, OB = y
and OC = z. Thus, if P(x, y, z) is any point in the
space, then x, y and z are perpendicular distances
from YZ, ZX and XY-planes, respectively. II. Given
x, y and z, we locate the three points A, B and C on
the three coordinate axes. Through the points A, B (a) Let A(0,7, −10), B(1,6, −6) and C(4,9, −6) are
and C, we draw planes parallel to the YZ-plane, ZX- the vertices of a triangle. Then, Side AB = Distance
plane and XY-plane, respectively. The point of between points A and B
intersection of these three planes, namely, ADPF, = √(0 − 1)2 + (7 − 6)2 + (−10 + 6)2
BDPE and CEPF is obviously the point P, = √1 + 1 + 16 = √18 = 3√2
corresponding to the ordered triplet (x, yz). and side BC = Distance between points B and C =
√(1 − 4)2 + (6 − 9)2 + (−6 + 6)2
= √9 + 9 + 0 = √18 = 3√2
Clearly, AB = BC. Hence, triangle is an isosceles
triangle.
(b) Let A(0,7,10), B(−1,6,6) and C(−4,9,6) are the
vertices of a triangle. Then,
Side AB = √(0 + 1)2 + (7 − 6)2 + (10 − 6)2
9. Ans (A)
= √1 + 1 + 16 = √18 = 3√2
On yz-plane, x = 0
Side BC = √(−1 + 4)2 + (6 − 9)2 + (6 − 6)2
10. Ans (A)
= √9 + 9 + 0 = √18 = 3√2
According to the question.
Distance of any point from XY-plane is and side CA = √(−4 − 0)2 + (9 − 7)2 + (6 − 10)2
The point on the X-axis is of form P(x, 0,0). Since, 21. Ans (B)
the points A and B are equidistant from P, PA2 = Three points are collinear if the sum of any two
i.e., (x − 3)2 + (0 − 2)2 + (0 − 2)2 PQ = √(−2 − 2)2 + (−2 − 4)2 + (−2 − 6)2
= (x − 5)2 + (0 − 5)2 + (0 − 4)2 = √16 + 36 + 64 = √116 = 2√29
49
⇒ 4x = 25 + 25 + 16 − 17 i.e., x =
4
QR = √(6 + 2)2 + (10 + 2)2 + (14 + 2)2
49
Thus, the point P on the X - axis is ( , 0,0) which is = √64 + 144 + 256 = √464 = 4√29
4
Here, the points are A(1,2,3), B(−1, −1, −1) and (4x + 25)2 = 25[(x + 4)2 + y 2 + z 2 ]
A = (2,9,12), B = (1,8,8), A = (2, −1,1), B = (1, −3, −5), C = (3, −4, −4)
C = (−2,11,8), D = (−1,12,12) a = BC = √4 + 1 + 1 = √6
AB = √1 + 1 + 16 = 3√2, b = CA = √1 + 9 + 25 = √35
BC = √9 + 9 + 0 = 3√2 c = AB = √1 + 4 + 36 = √41
On y-axis, x = 0 and z = 0 and given point P(3,4,5) We know that x-axis, y = 0 and z = 0.
⟹ x 2 + 1 − 2x + 4 + 9 = x 2 + 9 − 6x + 25 + 4 2m − 3 3
⟹ =0 ⟹m=
⟹ x = 6 ∴ Required point is (6, 0, 0) m+1 2
3 + 2 × 5 13
Now, x= =
3+2 5
3. Ans (C) 3 × (−2) + 2 × (−2)
and z = = −2
Let A(5, −4, 2), B(4, −3, 1), C(7, −6, 4) and 5
D(8, −7, 5) 13
∴ Required points is ( , 0, −2)
5
Then, AB = √(4 − 5)2 + (−3 + 4)2 + (1 − 2)2
6. Ans (C)
= √1 + 1 + 1 = √3
The distance between given points
BC = √(7 − 4)2 + (−6 + 3)2 + (4 − 1)2
= √(2 − 1)2 + (2 − 4)2 + (3 − 5)2
= √9 + 9 + 9 = 3√3
= √1 + 4 + 4 = 3
CD = √(8 − 7)2 + (−7 + 6)2 + (5 − 4)2 7. Ans (A)
= √1 + 1 + 1 = √3 Let the coordinate of a point Q on x-axis be (a, 0, 0)
AD = √(8 − 5)2 + (−7 + 4)2 + (5 − 2)2 ∴ Distance, PQ = √(a − a)2 + (b − 0)2 + (c − 0)2
= √9 + 9 + 9 = 3√3 = √b 2 + c 2
Position vector of 8. Ans (B)
⃗⃗⃗⃗⃗ = (4 − 5)î + (−3 + 4)ĵ + (1 − 2)k̂
AB Given that, A(5, −1, 1), B(7, −4, 7), C(1, −6, 10) and
= −î + ĵ − k̂ D(−1, −3, 4)
And position vector of Now, AB = √(7 − 5)2 + (−4 + 1)2 + (7 − 1)2
BC = (7 − 4)î + (−6 + 3)ĵ + (4 − 1)k̂
⃗⃗⃗⃗⃗ = √4 + 9 + 36 = 7
= 3î − 3ĵ + 3k̂ BC = √(1 − 7)2 + (−6 + 4)2 + (10 − 7)2
⃗⃗⃗⃗⃗ = (−î + ĵ − k̂). (3î − 3ĵ + 3k̂)
⃗⃗⃗⃗⃗ ∙ BC
Now, AB = √36 + 4 + 9 = 7
= −3 − 3 − 3 ≠ 0
CD = √(−1 − 1)2 + (−3 + 6)2 + (4 − 10)2
∴ ABCD is parallelogram. = √4 + 9 + 36 = 7
4. Ans (C)
DA = √(5 + 1)2 + (−1 + 3)2 + (1 − 4)2
Let point is (α, β, γ)
= √36 + 4 + 9 = 7
∴ (α − α)2 + β2 + γ2 = α2 + (β − b)2 + γ2
∴ AB = BC = CD = DA = 7,
= α2 + β2 + (γ − c)2
⃗⃗⃗⃗⃗ ∙ BC
Also, AB ⃗⃗⃗⃗⃗ ≠ 0 (These are not perpendicular)
= α2 + β2 + γ2
a b c
∴ ABCD is not square. It is rhombus
We get, α = , β = and γ =
2 2 2 9. Ans (D)
a b c
∴ Required point is ( , , ) Let the point in xy-plane be P(x1 , y1 , 0). Let the
2 2 2
Such events A, B and C are called exhaustive events. = P(A) + P(A‾)P(A‾)P(A) + P(A‾)P(A‾)P(A‾)P(A‾)P(A)
In general, if E1 , E2 , … , En are n events of a sample +⋯
space S and if 1 5 5 1 5 5 5 5 1
= + ( )( )( ) + ( )( )( )( )( )+ ⋯
E1 ∪ E2 ∪ E3 ∪ … ∪ En = ⋃ni=1 Ei =S 6 6 6 6 6 6 6 6 6
1 1 5 2 1 5 4
Then E1 , E2 , … , En are called exhaustive events. In = + ( ) + ( ) +⋯
6 6 6 6 6
other words, events E1 , E2 , … , En are said to be
1/6 6
exhaustive, if atleast one of them necessarily = 2 =
5 11
1−( )
occurs whenever the experiment is performed. 6
Further, if Ei ∩ Ej = ϕ for i ≠ j, i.e., events, Ei and Ej 22. Ans (C)
Total number of outcomes = 8
are pairwise disjoint and ⋃ni=1 Ei = S, then
Favourable cases are HTT, THT, TTH.
E1 , E2 , … … , En are called mutually exclusive and
∴ Number of favourable outcomes = 3
exhaustive events.
3
18. Ans (D) ∴ Required probability =
8
1 5 mutually exclusive.
∴ P(A‾) = 1 − P(A) = 1 − =
6 6 Using axiom (iii) of probability, we get
Since, first time 1 occurs at the odd throw.
P(A ∪ B) = P(A) + P(B − A)
So, the required probability
and P(B) = P(A ∩ B) + P(B − A).
Solve Eq. (iii) from Eq. (ii), (i) Since, P(D) = −0.20, this is not possible as 0 ≤
If A and B are disjoint sets, i.e., they are mutually P(A) ≤ 1 for any event A.
exclusive events, then A ∩ B = ϕ (ii) P(S) = P(A ∪ B ∪ C ∪ D)
9 45 27 46 127
Therefore, P(A ∩ B) = P(ϕ) = 0 = + + + = ≠ 1.
120 120 120 120 120
Thus, for mutually exclusive events A and B, we
This violates the condition that P(S) = 1
have
29. Ans (A)
n(S) = 9 C3
n(E) = 3 C1 × 4 C1 × 2 C1
3×4×2 24×3!
Probability = 9C = × 6!
3 9!
24×6 2
= = .
9×8×7 7
Also, we know that A′ and A are mutually exclusive By addition theorem of probability,
and exhaustive events i.e., P(A ∪ B) = P(A) + P(B) − P(A ∩ B)
A ∩ A′ = ϕ and A ∪ A′ = S ⇒ P(A ∪ B) = P(A) + P(B)
′)
or P(A ∪ A = P(S) ⇒ P(A ∪ B) = 0.25 + 0.4
Now, P(A) + P(A′ ) = 1, by using axioms (ii) and [∵ P(A) = 0.25, P(B) = 0.4( given )]
(iii) P(A ∪ B) = 0.65..(i)
′)
Or P(A = P(not A) = 1 − P(A) Now, we have P(Ac ∩ B c ) = P(A ∪ B)c
33. Ans (D) ⇒ P(Ac ∩ B c ) = 1 − P(A ∪ B)
Let E and F denote the events that Anil and Ashima = 1 − 0.65 = 0.35
will qualify the examination, respectively. Given 35. Ans (A)
that, Given, P(A ∩ B c ) = 0.25 and
P(E) = 0.05, P(F) = 0.10 and P(Ac ∩ B) = 0.5
P(E ∩ F) = 0.02. Then, ∴ P{(A ∪ B)c } = 1 − P(A ∪ B)
I. The event 'both Anil and Ashima will not qualify = 1 − {P(A ∩ B c ) + P(Ac ∩ B) + P(A ∩ B)}
the examination' may be expressed as E ′ ∩ F ′ . = 1 − (0.25 + 0.5 + 0)
′
Since, E is 'not E ', i.e., Anil will not qualify the [∵ A and B are mutually exclusive events
examination and F ' is 'not F ', P(A ∩ B) = 0] = 1 − 0.75 = 0.25
i.e., Ashima will not qualify the examination 36. Ans (C)
′ ′ ′ 1
Also E ∩ F = (E ∪ F) We have P(A ∪ B) =
2
Now P(E ∪ F) = P(E) + P(F) − P(E ∩ F) 1
⇒ P(A ∪ (B − A)) =
or P(E ∪ F) = 0.05 + 0.10 − 0.02 = 0.13 2
Therefore, P(E ′ ∩ F ′ ) = P(E ∪ F)′ 1
⇒ P(A) + P(B − A) =
2
= 1 − P(E ∪ F)
(Since A and B − A are mutually exclusive)
= 1 − 0.13 = 0.87
1
II. P (atleast one of them won’t qualify) ⇒ 1 − P(A‾) + P(B − A) =
2
= 1 − P (both of them will qualify) 2 1
⇒ 1− + P(B − A) =
= 1 − 0.02 = 0.98 3 2
1
III. The event only one of them will qualify the ⇒ P(B − A) =
6
examination is same as the event either (Anil will 1
⇒ P(A‾ ∩ B) = ( Since A‾ ∩ B = B − A)
6
qualify and Ashima will not qualify) or (Anil will
37. Ans (B)
not qualify and Ashima will qualify)
1
i.e., E ∩ F ′ or E ′ ∩ F, where E ∩ F ′ and E ′ ∩ F are Given, P(A) = P(B)
2
No. of days in a leap =366 And for each of the above, the third dice can have
So, there will be 52 weeks and 2 days any of the 5 remaining faces
So, every leap year has 52 Tuesday The possible outcomes are P(A)=1/6, P(B)=1/6,
Now, the probability depends on P(C)=5/6, P(A)=1/6, P(B)=5/6, P(C)=1/6
remaining 2 days. The possible pairing of days and P(A)=5/6, P(B)=1/6, P(C)=1/6. Hence the
48. Ans(C) 10
X ∈N, Total ways = 𝐶1 =100
"Since " A" and " B" are mutually exclusive"
⇒n(S)=100
∴ P(A ∩ B) = 0
(x−30)(x−20)
<0
∴ P(A ∪ B) = P(A) + P(B) (x−40)
5 55. Ans(A)
9x = 5 ⇒ x =
9 The sample space for given condition is
1 5
∴ x ∈ [− , ] (1,8),(2,8),(3,8),(4,8),(5,8),(6,8),(7,8)
3 9
(1,7),(2,7),(3,7),(4,7),(5,7),(6,7)
49. Ans(A)
40 (1,6),(2,6),(3,6),(4,6),(5,6)
C2 + 60 C2 17
Prob = 100 C
= (1,5),(2,5),(3,5),(4,5)
2 33
50. Ans(C) (1,4),(2,4),(3,4)
n(S) = 100
C1 = 100 (1,3),(2,3)
12
1
(excluding his own car) have left. n(S) = C4 , n(E) = 6 C2 , P(E) =
33
22
𝐶8
Required probability = 24 57. Ans(B)
𝐶10
Since, if is an eight-storey building. So, there are 7
22! 14! ×10! 15
= × = possible options for them in 7 floors in total if
14! × 8! 24! 92
=2×
5×4×3×2×1
=
1 58. Ans(C)
6×5×4×3×2×1 3
8
53. Ans(D) C2= all possible outcomes smallest of the 2 has
20
C2 19 is < 4.
P(E) = 25 C
=
2 30 If the small no is 1,then {2,3….,...8} can be the
54. Ans(C) others no. So, for 1,there will be 7 outcomes
similarly, for 2, there will be 6 outcomes
MATHEMATICS Page | 9A. 7
PROBABILITY
for 3, there will be 5 outcomes February has 29 days in a leap year out of
𝟕+𝟔+𝟓 9 which 28 days comprise 4 complete weeks,
P(A)= 8 =
𝐂2 14
thereby 4 Saturdays.
59. Ans(A)
Remaining one day can be any of the 7 days.
15 c × 5 c
2 1
Required Probability = 20 c ∴n(S)=7
3
1 1 1 1 1 1 1
Probability when all are tails = × × = Req. prob = × =
2 2 2 8 12 7 84
Case 2: OEE= When one is tails and two are 65. Ans(C)
head =THH=3 possibility. Non-leap year has 365 days
Probability when one is tails and two are 1 year =52 weeks=52×7days
1 1 1 3
head=3× × × = =364 days [It contains 52 Sundays, Mondays and so
2 2 2 8
The following are possibilities for the sum to be 6 P(53 Sundays or 53 Tuesdays or 53 Thursdays)
1+2+3→ can be interchanged in 3!=6 ways =P(53 Sun)+P(53 Tue)+P(53 Thu)
0+3+3→ can be interchanged in 3!/2!=3 ways 1 1 1 3
= + + =
7 7 7 7
2+2+2→ = 1 way
66. Ans(B)
So, number of ways = 10 ways 13 c × 13 c × 13 c × 13 c
1 1 1 1
Required probability = 52 c
Total number of possibilities = 6*6*6 4
75. Ans(C)
In 'ASSISTANT' and 'STATISTICS', the same letters
are A,I,S,T
n(E) 145 19
P(E) = = = 2c 1c 1
n(S) 220 44 Prob. of choosing A= 9 1 × 10 1 =
c1 c1 45
70. Ans(C) 1c 2c 1
Prob. of choosing I= 9 1 × 1
10 c =
c1 1 45
Selection of 6 months from the 12 months and
3c 3c 1
then deciding the order of the birthdays of the six Prob. of choosing S= 9 1 × 10 1 =
c1 c1 10
12 2c 3c 1
girls can be done in 𝐶6∗6! ways. Prob. of choosing T= 9 1 × 10 1 =
c1 c1 15
12 12
𝐶6 ×6! 𝑝6 So, total probability
The required probability = =
126 126 1 1 1 1 19
= + + + =
71. Ans(C) 45 45 10 15 90
Let A = the event of choosing odd numbers Only 4 Favourable cases for G.P are
B = The event of getting the sum an even number. (1,2,4), (2,4,8), (1,3,9), (4,6,9)
4
Then A∩B = The event of choosing odd numbers Required probability =11
C3
whose sum is even
77. Ans(B)
∴n(B)= 4 c2 + 5 c2 =16 20
n(S) = C2 = 190
5
And n(A∩B)= c2 =10
Sum odd = one even and one odd
n(A∩B) 10 5
∴ Required probability = = = n(A) = 10
C1 × 10
C1 = 100
n(B) 16 8
3 1 4 2 13 × 13 13
P(A) = = , P(B) = = , P(E) = ×2×1 =
6 2 6 3 52 × 51 102
1 87. Ans(C)
P(A ∩ B) =
6 Total 12 persons it can be arranged in 12 !
P(A ∪ B) = P(A) + P(B) − P(A ∩ B) Ways i.e., n(s) = 12 !
1 2 1 We have 2 cases
= + −
2 3 6
(i)TSTSTSTSTSTS (i.e., 6!×6! = (6!)2 Ways)
3+4−1 6
= = =1 (ii)STSTSTSTSTST(i.e., 6!×6! = (6!)2 Ways)
6 6
82. Ans(D) Teacher sit 1st or Student sit 1st
P(E) =
total possible arrangements
P(A‾) + P(B‾)
(6!)2 + (6!)2 2 × (6!)2
= 1 − P(A) + 1 − P(B) = =
(12)! (12)!
= 2 − {(P(A ∪ B) + P(A ∩ B))} 2×6×5×4×3×2×1 1
= =
= 2 − {1.1} = 0.9 12×11×10×9×8×7 462
n(E) = 1 P(A)+P(B)+P(C)=1………………….(1)
2 1
1 P(B) + P(B) + P(B) = 1
3 2
P(E) =
4 4P(B) + 6P(B) + 3P(B)
84. Ans(A) =1
6
Given, n(S) = 200 6
P(B) =
E1 = Divisible by 6 13
3
= {6,12,18,24,30,36,42,48,54, … … } Given P(B) = P(A)
2
n(E1 ) = 33 4
So, P(A) =
13
MATHEMATICS Page | 9A. 10
PROBABILITY
1 6 1 2 1
Also P(C) = × Choosing a white ball from bag ' X ′ = ⋅ =
2 13 2 5 5
4 3 7 95. Ans(C)
= + −0=
13 13 13 1 1
P(H) = , P(W) = ; Since events are independent
7 5
89. Ans(B)
1 1 1 here
P(A) = , P(B) = , P(C) = 1
3 4 5 P(H ∩ W) = P(H) × P(W) =
35
2 3 4
P(A‾) = P(B‾) = , P(C‾) = 96. Ans(D)
3 4 5
100
n(S) = c3
P(A ∪ B ∪ C) = 1 − P(A‾)P(B‾)P(C‾)
2 3 4 L.C.M of 2 & 3 gives the numbers which are
P(A ∪ B ∪ C) = 1 − ⋅ ⋅
3 4 5 divisible by both 2 & 3
60 − 24 36 E = {6,12,18,24,30,36,42,48,54,
= =
60 60
60,66,72,78,84,90,96}
90. Ans(C) 16
n(E) = c3
n(S) = 90 16
c3 16 × 15 × 14
L.C.M of 6 & 8 gives the numbers which are P(E) = 100 c
=
3 100 × 99 × 98
divisible by both 6 & 8 4 4
= =
i.e., E = {24,48,72} 35 × 33 115
n(E) = 3 97. Ans(B)
3 1 n(S) = 4 × 4 × 3(ignore 0 in Hundreds place)
P(E) = =
90 30 if A be the event of getting 3-digit even numbers
91. Ans(D) =(unit place ends with 0 )+( unit place ends with
P(A ∩ B) = P(A) + P(B) − P(A ∪ B) 2 or 4)
= 0.4 + 0.3 − 0.5 = 0.2 =(4 × 3 × 1)+(3× 3 × 2)
P(B ∣ ∩ A) = P(A) − P(A ∩ B) =12+18
1 n(A) 30 5
= 0.4 − 0.2 = 0.2 = P(A) = = =
5 n(S) 4 × 4 × 3 8
92. Ans(A)
98. Ans(B)
n(S) = 8 C2
n(S) = 9 C3
3 2
n(E) = C2 × C1
n(E) = 3 C1 × 4 C1 × 2 C1
3 2
n(E) C2 × C1 3
P(E) = = 8C
n(E) C1 × 4 C1 × 2 C1 2
n(S) 2 P(E) = = 9C
=
n(S) 3 7
3×2 6 3
= = = 99. Ans(B)
8 × 7 56 28
93. Ans(A) In a non-leap year' there are 365 days which have
Total 3 persons can be arranged in 33 52 weeks and 1 day. If this day is a Tuesday or
n(E) = 3 C1 = 3 53 Wednesday.
3 1 1 1 2
P(E) = = ∴ Required probability = + =
27 9 7 7 7
94. Ans(C) 100.Ans(B)
Since, the set of three consecutive numbers from 1 P(A ∪ B) = P(A) + P(B) − P(A ∩ B)
to 20 are (1,2,3), P(A ∪ B) = P(A) + P(B)
(2,3,4), (3,4,5), … , (18,19,20), i.e., 18 . P(A) + P(B) ≤ 1
18
P( numbers are consecutive) = P(A) ≤ 1 − P(B)
20 C
3
18 3 ∴ P(A) ≤ P(B‾)
= =
20 × 19 × 18 190 105.Ans(A)
3!
We have, P(A ∪ B) = P(A ∩ B)
P( three number are not consecutive )
P(A) + P(B) − P(A ∩ B) = P(A ∩ B)
3 187
=1− = [P(A) − P(A ∩ B)] + [P(B) − P(A ∩ B)] = 0
190 190
101.Ans(C) But P(A) − P(A ∩ B) ≥ 0 and
∴ Required probability =
2!6!
=
2 107.Ans(B)
7! 7
Total number of alphabets in the word probability
103.Ans (D)
= 11
We have digits 0,2,3,5.
Number of vowels = 4(O, A, I, I)
Number of divisible by 5 if unit place digit is ' 0 '
4
or ' 5 ' P( letter is vowel ) =
11
If unit place is ' 0 ' then first three places can be 108.Ans(C)
filled in 3 ! Ways Given, P(A fail ) = 0.2 and P(B fail ) = 0.3
If unit place is ' 5 ' then first place can be filled in ∴ P( either A or B fail ) ≤ P(A fail ) + P(B fail )
two ways and second and third place can be filled ≤ 0.2 + 0.3 ≤ 0.5
in 2 ! ways. 109.Ans(C)
So, number of numbers ending with digit ' 5 ' is We have, P(A ∪ B) = 0.6 and P(A ∩ B) = 0.2
2×2!= 4 P(A ∪ B) = P(A) + P(B) − P(A ∩ B)
∴ Total number of numbers divisible by 5 = 3! + 0.6 = P(A) + P(B) − 0.2
4 = 10 = n(E) P(A) + P(B) = 0.8
Also, total number of numbers = 3 × 3! = 18 P(A‾) + P(B‾) = 1 − P(A) + 1 − P(B)
∴ Required probability =
10
=
5 = 2 − [P(A) + P(B)] = 2 − 0.8 = 1.2
18 9
110.Ans (B)
104.Ans(A)
If M and N are any two events.
For mutually exclusive events, P(A ∩ B) = 0
MATHEMATICS Page | 9A. 12
PROBABILITY
121.Ans (A)
So P(E′)= n(E′)/ n(S)=10/36=5/18
1 1 1
Hence P(E)=1−P(E′)=1−5/18=13/18 We have, 𝑃(𝐴) = , 𝑃(𝐵) = and 𝑃(𝐶) =
2 4 3
116.Ans(C)
MATHEMATICS Page | 9A. 13
PROBABILITY
2 4. Ans (B)
⇒ 𝑃(𝐴) = 2𝑃(𝐵) and 𝑃(𝐶) = 𝑃(𝐵)
3
We have been given that, a committee has to be
∴ 𝑃(𝐴 ∪ 𝐵 ∪ 𝐶) = 𝑃(𝐴) + 𝑃(𝐵) + 𝑃(𝐶) made of 5 members from 6 men and 4 women.
We need to find probability that at least one
2
⇒ 1 = 2𝑃(𝐵) + 𝑃(𝐵) + 𝑃(𝐵) woman is present in the committee.
3
Let S be the total outcomes and A be the possible
11 3
⇒1= 𝑃(𝐵) ⇒ 𝑃(𝐵) = outcomes.
3 11
As a committee has to be made of 5 members
123.Ans (C)
from 6 men and 4 women, total cases would be,
P(A1 ∩ B1 ) = 1 − P(A ∪ B)
= 4C1 × 6C4 + 4C2 × 6C3 + 4C3 × 6C2 + 4C4 × 6C1
= 1 − [0.59 + 0.3 − 0.21]
+ 6 C5
= 0.32
= 60 + 120 + 60 + 6 + 6 = 252
124.Ans (B)
No. of ways in which at least one woman exist are
1 3 13
P(A) = , P(B) = , P(A ∪ B) = = 4C1 × 6C4 + 4C2 × 6C3 + 4C3 × 6C2 + 4C4 × 6C1 =
4 4 20
1 1 13 246
+𝑥− ⋅𝑥 =
4 4 20 246 41
Hence required probability = = .
252 42
3 13 5 8
𝑥− − = 5. Ans (A)
4 20 20 20
8 4 8 Mohan can gets one prize, 2 prizes or 3 prizes
𝑥= × =
20 3 15 and his chance of failure means he get no
125.Ans (B)
prize.
Use ∑ 𝑃(𝑤𝑖 ) = 1 12
Number of total ways = C3 = 220
Favourable number of ways to be failure
KCET MOCKTEST SOLUTIONS: = 9C3 = 84
1. Ans (B) 84 34
Hence required probability = 1 − = .
220 55
Total number of outcomes = 36
6. Ans (A)
Favourable number of outcomes = 6
(i) This question can also be solved by one student
i.e., (1, 1), (2, 2), (3, 3), (4, 4), (5, 5), (6, 6),
6 1
(ii) This question can be solved by two students
Required probability = = .
36 6 simultaneously
2. Ans (B) (ii) This question can be solved by three students
The second ball can be red in two different all together.
ways 1 1 1
P(A) = , P(B) = , P(C) =
(i)First is white and second red 2 4 6
3 2 6 P(A ∪ B ∪ C) = P(A) + P(B) + P(C)
P(A) = × =
5 4 20 −[P(A). P(B) + P(B). P(C) + P(C). P(A)]
(ii) First is red and second is also red + [P(A). P(B). P(C
2 1 2 1 1 1 1 1 1 1 1 1
P(B) = × = = + + −[ × + × + × ]
5 4 20 2 4 6 2 4 4 6 6 2
6 2 2
Req. Prob= + = 1 1 1 33
20 20 5
+[ × × ]=
2 4 6 48
MATHEMATICS Page | 9A. 14
PROBABILITY
23. Ans(C)
4 13 39
c1 × c9 × c4
P(E) = 52 c
13
24. Ans(A)
10
n(S) = c2 = 45
A={(5,6)(5,7)(5,8)(5,9)(5,10)}
n(A) = 5
5 1
so, P(A)= =
45 9
26. Ans(C)
2c 1 5
2
Required probability 1 − 4c =1− =
2 6 6
27. Ans(B)
20 10
n(S) = c3 , n(E) = c3
28. Ans(B)
If two persons sit together consider them as single
object,
thus n(E) = no. of objects = m − 1
m
n(S) = c2
𝑚−1
hence P(E)= m
c2
29. Ans(A)
n(S) = 100, n(E) = 74
30. Ans(C)
n(S) = 900
E={102,119……986}
1. Ans (B) 1 1
a5 = , a9 =
S2n = 3Sn 9 5
1 1
2n 3n a + 4d = → (1), a + 8d = → (2)
[2a + (2n − 1)d] = [2a + (n − 1)d] 9 5
2 2
4
4a + 4nd − 2 d = 6a + 3nd − 3 d (2) − (1) ⇒ d =
45
nd + d = 2a 4
from (1), a = ⇒a=d
3n 45
S3n [2a + (3n − 1)d]
= 2n 7. Ans (D)
Sn [2a + (n − 1)d]
2 t10 from beginning = 7 + 9 d
nd + A + 3nd − A
= 3[ ] t10 from end = t n−9 from beginning
nd + A + nd − d̸
4nd = 7 + (n − 10)d
= 3. = 3. (2) = 6
2nd But 55 = 7 + (n − 1)d
2. Ans (D) (n-1) d = 48
n = 1 ⇒ S1 = 5 therefore t10 + t n−9
n = 2 ⇒ S2 = 3 ⋅ 2 + 2 ⋅ 22 = 14 = 7 + 9d + 7 + (n − 10)d
T2 = S2 − S1 = 14 − 5 = 9 = 14 + (n − 1)d = 14 + 48
T1 = 5 8. Ans (A)
d = T2 − T1 = 4 5(a + 4d) = a(a + 8d)
3. Ans (D) a + 13d = 0
Given, T14 = a + 13d = 0
a1 + a1 + 4d + a1 + 9d + a1 + 14d + a1 + 19d 9. Ans (C)
+ a1 + 23d = 225 T4 + T8 = 24, T6 + T10 = 34
6a1 + 69d = 225 a + 3d + a + 7d = 24, a + 5d + a + 9d = 34
a1 + a2 + a 3 + ⋯ … + a 24 = 24a1 + 276d 2a + 10d = 24,2a + 14d = 34
= 4(6a1 + 69d) = 4 × 225 = 900 a + 5d = 12 → (1), a + 7d = 17 → (2)
4. Ans (A) 5
(2) − (1) ⇒ 2d = 5 ⇒ d =
T7 = 40 2
10. Ans (C)
a + 6d = 40
13 d = −5, a = 253
s13 = [2a + 12d] = 13[a + 6d]
2 T12 = a + 11d
= 13 × 40 = 520 = 253 − 55 = 198
5. Ans (A) 11. Ans (B)
1 1 a + 7d = 22
T9 = ,T =
7 7 9
a + 19d = 46
1
a=d= ⇒a = 8d= 2
63
63 2 62 64 t18 = 8 + 17(2) = 42
S63 = [ + ]= = 32
2 63 63 2 12. Ans (C)
6. Ans (B)
MATHEMATICS Page |10A. 1
SEQUENCES AND SERIES
n n−1
[2a + (n − 1)d] 5n + 4 a+( )d 3n + 8
2 = 2
n =
[2a′ + (n − 1)d′ ] 9n + 16 a1 + (
n − 1
) d1 7n + 15
2 2
(n − 1) n−1
a+ d 5n + 4
2 = = 11
(n − 1) ′ 9n + 16 2
a′ + d
2 n = 23
n−1
18th element, = 17 ⇒ n = 35 T12 77 7
2
1 = =
T12 176 16
√2 (√2)2 (√2)3 ar 2 (1 + r) 60
or 0, , , …. ⇒ =
3 3 32 ar 10
√2
which is an infinite GP whose, first term a = = ⇒ r(r + 1) = 6
30
⇒r=2⇒a=5
√2 and common ratio
⇒ T7 = ar 6 = (5)(2)6 = 320
(√2)2 1 √2
r= × = 28. Ans (B)
3 √2 3
16 Let first term = a > 0
Let nth term of GP=
2187
Common ratio = r > 0
n−1
16
∴ ar = ar + ar 2 + ar 3 = 3
2187
n−1 ar 5 + ar 6 + ar 7 = 243
√2 16
⇒ √2 ( ) = r 4 (ar + ar 2 + ar 3 ) = 243
3 2187
n−1 7 r 4 (3) = 243
√2 8√2 √2
⇒( ) = =( ) ⇒ r = 3 as r > 0
3 2187 3
On comparing the powers, we get from (1)
1
n−1=7⇒n=8 3a + 9a + 27a = 3 ⇒ a =
13
25. Ans (D)
a(r 50 − 1) 1 50
x, 2x + 2,3x + 3 are in G.P. S50 = = (3 − 1)
(r − 1) 26
⇒ (2x + 2)2 = x(3x + 3) 29. Ans (D)
⇒ 4x 2 + 8x + 4 = 3x 2 + 3x
MATHEMATICS Page |10A. 3
SEQUENCES AND SERIES
a
Let three terms of G.P. are , a, ar we get a = 6
r
are +ve. a + ar + ar 2
⇒
For r =
1 a + ar + ar 2 + ar 3 + ar 4 + ar 5
3
125
a 4 4⋅3 =
∴ S∞ = = = =6 152
1−r 1−1 2
3 1 + r + r2 125
⇒ =
Similarly, we can find S∞ , 1 + r + r 2 + r 3 + r 4 + r 5 152
1 + r + r2 125
when r = 2√2/3 ⇒ 2 3
=
(1 + r + r )(1 + r ) 152
32. Ans (C)
1 125 152
Here, a3 = ar 2 = 24 …. (i) ⇒ = ⇒ 1 + r3 =
1 + r 3 152 125
and a 6 = ar 5 = 192. …. (ii) 152 27 3 3
⇒ r3 = −1= =( )
Dividing Eq. (ii) by Eq. (i), we get 125 125 5
192 3
r3 = ⇒ r3 = 8 ⇒ r = 2 ⇒r=
24 5
Substituting r = 2 in Eq. (i), 36. Ans (B)
we get: a =
10
=
5 7 1 1 20
8 4 = [20 − {1 − ( ) }]
9 9 10
Let there be n terms in the given G.P. then, a n = 20
7 179 1 1
2650 = [ + ( ) ]
9 9 9 10
n−1 7
⇒ ar = 2560
= [179 + (10)−20 ]
10 n−1 81
⇒ (2 ) = 2560 ⇒ 2n−4 = 256
8 41. Ans (D)
⇒ 2n−4 = 28 ⇒ n − 4 = 8 a n = a n−1 + a n−2
⇒ n = 12 a n+1 = a n + a n−1 , n > 1
37. Ans (D) a n+1 a n−1
=1+ ,n > 1
an an
Given geometric series is
a n+1 a2 a3 a4 a5 a6
2 + 6 + 18 + 54 + ⋯ , n = 1,2,3,4,5 = { , , , , }
an a1 a 2 a 3 a 4 a 5
6
Here, a = 2, r = = 3 3 5 8
2
= {1,2, , , }
Sum of first 8 terms 2 3 5
42. Ans (B)
a(r 8 − 1) 2(38 − 1)
= = 1 + 3 + ⋯ … + 2n − 1 = n2
r−1 3−1
= 38 − 1 = 6561 − 1 = 6560 (1 + 3 + 5 + 7 + ⋯ … + 29) + (30 + 31 + 32
38. Ans (A) + ⋯ … . +60)
Let 131072 be the nth term of the given G. P. 31
= (15)2 + [60 + 30]
2
Here a = 2 and r = 4.
= 225 + 31 × 45
∴ 131072 = a n = 2(4)n−1
= 225 + 1395 = 1620
⇒ 65536 = 4n−1
43. Ans (B)
⇒ 48 = 4n−1 ⇒ n − 1 = 8 ⇒ n = 9.
100T100 = 50T50
Hence, 131072 is the 9th term of the G.P.
2[a + 99d] = a + 49d
39. Ans (B)
a+149d=0
We have: 61/2 ⋅ 61/4 ⋅ 61/8 …
T150 =0
= 6[1/2+1/4+1/8+⋯ up to ∞]
44. Ans (A)
= 6[(1/2)/(1−1/2)] = 61 = 6
A1 , A2 , … … … An are n AM's inserted between a, b
40. Ans (C)
⇒ a, A1 , A2 , … … . An , b are in A.P
Let S = 0.7 + 0.77 + 0.777 + ⋯
⇒ A1 + A2 + ⋯ … … + An = n(A. M of a, b)
7 77 777
= + 2 + 3 + ⋯ 20 terms a+b
10 10 10
= n( )
1 11 111 2
=7 [ + + + ⋯ 20 terms ]
10 102 103
Explanation
7 9 99 999
= [ + + + ⋯ 20 terms ] a, A1 , A2 , … An , b are in A.P
9 10 100 1000
7 1 1 1
= [(1 − ) + (1 − ) + (1 − ) A1 = a + d, b = a + (n + 1)d
9 10 102 103
MATHEMATICS Page |10A. 5
SEQUENCES AND SERIES
A2 = a + 2d n[−8 + n − 1] = 52
An = a + nd n(n − 9) = 13 × 4
A1 + A2 + ⋯ + An = na + (1 + 2 + 3 + ⋯ + n)d n = 13
n(n + 1) 50. Ans (A)
= na + d
2 (a − d) + (a) + (a + d) = 27
b−a
= n [a + ] a=9
2
a+b (a − d)(a + d) = 77
= n[ ]
2 a2 − d2 = 77
45. Ans (B) 81 − 77 = d2
Sn = 2n2 + n d=2
Tn = Sn − Sn−1 51. Ans (C)
2 2
Tn = 2n + n − (2(n − 1) + (n − 1)) T1 − T2 + T3 − T4 + ⋯ … − T20 + T21
= 2n2 − 2(n2 − 2n + 1) + 1 = (T1 + T21 ) − (T2 + T20 ) + (T3 + T19 ) …
= 4n − 1 − (T10 + T12 ) + T11
T8 = 32 − 1 = 31 = T11 = a + 10d(∵ a1 + a n = a 2 + a n + ⋯ . )
46. Ans (C) 52. Ans (C)
a1 , a 2 , a 3 , … … a n a = 56
are in A.P d=7
⇒ a1 + a n = a 2 + a n−1 = a 3 + a n−3 = ⋯.. I = 497
a1 + a5 + a10 + a15 + a 20 + a 24 = 450 a + (n − 1)7 = 497
3(a1 + a 24 ) = 450 n − 1 = 71 − 8
a1 + a24 = 150 n = 64
a1 + a8 + a17 + a 24 = 2(a1 + a 24 ) n
S = [2a + (n − 1)d] = 32[56 + 497] = 17,696
2
= 2(150)
53. Ans (A)
= 300
a = 1000 , d = 40
47. Ans (B)
a+b
Sum of interior angles of a polygon = (n − 2)π
Sum of n AM's = n ( ) n
2
[2a + (n − 1)d] = (n − 2)π
2 + 38 2
160 = n ( )
2 n[100 + (n − 1)2] = (n − 2)1800
160 = n(20) n(n + 490 ) = (n − 2)900
n=8 n=5
48. Ans (C) 54. Ans (B)
′ n
α sum of mAM s of a, b
= S1 = [2a + (n − 1)d]
β sum of nAM ′ s of a, b 2
a+b n n(n + 1)
m( ) m = [2 + n − 1] =
= 2 = 2 2
a+b n n n
n( ) S2 = [4 + (n − 1)3] = [3n + 1]
2
2 2
49. Ans (B) n
Sp = [2p + (n − 1)(2p − 1)]
a = −8, d = 2 2
Sn = 52 S1 + S2 + ⋯ . +Sp
n
[2a + (n − 1)d] = 52
2
MATHEMATICS Page |10A. 6
SEQUENCES AND SERIES
n a=q+p−1
= [2(1 + 2 + ⋯ + p) + (n − 1)(1 + 3 + ⋯ .2p
2
∴ Tq = a + (q − 1)d
− 1)]
n =q+p−1−q+1
= [p(p + 1) + (n − 1)p2 ]
2 =p
np 59. Ans (C)
= [np + 1]
2
n=m=q
"Alternative Solution "
Sq = q3
n=1
60. Ans (A)
S1 + S2 + S3 + ⋯ … … . +Sp = 1 + 2 + 3 + ⋯ … + p
p (x12 − x22 ) + (x32 − x42 ) + ⋯
= (p + 1)
2 = −d[x1 + x2 + x3 + ⋯ + x2n ]
np
= (np + 1) = d[n(x1 + x2n )]
2
−(x2n − x1 )(x2n + x1 )(n)
55. Ans (B) =
2n − 1
a=2 x2n − x1 n(x12 − x2n
2 )
where d = =
T1 + T2 + T3 + T4 + T5 2n − 1 2n − 1
1 61. Ans (B)
= [T6 + T7 + T8 + T9 + T10 ]
4 Let first term is a and common difference is d.
5(S5 ) = S10
Then
2
5 × [4 + 4d] = [4 + 9d] T9 = 0
−
20 + 20d = 8 + 18d ⇒ a + 8d = 0
d = −6 ⇒ a + 8d = 0
Tp = q, Tp+q = 0 ∵ T2 − T1 = T3 − T2
So, it is an AP
a + (p − 1)d = q
Let nth of this AP is purely imaginary.
a + (p + q − 1)d = 0
Then, Tn = a + (n − 1)d
⇒ q + qd = 0
= (−8 + 18i) + (n − 1)(2 − 3i)
d = −1
= −8 + 18i + n(2 − 3i) − 2 + 3i
a−p+1=q
MATHEMATICS Page |10A. 7
SEQUENCES AND SERIES
20 ⇒ q = 41 4k = 20 ⇒ k = 5
T2 = a + d a=3
T6 = a + 5d a = 3,
ar 4 = 2 ∴ r = 2 + √3
= 29 = 512 ⇒ 1 = r + r2
G1 , G2 , … G8 a=2
= (√2 × 3)8 = 64 a 39
+ a + ar =
r 10
98. Ans (B) a
⋅ a ⋅ ar = 1
1 + 3 + 9 + ⋯ … terms = 364 r
3n − 1 a=1
= 364 ⇒ 3n − 1 = 728
3−1 1 39
n 6 ∴ +1+r=
3 = 729 = 3 ⇒ n = 6 r 10
99. Ans (B) 1 29 5 2 5
⇒r+ = = + ⇒r=
r 10 2 5 2
Given,
104. Ans (A)
2 3
1 − y6
4 5
1 + 2x + (2x) + (2x) + (2x) + (2x) = 9 99 999 9999
1−y + + + +⋯
19 192 193 194
Here a=1, r=2x & n=6, then sum of n-terms of G.P 10−1 102 −1 103 −1 104 −1
= + + + +⋯
1((2x)6 −1) −(y6 −1) 19 192 193 194
is =
2x−1 −(y−1) 10 10 2 10 3 1 1 2
y
=( + ( ) + ( ) + ⋯ ∞) − ( +( ) +
19 19 19 19 19
Solve and take take, 2x = y ⇒ = 2
x
1 3
100. Ans (C) ( ) + ⋯ + ∞)
19
a = 4, b = 2916 10 1 10 1
= 19 − 19 = 19 − 19
a, G1 , G2 , … G2n+1 , b are in G.P 10 1 9 18
1− 1−
b = a ⋅ r 2n+2 19 19 19 19
10 1 20 − 1 19
2916 = 4r 2n+2 ⇒ 729 = r 2n+2 = − = =
9 18 18 18
Rise power to 2n+2 on both sides,
105. Ans (B)
1
n+1
r= (729)2n+2 ⇒ Gn+1 = a ⋅ r Let α be the 1st term and d be the common
difference, then a= α + 10d,b= α + 14d
MATHEMATICS Page |10A. 12
SEQUENCES AND SERIES
& c= α + 16d ∴ r = ±2
Since a, b &c are in G.P 109. Ans (A)
(α + 14d)2 = (α + 10d)(α + 16d) Let the numbers are a, ar, ar 2
⇒ α2 + 196d2 + 28αd = α2 + 26αd + 160d2 a + ar + ar 2 = 38
⇒ 36d2 = −2αd a(1 + r + r 2 ) = 38 and a. ar ⋅ ar 2 = 1728
⇒ α = −18d ⇒ a3 r 3 = 1728
a+b ⇒ (ar)3 = 1728
∴ =6
c 1
⇒ ar = (1728)3
106. Ans (C)
S5 ⇒ ar = 12 … (ii)
⇒ = 49 (Here, S5 = sum of first 5 terms
s′5
a(1 + r + r 2 ) = 38 … (i)
and s5′ = sum of their reciprocals) From equations (i) and (ii)
5
a1 (r1 − 1) 12
(r − 1) (1 + r + r 2 ) = 38
⇒ −1 1 −5 = 49 r
a (r − 1)
(r −1 − 1) ⇒ 12(1 + r + r 2 ) = 38r
⇒ (−1)n−1 5n = (5)5 let a be the first term and r be the common ratio,
then
⇒ (−1)n−1 5n = (−1)5−1 (5)5
a + ar = 1 … (i)
∴n=5
and ar = 2a[∵ a 2 = 2a1 ]
108. Ans (B)
⇒ r = 2 …. (ii)
Given 6th term, a 6 = 32 and 8th term,
from equation (i) and (ii)
a 8 = 128
3a = 1
Let a be the first term and r be the common ratio
1
of the GP, then, a 6 = ar 5 = 32 ∴a=
3
[∵ a n = ar n−1 ]
111. Ans (C)
and a 8 = ar 7 = 128 Since, a1 , a 2 , a 3 , … … a 50 are in GP
ar 7 128
= =4 Let a1 = a, a 2 = ar, a 3 = ar 2
ar 5 32
a 4 = ar 3 , a 49 = ar 48 , a 50 = ar 49
⇒ r2 = 4
Now, using the above terms
MATHEMATICS Page |10A. 13
SEQUENCES AND SERIES
a1 − a 3 + a 5 … a 49 a 7 ± 3√5
a 2 − a4 + a 6 + ⋯ a 50 =
b 2
a − ar 2 + ar 4 … ar 48 a
Since, > 0
= b
ar − ar 3 + ar 5 + ⋯ + ar 49
a 7 + 3√5
a(1 − r 2 + r 4 … … r 48 ) 1 =
= = b 2
ar(1 − r 2 + r 4 … r 48 ) r
114. Ans (B)
a1 a 1
Also, = = If a, b, c are in G.P. Then b2 = ac
a 2 ar r
a1 − a3 + a 5 + ⋯ + a 49 a1 ⇒ b2 (a − c) = ac(a − c)
=
a 2 − a4 + a 6 + ⋯ + a 50 a 2 ⇒ b2 a − b2 c = a2 c − ac 2
112. Ans (B) ⇒ a(b2 + c 2 ) = c(a2 + b2 ).
Let five GM's be G1 , G2 , G3 , G4 , G5 between 486 and Trick: Put a = 1, b = 2, c = 4 and check the
2
3
alternates.
2
So, 486, G1 , G2 , G3 , G4 , G5 , are in GP 115. Ans (D)
3
Given sequence is √2, √10, √50 … ….. Common
Here, a = 486, n = 7
th
2 ratio r = √5, first term a = √2, then 7 term t 7 =
and l =
3
√2(√5)7−1
Let r be the common ratio of the given GP
2 = √2(√5)6 = √2(5)3 = 125√2.
∴ = ar n−1 [∵ l = ar n−1 ]
3 116. Ans (C)
2 Let first term of G.P. = A and common ratio = r
⇒ = 486. r 6
3
We know that nth term of G.P.
2 1
⇒ r6 = = = Ar n−1
3 × 486 729
1 1/6 1 1/6 1 Now t 4 = a = Ar 3 , t 7 = b = Ar 6 and t10 = c = Ar 9
⇒r=( ) = ( 6) =
729 3 3 Relation b2 = ac is true because b2 = (Ar 6 )2 =
∴ G4 = ar 4 A2 r12 and ac = (Ar 3 )(Ar 9 ) = A2 r12
1 4 486 Aliter : As we know, if p, q, r in A.P., then
= 486 × ( ) = =6
3 81 pth , qth , r th terms of a G.P. are always in G.P.,
113. Ans (B)
therefore, a, b, c will be in G.P. i.e. b2 = ac.
AM 3
= ⇒ a + b = 3√ab 117. Ans (B)
GM 2
Given that first term a = 5 and common ratio r =
a b
√ +√ =3 −5 . Suppose that nth term is 3125,
b a
then ar n−1 = 3125
a
put, √ = t ⇒ 5(−5)n−1 = 3125 ⇒ (−5)n−1 = 54
b
1 Hence n = 5.
t+ =3
t 118. Ans (B)
2
t − 3t + 1 = 0 Suppose that the added number be x then x +
−b ± √b 2 − 4ac 2, x + 14, x + 62 be in G.P.
t=
2a Therefore (x + 14)2 = (x + 2)(x + 62)
3 ± √9 − 4 ⇒ x 2 + 196 + 28x = x 2 + 64x + 124
=
2
⇒ 36x = 72 ⇒ x = 2.
a 3 ± √5 a 14 ± 6√5
√ = ⇒ =
b 2 b 4
Trick: (a) Let 1 is added, then the numbers will be 124. Ans (A)
3,15,63 which are obviously not in G.P. a = ARp−1 , b = ARq−1 , c = ARr−1
(b) Let 2 is added, then the numbers will be 4,16, c p b r a q
∴( ) ( ) ( )
64 which are obviously in G.P. b a c
p r q
ARr−1 ARq−1 ARp−1
119. Ans (A) =( ) ( ) ( )
ARq−1 ARp−1 ARr−1
Under condition Tn = Tn+1 + Tn+2 = R(r−q)p+(q−p)r+(p−r)q = R0 = 1
n−1 n n+1
⇒ ar = ar + ar 125. Ans (A)
n−1 n 2
⇒r = r (1 + r) ⇒ r + r − 1 = 0 Accordingly, ar 9 = 9 and ar 3 = 4
−1 ± √1 + 4 −1 ± √5 ⇒ r 3 = and a = .
3 8
⇒r= = 2 3
2 2
8 3 2
Since, each term is +ve. ∴ 7th term i.e. ar 6 = ( ) = 6.
3 2
√5−1
Hence common ratio is . Trick: 7th term is equidistant from 10th and 4th so
2
x
] −1 10 −1 n−1
⇒( ) =( ) ⇒ 10 = n − 1
8 2 2
= (x + 1)3
x2 ⇒ n = 11.
Putting x = −4 128. Ans (C)
8 27
We get T4 = 3
(−3) = − = −13.5. a2 = ar 2 ⇒ a = r 2 . Also ar = 8 ⇒ r = 2 and a = 4.
16 2
1 1. (1091 − 1)
r= = 0.1 =
10 10 − 1
1 (1013 )7 − 1 1013 − 1
1 − r100 9 1 − 10100 = ×
∴ S100 = a( )= ( ) 1013 − 1 10 − 1
1−r 10 1 − 1
10 = [(1013 )6 + (1013 )5 + (1013 )4 (1012 +
=1−
1 1011 +……+1)
10100
It is the product of two integers and hence not
133. Ans (D)
prime.
Let first term and common ratio of G.P. are
137. Ans (B)
respectively a and r, then under condition,
1
Tn = Tn−1 + Tn−2 The sequence is a G.P. with common ratio .
3
⇒ ar n−1
= ar n−2
+ ar n−3 a(1−rn ) 2[1−(1/3)20 ] 1
Now from , = 3 [1 − ].
1−r 1−(1/3) 320
n−1 n−1 −1 n−1 −2
⇒ ar = ar r + ar r 138. Ans (B)
1 1
⇒ 1 = + 2 ⇒ r2 − r − 1 = 0 a1 = 3, a n = 96 ⇒ a1 r n−1 = 96
r r
1 ± √1 + 4 1 + √5 ⇒ r n−1 = 32 ⇒ r n = 32r
⇒r= =
2 2 a1 (r n − 1)
Taking only (+) sign (∵ r > 1). Sn = = 189
r−1
134. Ans (A) 3(32r − 1)
⇒ = 189
a(rn −1) r−1
Given that = 255 (∵ r > 1)
r−1
Hence r = 2 and n = 6.
ar n−1 = 128
139. Ans (A)
and common ratio r = 2
Let three number of G.P. are a, ar, ar 2
n−1
we get a(2) = 128
a + ar + ar 2 = 38
a(2n −1)
and = 255 a(1 + r + r 2 ) = 38 … (i)
2−1
(ar n−1 ⋅ r − a) 2 2
⇒ = 889 ⇒8= (∵ a = )
r−1 r(1 − r) r
⇒
448r−7
= 889 ⇒ r = 2. ⇒ 4r(1 − r) = 1 ⇒ 4r − 4r 2 − 1 = 0
r−1
⇒ 4r 2 − 4r + 1 = 0
141. Ans (A)
1 1
arn −a arn−1 ⋅r−a ⇒ (r − ) (4r − 2) = 0 ⇒ r =
= 364 ⇒ = 364 2 2
r−1 r−1
3 × 243 − a So, first term a = 4.
⇒ = 364 ⇒ a = 1
2 148. Ans (D)
Now, putting this in (i), n = 6. x x
y= ⇒y=
1−(−x) 1+x
142. Ans (A)
y
⇒ y + yx = x ⇒ x = .
Let α and β be the roots of equation 1−y
√2+1
,
1 1
, , … …. S1 = 3(1) + 2(1)2 = 5
√2−1 √2(√2−1) 2
S2 = 3(2) + 2(4) = 14
1
Common ratio of the series =
√2(√2+1) S1 = a1 = 5
a
∴ sum = S2 − S1 = a 2 = 14 − 5 = 9
1−r
∴ Common difference d = a 2 − a1
√2 + 1 1
=( ) / (1 − ) =9−5=4
√2 − 1 √2(√2 + 1)
(√2+1) √2(√2+1)
159. Ans(C)
= ⋅ = √2(√2 + 1)2 .
(√2−1) (1+√2) Given that T3 = 4
153. Ans (A) ⇒ ar 3−1 = 4
2
Common ratio r =
x
⇒ ar 2 = 4
⇒ 3z = y and z ≠ y 3n n
= [2a + (3n − 1)d]: [2a + (n − 1)d]
z 1 1 2 2
⇒ = ⇒ r= 3n
y 3 3 [2a + (3n − 1)d]
= 2n
[∵ z and y are distinct numbers ] [2a + (n − 1)d]
2
162. Ans(C)
3[2a + (3n − 1)d]
The given series is A.P. whose first term is a and =
2a + (n − 1)d
common difference is d 3[(n + 1)d + (3n − 1)d]
=
n (n + 1)d + (n − 1)d
∴ Sn = [2a + (n − 1)d] = qn2
2 3d[n + 1 + 3n − 1] 3[4n]
= 2a + (n − 1)d = 2qn = = =6
d(n + 1 + n − 1) 2n
m
Sm = [2a + (m − 1)d] = qm2 164. Ans(B)
2
We know that AM ≥ GM
⇒ 2a + (m − 1)d = 2qm
4x + 41−x
Solving eq. ( i ) and eq. (ii) we get d=2q ∴ ≥ √4x ⋅ 41−x
2
Putting the value of d in eq. (ii) we get
⇒ 4x + 41−x ≥ 2√4x+1−x
2a + (m − 1) ⋅ 2q = 2qm
⇒ 4x + 41−x ≥ 2 ⋅ 2 ⇒ 4x + 41−x ≥ 4
⇒ 2a = 2qm − (m − 1)2q
165. Ans(A)
⇒ 2a = 2q(m − m + 1)
Sr S1 S2 S3 Sn
Given that ∑ni=1 = + + + ⋯+
⇒ 2a = 2q ⇒ a = q sr s1 s2 s3 sn
t 50 = 2 + (1 + 3 + 5 + 7 + ⋯ upto 49 terms ) a = 4 − 4r
49 3
t 50 = 2 + [2 × 1 + (49 − 1)2] = 4r − 4r 2
2 4
49 16r 2 − 16r + 3 = 0
=2+ [2 + 96]
2 1 3
49 r = ( or )r =
=2+ × 98 = 2 + 49 × 49 = 492 + 2 4 4
2 1
167. Ans(A) r= ⇒a=3
4
Let the length, breadth and height of a rectangular 169. Ans(A)
a
block be , a and ar. ar = 24, ar 4 = 3
r
V=l×b×h ar 4 3 1
r3 = = =
a ar 24 8
= × a × ar 1
r r = ⇒ a = 48
2
a3 = 216
⇒ a + ar + ar 2 + ⋯ … … + ar 5
⇒a=6
1
Now total surface area = 2[lb + bh + lh] a(1 − r 6 ) 48 [1 − 64]
= =
a a 1−r 1
252 = 2 [ ⋅ a + a ⋅ ar + ⋅ ar] 1−
2
r r
63 3 × 63 189
a2 = 48 × ×2 = =
⇒ 252 = 2 [ + a2 r + a2 ] 64 2 2
r
170. Ans(D)
1 2
⇒ 252 = 2a [ + r + 1] a=a
r
1 + r2 + r b=a+d
⇒ 252 = 2 × (6)2 [ ]
r c = a + 2 d & x=x
2
1+r +r y = xr, z = xr 2
⇒ 252 = 72 [ ]
r ∴ x b−c ⋅ y c−a ⋅ z a−b
2
252 1 + r + r = r c−a ⋅ r 2a−2b = r c+a−2b = r 0 = 1
⇒ =
72 r
171. Ans (B)
7 1 + r + r2
= ar 2 = 9
2 r
⇒ 2 + 2r + 2r 2 = 7r a ⋅ ar ⋅ ar 2 ⋅ ar 3 ⋅ ar 4 = a5 ⋅ r10
2r 2 − 5r + 2 = 0 = (ar 2 )5 = 95 = 310
(2a1 − d) + pd p
⇒ a = −24d ⇒ =
(2a1 − d) + qd q
∴ T25 = −24d + 24d = 0
⇒ (2a1 − d)(p − q) = 0
14. Ans (C)
d
Let Sn denote the sum of n terms. Then, ⇒ a1 =
2
Sn = 3n2 + 5 Now,
Now, a6 a1 + 5d
=
a n = Sn − Sn−1 a 21 a 2 + 20d
⇒ a n = (3 n2 + 5) − (3(n − 1)2 + 5) = 6n − 3 d
+ 5d 11
= 2 =
∴ a n = 159 ⇒ 6n − 3 = 159 d
+ 20d 41
2
⇒ 6n = 162 ⇒ n = 27
20. Ans (A)
15. Ans (C)
Let α and β are the roots of the equation
Let Sn denote the sum of n terms of the given
x 2 − 2ax + a2 = 0
series.
∴ α + β = 2a and αβ = a2 …(i)
Then,
α+β
Since, A = and G = √αβ
Sn = 5n2 + 2n 2
Second term = S2 − S1 ⇒ G2 = A2 ⇒ G = A
(a. r n−1
. r − a)
Sn =
r−1
Given, Sn = 364
(a.rn−1 .r−a)
So, = 364
r−1
=2×2×2=8 = 9 × 8 × 7 × 6 = 3024.
options for this place are 2 and 4 only and not allowed)
the ten's place can be filled by any of the 5 (as hundred's, ten's and unit's places can be
digits in 5 different ways as the digits can be filled in 5, 4 and 3 ways respectively)
n−1 P
4! 24 3 1
M(ALLS) → = = 12 I. Here, nP =9
2! 2 4
3! (n − 1)(n − 2)(n − 3) 1
SA(MLL) → = 3 ⇒ =
2! n(n − 1)(n − 2)(n − 3) 9
SL(ALM) → 3! = 6 1 1
⇒ = ⇒n=9
Total words = 12 + 24 + 12 + 3 + 6 n 9
5!
5P
= 57 II. Here, r
=2⇒ (5−r)!
6! =2
6P
r=1
[6−(r−1)!
∴ the position of the word SMALL is 58th
5! (7 − r)!
19. Ans (A) ⇒ × =2
(5 − r)! 6!
As shown in figure 1, 2 and X are the three
5! (7 − r)(6 − r)(5 − r)!
boys and 3, 4 and Y are three girls, Boy X will ⇒ × =2
(5 − r)! 6 × 5!
have neighbours as boys 1 and 2 and the girl (7 − r)(6 − r)
Y will have neighbours as girls 3 and 4 ⇒ =2
6
1 and 2 can be arranged in → 42 − 13r + r 2 − 12
= 2! = 2 × 1 = 2 ways ⇒ r 2 − 13r + 30 = 0
Also, 3 and 4 can be arranged in = 2! = ⇒ (r − 10)(r − 3) = 0
2 × 1 = 2 ways ⇒ r = 10,3
Hence, required no. of permutations ( r = 10 rejected as 5 P10 and 6 P9 are not
defined) ∴ r = 3
22. Ans (D)
In the word MONDAY, all letters are different.
I. Out of 6 different letters, 4 letters can be
selected in 6 P4 ways.
∴ Required number of words = 6 P4
6! 6!
= =
=2×2=4 (6 − 4)! 2!
20. Ans (A) 6 × 5 × 4 × 3 × 2!
= = 360
The number of ways in which 5 beads of 2!
different colors can be arranged in a circle to II. The word 'MONDAY' has 6 different letters.
(5 − 1)! = 4 ! time = 6 P6
II. The word 'MISSISSIPPI' has 11 letters in Hence, by FPC total number of ways selecting
12! 8! 12! r 36
= × ×1= (1) ÷ (2) gives = 84
n−r+1
8! × 4! 4! × 4! (4!)3
⇒ 84r = 36n − 36r + 36
30. Ans (B)
n
Cr n−r+1 15
Cr 16 − r ⇒ 120r = 36n + 36 … (4)
nC
= ⇒ 15 C
= r+1 84
r−1 r r−1 r (2) ÷ (3) gives = 126
n−r
16 − r
∴ ∑15 2
r=1 r ( ) = ∑15 2
r=1 (16r − r )
⇒ 126r + 126 = 84n − 84r
r
15×16 15×16×31 ⇒ 210r = 84n − 126 … (5)
= 16 × 2
− 6
[∵ ∑nr=1 r 2 =
Solving (4) and (5) n = 9, r = 3
n(n+1)(2n+1)
6 So n C8 = 9 C8 = 9
= 680 35. Ans (B)
31. Ans (B) Total number of players = 16
We have, x C15 = x C14 ∴ No. of ways of selecting 11 players out of 16
16
⇒ x = 14 + 15 players= C11 .
⇒ x = 29 (i) If 2 particular players are included, then
[∴ n Cz = n Cy number of ways
16−2 14
⇒ z = y or n = z + y] = C2 = C9
So, x C29 = 29
C29 = 1 (ii) If 2 particular players are excluded, then
32. Ans (A) number of ways
n
C3 + n C4 > n+1
C3 = 16−2
C11 = 14
C11
n+1 n+1
⇒ C4 > C3 36. Ans (D)
(∵ n Cr + n Cr+1 = n+1
Cr+1 ) Given, 32 P6 = k( 32 C6 )
n+1 32! 32! n!
C4 ⇒ (32−6)! = k ⋅ 6!(32−6)! [∵ n Pr = (n−r)! and
⇒ n+1 > 1
C3
n n!
n−2 Cr = r!(n−r)!]
⇒ >1⇒n>6
4 k
⇒ 1 = 6! ⇒ k = 6 !
33. Ans (C)
⇒ k = 6 × 5 × 4 × 3 × 2 × 1 = 720
D, R, G, H, T → 5 units
r
= n Cr
r!
Total = 6 units ∑nr=1 n Cr = n C1 + n C2 + ⋯ n Cn
n
R. A = 6! × 2 ! =2 −1
= 720 × 2 = 1440 54. Ans(D)
46. Ans(A) Given word is ARTICLE
X−X−X−X−X R. A = (7−1) C(4−1) × 4!
' X ' for boys, ' - ' for girls = 6 C3 × 4 !
R. A = 5! × 4 P3 = 20 × 24
The number of ways = 4(3! 5!) = 480
= 4(720) = 2880 55. Ans(B)
47. Ans(D) Given digits 0,2,4,5
×B×B×B×B×B×B×B× The number of four-digit numbers
× represents position for girls = 3 × 3! = 18
R.A = 7! × 8 P3 The number of four-digit number which are
= 42 × 8! divisible by 5 is 10
48. Ans(B)
R. A = 6 P1 + 6 P2 + 6 P3 + 6 P4 + 6 P5 + 6 P6
= 6 + 30 + 120 + 360 + 720 + 720 = 1956
49. Ans(D)
R. A = 4 P2 × 6 P3
50. Ans(D) ∴ R. A = 18 − 10 = 8
A, C, H, I, N, S 56. Ans(A)
Number of words start with A is 5! 3,4,5,6 → 18
Number of words start with C is 5! 2,4,5,6 → 17
Number of words start with H is 5! 2,3,5,6 → 16
Number of words start with i is 5! 2,3,4,6 → 15
Number of words start with N is 5! 2,3,4,5 → 14
⇒ total number of words hence formed Number of way = 2(4!) = 48
= 5(5!) = 600 57. Ans(A)
12
∴ 601th word start with S Pr = 11 P6 + 6. 11 P5
i.e., SACHIN r = 6 (∵ (n−1) Pr + r ⋅ (n−1) Pr−1 = n Pr )
51. Ans(C) Alternatively, verify the options
Given digits are 0,1,2,3,4,5,6 58. Ans(B)
R. A = 5 P4 (Repetations are not allowed)
59. Ans(A)
Given digits 3,4,5,6,7
Required sum
= (n − 1) ! (sum of digits) (11111)
= 4! (25) (1111) = 6666600
60. Ans(B)
R ⋅ A = 300 + 120 = 420
Now, we have been given 15 stations in a
52. Ans(D)
railway line and we need to find the total
MATHEMATICS Page | 11A. 7
PERMUTATIONS AND COMBINATIONS
n(n − 1)
⇒ = 66
2 Number of
⇒ n(n − 1) = 12 × 11 3B 2G 4C
Selection
So, n = 12
87. Ans(B) 0 2 2 3
c0 × 2 c2 × 4 c2
Total no.f ways
= 5 C4 × 8 C6 + 5 C5 × 8 C5 2 0 2 3
c2 × 2 c0 × 4 c2
= 140 + 56 = 196
3
1 1 2 c1 × 2 c1 × 4 c2
5 8 13
5 8 10 94. Ans(A)
C4 C6
The number of ways 52 cards can be divided
5 8 10
C5 C5 among 4 players so that each have 13 cards =
(52)!
4!(13!)4
×4!
88. Ans(C) 95. Ans(A)
Let the number of teams = n Required number of ways = 3!(2!)3 ⋅1!
7!
n
C2 = 153
7×6×5×4
n(n − 1) = 51 × 3 × 2 =
8
= 17 × 18,n = 18
= 105
89. Ans(D)
96. Ans(D)
Required number of selections
n = 12
= (12−1) C(5−1)
P = 5 (collinear points).
= 11 C4 The no. of straight lines = n C2 − P C2 + 1
90. Ans(B) = 12 C2 − 5 C2 + 1
C1 C2 Number of selection
8
= 66 − 10 + 1 = 57
4 4 C4 × 4 C4
8
97. Ans(D)
5 3 C5 × 3 C3
The no. of triangles = 12 C3 − 7 C3
Total number of ways = 8 C4 × 8 C5
= 220 − 35 = 185
= 70 + 56
98. Ans(C)
= 126
Number of positive numbers
91. Ans(C)
= (5 + 1)(6 + 1)(3 + 1)
Required number of selections
= 6.7.4 = 168
= n C2 − n = 12 C2 − 12
99. Ans(A)
= 66 − 12 = 54 2n C
3 44
92. Ans(D) nC = 3
⇒n=6
2
Given there are three boxes, each containing Required: nPn=n!=6!=720
10 balls labelled 1, 2, 3, …, 10. Now, one ball is 100. Ans(A)
randomly drawn from each box, and ni denote Given that n C12 = n C8
the label of the ball drawn from the ith box, (i ⇒ n C12 = n Cn−8
= 1, 2, 3). Then, the number of ways in which [∵ n Cr = n Cn−r ]
the balls can be chosen such that n1 < n2 < n3 ∴ n − 8 = 12
is same as selection of 3 different numbers ⇒ n = 12 + 8 = 20
from numbers {1, 2, 3, …, 10} = 10C3 = 120. 101. Ans(A)
93. Ans(A) We know that a coin has Head and Tail (H, T)
60 60 10 ∴ When a coin is tossed 6 times, the number
P(E) = 9 = =
C4 9 × 7 × 3 21 of Possible outcomes = 26 = 64
If digits are not repeated, then 5-digit Possible number of choosing 4 blue dyes = 24
telephones, can be formed in 10 P5 ways and possible number of choosing 3 red dyes
∴ Required number of ways = 23
= (10)5 − 10 P5 If at least one blue and one green dyes are
10! selected then the total number of selection
= 100000 −
(10 − 5)! =(25 − 1) × (24 − 1) × 23
10 × 9 × 8 × 7 × 6 × 5! = 31 × 15 × 8 = 3720
= 100000 −
5! 115. Ans(B)
= 100000 − 30240 = 69760 R. A = 1 × 1 × 8 P3 = 8 × 7 × 6
111. Ans(A) 116. Ans(C)
Number of men = 4 n(A) = 4, n(B) = 5
Number of women = 6 The no. of injective mappings from A to B
We are given that the committee includes 2 n(B)
men and exactly twice as many women as Pn(A) = 5 P4 = 120
men. 117. Ans(D)
Thus, the possible selection can be 5G, 3 B
2 men and 4 women and 3 men and 6 women. ×G×G×G×G×G×
So, the number of committees Arrangement of 5girls is 5!
= 4 C2 × 6 C4 + 4 C3 × 6 C6 Arrangement of 3 boys in cross marked
=6 × 15 + 4 × 1 = 90 + 4 = 94 places is 6P3
112. Ans(C) R. A = 5 ! × 6P3 = 120 × 6 × 5 × 4
We have to form 9-digit numbers from = 14400
0,1,2,3,4,5,6,7,8,9 and we know that 0 can not 118. Ans(B)
be put on extremely left place. The number of persons = n
So, first place from the left can be filled in 9 nc2 = 45
ways. n(n − 1) = 10 × 9
Now repetition is not allowed. So, the n = 10
remaining 8 places can be filled in 9! 119. Ans(D)
∴ The required number of ways = 9 × 9 ! odd = 1,3,5,7
113. Ans(B) even = 2,4,6
Total number of letters in the 'ARTICLE' is 7 R. A = 3 C2 × 4 C2 × 4!
out which A, E, I are vowels and R, T, C, L are = 3 × 6 × 24 = 432
consonants 120. Ans (B)
Given that vowels occupy even place n(A × B) = 35 = 7 × 5, 7C5 = 7C2 = 21
∴ possible arrangement can be shown as 121. Ans (D)
below C, V, C, V, C, V, C i.e. on 2nd , 4th and 6th AKMS
places Number of words start with A → 3!
Therefore, number of arrangements = 3 P3 = Number of words start with K → 3!
3! = 6 ways Number of words start with M → 3!
Now consonants can be placed at 1,3,5 and 7 ∴ 19th word start with S i. e. , SAKM
th place 122. Ans (𝐂) or (Grace)
∴ Number of arrangements = 4 P4 = 4! = 24 6c3 × 4c2 or 6c3 × 7c2
So, the total number of arrangements
= 6 × 24 = 144
123. Ans (𝐀)
114. Ans(B)
124. Ans (𝐂)
Possible number of choosing 5 different
125. Ans (𝐂)
green dyes = 25
7!
= 2520 26. Ans(C)
2 n
C2 − n = 44
18. Ans (A)
n(n − 1)
Reqd. no. of ways ⇒ − n = 44
2
=(4 − 1)! 4P3 = 6 × 4! ⇒ n2 − 3n − 88 = 0
= 6 24 = 144 ⇒ n = 8 or n = 11
19. Ans (A) 27. Ans(D)
(6−1)! 6P3
Reqd. no. of ways = For each of the things there are two
2
1
HINTS AND SOLUTIONS
∴n×2=8⇒n=4 32 40 20 5x 3 x 5
= 5
− 3+ − 5x + −
4. Ans (C) x x x 8 32
10. Ans (A)
√5[(√5 + 1)50 − (√5 − 1)50 ]
We know that,
= 2√5[ 50 C1 (√5)49 + 50
C3 (√5)47 + ⋯ ]
2n−1 = n C0 + n C2 + n C4 + ⋯
50 50 50 48
= 2[ C1 (√5) + C3 (√5) + ⋯] = n C1 + n C3 + n C5 + ⋯
is a natural number ⇒ 10
C1 + 10
C3 + 10
C5 + ⋯ + 10
C9
5. Ans (B) =2 10−1
=2 9
expansion of
9+1
26. Ans(C)
(1 + 5√2x)9 + (1 − 5√2x)9 has ( ) = 5 terms
2
We have, (a2 x 2 − 2ax + 1)51
22. Ans (C)
The sum of the binomial coefficient of the
Let the consecutive coefficient of
expansion is (a2 − 2a + 1)51
n n n n
(1 + x) are Cr−1 , Cr and Cr+1
The sum of the coefficient of the expansion is
From the given condition, n Cr−1 : n Cr : n Cr+1 =
vanishes, if a2 − 2a + 1 = 0
6: 33: 110
⇒ (a − 1)2 = 0 ⇒ a = 1
n n
Now Cr−1 : Cr = 6: 33
27. Ans(C)
n! r! (n − r)! C1 C2 Cn n
⇒ × C0 + x + x2 + ⋯ + x
(r − 1)! (n − r + 1)! n! 2 3 n+1
6 (1 + x)n+1 − 1
= =
33 (n + 1)x
r 2
⇒ = put x = 1
n − r + 1 11
⇒ 11r = 2n − 2r + 2 C1 C2 Cn 2n+1 − 1
⇒ C0 + + + ⋯+ =
2 3 n+1 n+1
⇒ 2n − 13r + 2 = 0
Alternatively, verify options for n=1, 2
and n Cr : n Cr+1 = 33: 110
28. Ans(D)
n! (r + 1)! (n − r − 1)!
n
⇒ × Cr + 2n Cr−1 + n Cr−2
r! (n − r)! n!
33 3 = n Cr + n Cr−1 + n Cr−1 + n Cr−2
= = n+1 n+1
110 10 = Cr + Cr−1
(r + 1) 3 = n+1+1
Cr = n+2
Cr
⇒ =
n−r 10
29. Ans(A)
⇒ 3n − 13r − 10 = 0
C02 + C12 + C22 + ⋯ + Cn2 = 2nCn (n = 50)
Solving (i) & (ii), we get n = 12
2 2 2 2
∴ (50C0 ) + (50C1 ) + (50C2 ) + ⋯ … + (50C50 )
23. Ans (A)
Using T.S.R = 100C50
32. Ans(A) ∴ na = 8 ⇒ n = 4
( 21 C1 + 21
C2 … + 21
C10 ) 39. Ans (C)
− ( 10 C1 + 10
C2 … . 10
C10 ) Coefficient of x17 = −
n ×(n+1)
2
1
= [( 21 C1 + ⋯ + 21
C10 ) + ( 21
C11 + ⋯ + 21
C20 )] 18 × 19
2 =− = −171
2
− (210 − 1)
40. Ans (C)
1
= [221 − 2] − (210 − 1) (1 + x 2 )5 (1 + x)4
2
= (1 + 5x 2 + 10x 4 + 10x 6 + 5x 8 + 10)(1 + 4x
= (220 − 1) − (210 − 1) = 220 − 210
+ 6x 2 + 4x 3 + x 4 )
33. Ans(A)
Coefficient of x 5 = 20 + 40 = 60 (use Pascal
We have, (2x 2 − 3x + 1)
Sum of all coefficient for the given expansion, we triangle)
∴ (2 − 3 + 1)11 = 0 (1 − x)5 (1 + x + x 2 + x 3 )4
1 + n. (ax) +
n(n−1) 2 2
a x + ⋯ . = 1 + 8x + 24x 2 + 45. Ans (D)
2
Standard result
⋯
n(n−1) 2
C1 C3 C5 2n − 1
⇒ na = 8 and a = 24 + + +⋯ =
2 2 4 6 n+1
n(n − 1) 2 8 8 Alternatively, verify the options for n=1, 2
a = 24 ⇒ ( − 1) a2 = 48
2 a a 46. Ans (B)
⇒ (8 − a) = 6
Sum of the even binomial coefficients
⇒a=2
= 2n-1 ,Here n = 9,
MATHEMATICS Page |12A. 4
BINOMIAL THEOREM
∑10
k=0
20
Ck = 20
C0 + 20
C1 + 20
C2 + ⋯ + 20
C10 Given 1! + 2! + 3!+. . . +11!
We have = (1! + 2! + 3!) + (4! + 5!. . . +11!)
220 = 2( 20 C0 + 20
C1 + 20
C2 + ⋯ + 20
C9 ) = 9 + (4! + 5!. . . +11!)
20 Note that 4!, 5!, ..., 11! are divisible by 12
+ C10
⟹ 219 = ( 20 C0 + 20
C1 + 20
C2 + ⋯ + 20
C9 ) ∴ Remainder = 9
1 20
65. Ans(D)
+ C10
2 1 14
⟹2 19
= 20
C0 + 20
C1 + 20
C2 + ⋯ + 20
C10 (x + )
√x
1 10
− 20
C10 14
1
2 T11 = C10 (x)4 ( )
20 20 20
√x
⟹ C0 + C1 + ⋯ + C10
1 14 × 13 × 12 × 11 1001
1 = =
= 219 + 20
C10 x 4×3×2×1 x
2
66. Ans(A)
60. Ans (D)
(x + a)47 − (x − a)47
5
x = (√3 + 1) n+1 48
The no. of terms = = = 24
2 2
5 4 3
5 5
= (√3) + C1 (√3) + C2 (√3) 67. Ans(C)
2 n+1
+ 5 C3 (√3) + 5 C4 (√3) + 5 C5 Number of terms = = 13
2
= 9√3 + 45 + 30√3 + 30 + 5√3 + 1 68. Ans(D)
16 16 16 16
= 76 + 44√3 We have, C9 + C10 − C6 − C7
5 17 17 n n n+1
∴ [x] = [(√3 + 1) ] = [76 + 44√3] = C10 − C7 (∵ Cr + Cr−1 = Cr )
17 17
= C10 − C10 = 0
= [76] + [44 × 1.732]
n
(∵ Cr = Cn−r ) n
= 76 + [76.2]
69. Ans(D)
= 76 + 76 = 152
(n+1)(n+2) 11×12
61. Ans (B) Required number of terms= = =
2 2
Trick: Solving conversely. coefficients in the 2nd bracket in which all the
⇒ 15
C8 + 15
C9 + ⋯ + 15
C15 = 214 We have, (1 + x)50 = ∑50
r=0
50
Cr x r .
1. Ans (C) 1 1
= =
We know that greatest integer function is not 4+2+1 7
7. Ans (B)
continuous at integer. So, limx→n [x] does not exist.
sin x(2 cos x − a)
Alternatively, at all integral points LHL≠RHL limx→0
x ⋅ x2
So, limit does not exist. sin x 2cos x − a
= limx→0 ( )( )
2. Ans (C) x x2
x For this limit to exists finitely,
[ ]
∴ limx→π 3 =0 2cos x−a
2 ln(1 + cot x) limx→0 = finite
x2
π π 0
Here [ ] = 0, Because < 1 ∴ It must be form
6 6 0
L.H.L = limx→−2−
|x+2| limx→3 [x(x + 1)] = 3(3 + 1) = 3(4) = 12
x+2
10. Ans (B)
−(x + 2)
= limx→−2 = −1 Put y = 1 + x,
x+2
R.H. L = limx→−2+
|x+2| so that y → 1 as x → 0
x+2
√1+x−1 √y−1
x+2 Then limx→0 = limy→1
= limx→−2 =1 x y−1
x+2 1
Since L. H. L. ≠ R. H. L. y2
−1 1 1 1
= limy→1 = (1)2−1 =
y−1 2 2
∴ Limit does not exist.
11. Ans (C)
5. Ans (B)
(1 − cos 2x)(3 + cos x) (2x − 3)(√x − 1)
limx→0 lim
x tan 4x
x→1 2x 2 + x − 3
(2x − 3)(√x − 1)
(2sin2 x)(3 + cos x) = lim
= limx→0 x→1 (2x + 3)(x − 1)
xtan 4x
(2x − 3)(√x − 1)
sin x 2 4x 1 = lim
= limx→0 ( ) ( ) (3 + cos x) × = 2 x→1 (2x + 3)(√x + 1)(√x − 1)
x tan 4x 2
(2x − 3)
6. Ans (D) = lim
x→1 (2x + 3)(√x + 1)
x−2
We have, limx→2 ( ) 2×1−3 1
x3 −x2 −x−2 = = −
x−2 (2 × 1 + 3)(√1 + 1) 10
= limx→2 ( )
(x − 2)(x 2 + x + 1) 12. Ans (D)
1 1 log(1 + sin 10x)
= limx→2 ( 2 ) limx→0
x +x+1 2 tan 3x
MATHEMATICS Page | 13A. 1
L LIMITS AND DERIVATIVES
x ln x x3 x2
= lim+ + lim+ lim ( − )
x→0 tan x x→0 1 x→∞ 3x 2 − 4 3x + 2
( )
x x 3 (3x + 2) − x 2 (3x 2 − 4)
1 = lim ( )
x ( ) x→∞ (3x 2 − 4)(3x + 2)
= lim+ + lim+ x = 1 + lim+ (−x) 3x 4 + 2x 3 − 3x 4 + 4x 2
x→0 tan x x→0 1 x→0
(− 2 ) = lim ( )
x x→∞ 9x 3 + 6x 2 − 12x − 8
= 1 + 0 = 1
2x 3 + 4x 2
14. Ans (C) = lim ( 3 )
x→∞ 9x + 6x 2 − 12x − 8
x1/4 − x1/5 4
limx→1 2+
x3 − 1 = lim ( x )
x→∞ 6 12 8
(x1/4 − 1) − (x1/5 − 1) 9+ − 2 − 3
x x x
= limx→1 2 a
x3 − 1 = [∵ lim = 0]
1 1 1 1 9 x→∞ x
x 4 − 14 x 5 − x 5 18. Ans (A)
= limx→1 x−1 − x−1
x 3 − 13 x 3 − 1 3 We have, limx→2 [
1
−
2(2x−3)
]
x−1 x−1 x−2 x3 −3x2 +2x
( ) 1 2(2x − 3)
1 1 1 = limx→2 [ − ]
= − = x − 2 x(x − 1)(x − 2)
12 15 60
x(x − 1) − 2(2x − 3)
15. Ans (D) = limx→2 [ ]
x(x − 1)(x − 2)
(√1 + 3x − √1 − 3x) x 2 − 5x + 6
limx→0 = limx→2 [ ]
x x(x − 1)(x − 2)
(√1 + 3x − √1 − 3x) (x − 2)(x − 3)
= limx→0 = limx→2 [ ]
x x(x − 1)(x − 2)
(√1 + 3x + √1 − 3x) (x − 3) −1
× = limx→2 [ ]=
(√1 + 3x + √1 − 3x) x(x − 1) 2
{(1 + 3x) − (1 − 3x)} 19. Ans (D)
= limx→0
x(√1 + 3x + √1 − 3x) 1 + 2 + 3 + ⋯ . +n n
6x limn→∞ { − }
= limx→0 n+2 2
x(√1 + 3x + √1 − 3x) n(n + 1) 1 n
= limn→∞ { × − }
6 2 n+2 2
= limx→0
(√1 + 3x + √1 − 3x) n(n + 1)
{∵ ∑nr=1 r = }
6 6 2
= = = 3.
(√1+√1) 2 (n + 1) 1
= limn→∞ { − }⋅n
Alternatively, use L-H rule 2(n + 2) 2
16. Ans (C) 1 n+1−n−2
= limn→∞ { }⋅n
x + x2 + ⋯ + xn − n 2 n+2
lim { } n (−1)
x→1 x−1 = limn→∞ ×
2 n+2
−1 1 1 1 (1 − cos 2x)sin 5x
= limn→∞ ( ) ⋅ =− ×1=− limx→0
2 2 2 2 x 2 sin 3x
(1 + )
n
2 sin2 x sin 5x
20. Ans (B) = limx→0
x 2 sin 3x
√x + 3 sin 5x
limx→−3 sin2 x ( 5x ) × 5
x+1 = limx→0 2 ( 2 )
x sin 3x
But √x + 3 is not defined on left hand side of -3. ( )×3
3x
Hence, function is not defined sin 5x
sin x 2 5lim5x→0 ( 5x )
21. Ans (B) = limx→0 ( ) ×
x sin 3x
3lim3x→0 ( )
sin2 x + cos x − 1 3x
limx→0 2 × 5 10
x2 = =
3 3
cos x − cos 2 x
= limx→0 25. Ans (D)
x2
1 − cos x sin |x|
= limx→0 ( ) cos x limx→0
x2 x
x LHL = −1, RHL = 1
2sin2
= limx→0 2 cos x
x2 Limit does not exist.
x 26. Ans (B)
2sin2
= limx→0 2 ⋅ limx→0 cos x cot 2x−cosec 2x
x 2 We have, limx→0
( ) ⋅4 x
2
cos 2x 1
2 1 −
= ×1= = limx→0 sin 2x sin 2x
4 2 x
22. Ans (C) cos 2x − 1
= limx→0
sec 5x − sec 3x xsin 2x
limx→0
sec 3x − sec x −(1 − cos 2x)
= limx→0
cos 3x − cos 5x cos x xsin 2x
= limx→0 ( )( )
cos x − cos 3x cos 5x 2 sin2 x
= −limx→0
2 sin 4x sin x cos x x(2 sin xcos x)
= limx→0 limx→0
2 sin 2x sin x cos 5x tan x
= −limx→0 = −1
2 sin 2x cos 2x x
= limx→0
sin 2x 27. Ans (B)
= limx→0 2cos 2x = 2 (x − 1)(x 2 + 2x + 2)
limx→1
23. Ans (C) sin(x − 1)
tan x − sin x x−1
limx→0 = limx→1 limx→1 (x 2 + 2x + 2)
x3 sin(x − 1)
sin x = 1(1 + 2 + 2) = 5.
− sin x
= limx→0 cos x 3
x 28. Ans (B)
sin x(1 − cos x) sin x
= limx→0 We have limx→0
x(1+cos x)
x 3 cos x
x x x
sin x ⋅ 2sin2 2sin cos 1
2 = limx→0 2 2
= limx→0
x 3 cos x 2 x =2
x (2cos )
2
x 2 x
2 sin x sin 1 tan 1
2
= limx→0
cos x
⋅(
x
)⋅( x ) ⋅
4 limx→0 x 2 =
2
2 2
1 1 29. Ans (B)
= 2⋅1⋅1⋅ =
4 2 2sin2 x+sin x−1 0
We have, limx→π ( form)
24. Ans (A) 2
6 2sin x−3sin x+1 0
MATHEMATICS Page | 13A. 3
L LIMITS AND DERIVATIVES
(sin x + 1)(2sin x − 1) dy
= limx→π ∴ = 2x + cos x + (−2)x −3
6 (sin x − 1)(2sin x − 1) dx
(sin x + 1) 2
= limx→π = 2x + cos x − 3
6 (sin x − 1) x
1 34. Ans (D)
limx→π/6 (sin x + 1) (2 + 1) f(x+h)−f(x)
= = = −3 I. f ′ (x) = limh→0
limx→π/6 (sin x − 1) (1 − 1) h
2
(x + h)3 − x 3
30. Ans (B) = limh→0
h
sin x n x 3 + h3 + 3xh(x + h) − x 3
limx→0 = limh→0
(sin x)m h
sin x n x n x m = limh→0 (h2 + 3x(x + h)) = 3x 2
= limx→0 ( ) ( ) ( )
xn x m sin x f(x+h)−f(x)
II. f ′ (x) = limh→0
= limx→0 x n−m = 0 [∵ m < n] h
1 1
31. Ans (C) − 1
(x + h)3 x 3
x100 x99 x2
= limh→0 [∵ f(x) = 3 ]
Given, f(x) = + + ⋯+ +x+1 h x
100 99 2
x 3 − (x + h)3
100x 99 99x 98 2x = limh→0
⇒ f ′ (x) = + + ⋯+ + 1+ 0 (x + h)3 x 3 h
100 99 2
x 3 − [x 3 + h3 + 3xh(x + h)]
[∵ f(x) = x n ⇒ f ′ (x) = nx n−1 ] = limh→0
(x + h)3 x 3 h
⇒ f ′ (x) = x 99 + x 98 + ⋯ + x + 1
−h3 − 3xh(x + h)
Putting x = 1, we get = limh→0
(x + h)3 x 3 h
f ′ (1) = ⏟
(1)99 + 198 + ⋯ + 1 + 1 −h[h2 + 3x(x + h)] −3
100 times = limh→0 = 4
(x + h)3 x 3 h x
=⏟
1 + 1 + 1…+ 1 + 1
d
100 times Alternatively, use (x n ) = nx n−1
dx
⇒ f ′ (1) = 100.
35. Ans (A)
Again, putting x = 0, we get f(0 + h) − f(0)
f ′ (0) = 0 + 0 + ⋯ + 0 + 1 f ′ (0) = limh→0
h
⇒ f ′ (0) = 1 … ( iii) 3−3 0
= limh→0 = limh→0 = 0
h h
From Eqs. (ii) and (iii), we get
f(3+h)−f(3)
f ′ (1) = 100f ′ (0) Similarly, f ′ (3) = limh→0
h
1 − cos 7x 72 1 − cos θ 1
2 49 θ2 1
= lim x = 22 = lim =2=
x→0 1 − cos 9x 9 81 θ→0 sin θ 1 2
θ 2
x2 2 θ
88. Ans (C) 94. Ans (B)
1 − cos x 1 sin x From 1st principle of derivative limx→1
g(x)−g(1)
=
limx→0 2
= , limx→0 =1 x−1
x 2 x
1 − cos 5x g ′ (1)
× 25x 2 1
(5x)2 2x 1
lim = 2 ×0 Where g ′ (x) =
2√25−x2
⇒ g ′ (1) =
√24
x→0 sin 4x 1
× 4x
4x 95. Ans (C)
89. Ans (D)
L- Hospital rule,
1−cos ax a2
We have, limx→0
x2
=
2
f(5) − 5f ′ (x)
lim
1−cos 7x (1−cos 2x) x→5 1
−
x2 x2
Given limit: lim sin 3xTan 5x = f(5) − 5f ′ (5)
x→0 ×3×5
3x 5x
= 7 − 5(7) = −28
[∵ subtractedandadded1
96. Ans (B)
72 22
−
= 2 2 = 45 = 3 10x − 2x − 5x + 1 = (5x − 1)(2x − 1)
15 30 2 ax −1
90. Ans (B) We have, Lt x→0 = log ae
x
−x + 1 − x − 1 = −2x,x ⩽ −1 ⇒ f is continuous
= { 2, − 1 < x ⩽ 1
Now, f ′ (x) = 1 + sin x
2x, x > 1
π
Here, f(−1− ) = 2, f(−1+ ) = 2, f(−1) = 2 ⇒ f′ ( ) = 1 + 1 = 2
2
f(+1− ) = 2, f(1+ ) = 2, f(1) = 2 118. Ans (C)
sof iscontinuous g(3 − h) − g(3)
′ (−1− ) ′ (−1+ )
g ′ (3− ) = limh→0
f = −2, f =0 −h
a√4−h−(3b+2)
f ′ (1− ) = 0, f 1 (1+ ) = 2 = limh→0 …(1)
−h
113. Ans (B) for existence of limit, limh→0 = 0
f isdifferentiableat x = x0 ∴ 2a − 3b = 2 … . . . (2)
⇒ f iscontinuousat x = x0 b(3+h)+2−(3b+2)
Now, g ′ (3+ ) = limh→0 = b … (3)
h
f(x0− ) = f(x0+ )and f ′ (x0− ) = f ′ (x0+ )
Substituting 3b + 2 = 2a in equation (1), we get
x02 = ax0 + b and 2x0 = a
a√4 − h − 2a
b = −x02 g ′ (3− ) = limh→0
−h
114. Ans (A)
(4 − h) − 4 a
f(3 + h2 ) − f(3 − h2 ) = limh→0 ( )=
(−h)(√4 − h + 2) 4
Lt
h→0 2h2
Hence,g ′ (3− ) = g ′ (3+ )
f(3 + h2 ) − f(3) + f(3) − f(3 − h2 ) a
= Lt = b ⇒ a = 4b … (4)
h→0 2h2 4
1 1
1 −(2x − 1) cos x − 2 sin x ,x <
2√x + h ′ 2
limh→0 = f (x) = {
1 2√ x 1
(2x − 1) cos x + 2 sin x ,x ⩾
Alternatively, from 1st principle of derivative, 2
1
1 L. H. D ≠ R. H. D at x = ⇒ f ′ (x) does not exist
2
f(x) = √x ⇒ f′(x) =
2 √x Alternatively,
120. Ans (D) |f(x)|isnotdifferentiable, wheneverf(x) = 0
f(x + y) = f(x) ⋅ f(y) 125. Ans (A)
put x = y = 0 x
x ,x > 0
f(0) = [f(0)]2 ⇒ f(0) = 1 f(x) = = {1 +
x
x
1 + |x| ,x ⩽ 0
f(x + h) − f(x) 1−x
f ′ (x) = Lt
h→0 h f(0+ ) = f(0− ) = f(0)
f(x) ⋅ f(h) − f(x)
Lt f is continuous every where
h→0 h
f(x)−f(0)
f(h) − 1 f ′ (0− ) = Lt−
= f(x) Lt x→0 x−0
h→0 h x
f(h) − f(0) −0
= f(x)L t = Lt 1 − x = 1f ′ (0+ )
h→0 h−0 x→0 x
= f(x) ⋅ f ′ (0) f(x) − f(0)
= Lt+
x→0 x−0
f ′ (5) = f(5) ⋅ f ′ (0) = 3 × 2 = 6 x
−0 1
= Lt 1 + x = =1
x→0 x 1
121. Ans (D)
f ′ (0+ ) = f ′ (0− )
−1
G′ (x) = (−2x) ∴ f is differentiable every where.
2√25 − x 2
G′ (x) − 0 126. Ans (D)
limx→1 = G′ (1) f(x) − f(5)
1
Lt = f 1 (5)
1 1 x→5 x−5
= =
√25 − 1 2√6 = 10a + b
122. Ans (C) 127. Ans (B)
f(x) − f(2) f(x) − f(y)
f ′ (2+ ) = limx→2+ | |≤ x−y
x−2 x−y
3x + 4 − 10 f(x) − f(y)
= limx→2 |Lt x→y |≤0
x−2 x−y
3(x − 2) ⇒ |f ′ (y)| ≤ 0 ⇒ f ′ (y) = 0
= lim =3
x→2 x − 2
⇒ f(x) isconstantwithf(0) = 0
123. Ans (B)
⇒ f(1) = 0
(x − 3) cos x ,x ⩾ 3
f(x) = { 128. Ans (B)
−(x − 3) cos x , x<3
−(x − 3) sin x + cos x , x⩾3 d
f ′ (x) = { cos x 0 =
(x − 3) sin x − cos x ,x < 3 dx
L. H. D ≠ R. H. D at x = 3, ⇒ f ′ (x) does not exist d πx πx π
cos ( ) = −sin ( )⋅
dx 180 180 180
Alternatively,
−π
|f(x)|isnotdifferentiable, wheneverf(x) = 0 = (sin x 0 )
180
124. Ans (B) 129. Ans (C)
log e x
y = log x5 =
log e 5
dy 1 1 f(x) = 1 + x 3
= ×
dx log e 5 x f ′ (x) = 3x 2
1
=
xlog e 5
137. Ans (A)
g1 (x) = x199 + x198 + x197 + ⋯ … + +x + 1
130. Ans (A)
g1 (0) = 1
d af(x) + b (ad − bc) 1
( )= . f (x) 138. Ans (D)
dx cf(x) + d (cf(x) + d)2
d 3x − 5 9 + 10 f ′ (x) = ex g ′ (x) + g(x) ⋅ ex
( )=
dx 2x + 3 (2x + 3)2 f ′ (0) = g ′ (0) + g(0)
131. Ans (B) =4+2=6
Given series is a geometrical series 139. Ans (A)
x
So, f(x) = f(x) = (ax + b)sin x + (cx + d)cos x
1+x
1−0 1 f ′ (x) = (ax + b) cos x + (sin x)a
⇒ f ′ (x) = 2
=
(1 + x) (1 + x)2 +(cx + d)(−sin x) + (cos x)c
132. Ans (A) = x ⋅ cos x
du dv b = c = 0, a = d = 1
d u v −u
( ) = ( dx 2 dx ) 140. Ans (D)
dx v v
1−cos(x−1)
Lt
f(x) = cosx ⋅ cos2x ⋅ cos4x ⋅ cos8x ⋅ cos16x x 3 + 2x 2 + x + 1 x→1 (x−1)2
Lt ( )
sin 25 x sin 32x
x→1 x 2 + 2x + 3
= = [ ∵ usingT. S. R
25 ⋅sin x 25 ⋅sin x 1−cos(x−1) 1
5 Lt 5 2
(x−1)2
1 sin x⋅cos 32x⋅32−sin 32x⋅cos x = ( )x→1 =( ) [ ∵ usingT. S. R
f ′ (x) = [ ] 6 6
25 sin2 x
= Limx→1 nx − 1 n 1 + tan2 x − 2
x − (1) = Limx→π
x−1 4 tan x − 1
m(1)m−1
m tan2 x − 1
= = = Limx→π
n(1)n−1 n 4 tan x − 1
sin[x] sin(−1) dy 1 1
LHL = Limx→0− = = − 3/2
[x] −1 dx 2√ x 2x
dy 1 1
[∵ as x → 0− , [x] → −1 ⇒( ) = − =0
dx at x=1 2 2
sin[x] sin[0 + h]
RHL = Limx→0+ = Limh→0 161. Ans (A)
[x] [0 + h]
x−4
sin[h] Given that f(x) =
2√x
= Limh→0 =1
[h] 1
x ⋅ 1 − (x − 4) ⋅ ]
LHL ≠ RHL 1 √ 2√ x
∴ f ′ (x) = [ ]
2 x
So, the limit does not exist.
156. Ans (C) 1 2x − x + 4 1 x+4
= [ ]= [ ]
Given Limx→0
|sin x| 2 2 √x ⋅ x 2 2(x)3/2
x
1 1+4 5
− sin x ∴ f ′ (1) = [ ]=
LHL = Limx→0− 2 2×1 4
x
sin x 162. Ans (A)
= −1 [∵ Limx→0 = 1] 1
x 1+ 2
x x2 +1
Given y = 1 ⇒y=
sin x 1− 2 x2 −1
x
RHL = Limx→0+ =1
x dy (x 2 − 1) ⋅ 2x − (x 2 + 1) ⋅ 2x
LHL ≠ RHL, sothelimitdoesnotexist. ∴ =
dx (x 2 − 1)2
157. Ans (D) 2x(x 2 − 1 − x 2 − 1) 2x(−2)
= = 2
x 2 − 1, 0 < x < 2 (x 2 − 1)2 (x − 1)2
Given f(x) = {
2x + 3, 2 ≤ x < 3 −4x
2 =
∴ Limx→2− f(x) = Limx→2 − (x − 1) (x 2− 1)2
=4−1=3 163. Ans (A)
MATHEMATICS Page | 13A. 15
L LIMITS AND DERIVATIVES
Given y =
sin(x+9) ⇒ Lt f(x) − 2 = π × Lt x 2 − 1 = 0
cos x x→1 x→1
dy
=
cos x⋅cos(x+9)−sin(x+9)(−sin x) ⇒ Lt f(x) = 2
dx cos2 x x→1
4. Ans (D)
180. Ans (A) Using L − Hospital rule
2
f(x) = 4cos 3 x − 3cos x = cos 3x Given limit:
1 1
− (8 + 3x)−3 = 0
2√4+x 3
Thus, limx→π cos 3x = cos 3π
5. Ans (B)
= −1
On rationalizing given limit
181. Ans (A)
x3 1 1 1
a+b+c = lim [sin−1(x3)] . =1. =
statement 1 is true[i. e. , = 1] x→0 1+√1−x2 2 2
a+b+c
6. Ans (C)
Statement 2 is false
1 1 Since given limit is finite and as x → 0,
lim x+2 1
Because is − denominator → 0
x → −2 x+2 4
(x + x 2 +. . . . . +x 200 ) − 200
= lim sin θ 1 − cos θ
x→1 x−1 = lim ( )( )
θ→0 cos θ θ2
θ
2θ
tan θ 2 sin 2
= lim .
θ→0 θ θ2
By L − Hospital rule 15. Ans (D)
Given limit is
(1 + 2x + 3x 2 +. . . . . +200. x190 ) 200 × 201
= lim = 2 sin x (cos x − 1) + 2(1 − cos 2 x)
x→1 1 2 =4
cos x (1 − cos x)
= 20100
16. Ans (B)
8. Ans (D)
4 sin3 x 4
sin x lim 2
= =1
lim = does not exist x→0 x tan 2 x 4
x→0 |x|
17. Ans (A)
9. Ans (C)
cos x (1 − cos x)
For x ∈ (2,3), [x] = 2; Lt
x→0 x 2 (1 + cos x)2
x 2 x
for ∈ ( , 1) cos 2 x (2 sin2 ) 1
3 3 = Lt 2 =
x x→0 x 2 8
⇒[ ]=0 x 2 (2 cos 2 )
2
3
18. Ans (C)
[x]3 x3
∴ lim+ ( −[ ] ) Given limit is
x→2 3 3
8 x2 x2
1 8 lim (1 − cos ) (1 − cos )
= (2)3 − (0)3 = x→0 x8 2 4
3 3
1 1 1
10. Ans (B) = 32 ( ) ( ) =
16 64 32
lim[cos x] = [1− ] = 0 19. Ans (B)
x→0
− −)
(∵ As x → 0 , cos x → 1 2x + 23−x − 6
lim
sin[cos x] 0 x→2 √2−x − 21−x
∴ lim = =0
x→0 1 + [cos x] 1+0 (2x )2 − 6. 2x + 23
= lim
11. Ans (B) x→2 √2x − 2
π [Multiplying Nr and Dr by 2x ]
limx 2 sin = 0 × a finite number between
x→0 x
lim(2x − 2)(√2x + 2)
-1 and +1=0 x→2
tan θ−sin θ 1
sin θ
−sin θ After applying the limit
= lim = lim cos θ 3
θ→0 θ3 2 θ→0 θ
1 2
2 1
1 1 1 n (√1 + 2 + 1)
1 + + +. . . +∞ 1− n (1 + 1)2
s = lim ( 2 4 )= 2 =4 Lt = =4
n→∞ 1 1 1 3 n→∞ 3 1 1
1 + + +. . . +∞ 1 n2 . √1 +
3 9 n6
1−
3
28. Ans (A)
21. Ans (A)
2
Divide by x Limx 3 cos = 0 × finite number between −
x→0 x
1 1 1 and + 1
x sin ( ) − x sin ( ) − 1
Lim ( x ) = Lim ( x )=0
x→∞ 1 − |x| x→∞ 1 − |x| =0
x 29. Ans (B)
22. Ans (C) sin|x|
π π
As x → 0, |x|
→ 1
x.2 sin(8x) cos(8x)
lim
x→∞ 2 [i. e. , closeto1fromleftside
π π sin|x|
x. sin ( ) ( ) i.e., <1
= lim 4x = 4 |x|
x→∞ 2 2
sin|x|
23. Ans (A) [ ]=0
|x|
Divide with ax and apply limit
30. Ans (C)
24. Ans (B) 12 +22 +32 +.....+n2 Σn2
Given limit = lim = lim
Divide with x10 n→∞ 1+n3 n→∞ 1+n3
1 1 n(n + 1)(2n + 1)
as x → ∞, → 0 = lim
x n→∞ 6 1 + n3
25. Ans (B) 1 1
1 (1 + n) (2 + n)
Given limit = lim
n→∞ 6 1
1 1 1 1 1 1 ( 3 + 1)
n
= lim ( − + − + ⋯ . +
2 n→∞ 1 3 3 5 (2n − 1) 1 2 1
= . 1. =( )
1 1 6 (1) 3
− )=
(2n + 1) 2
26. Ans (C)
1
limn→∞ [12 + (12 + 22 ) + (12 + 22 + 32 ) + ⋯ +
n4
(12 + 22 +…+n2 )
1 n
= limn→∞ ∑ (12 + 22 + 32 + ⋯ . r 2 )
n4 r=1
1 r(r + 1)(2r + 1)
= limn→∞ 4 ∑nr=1
n 6
1 n 2r 3 + 3r 2 + r
= limn→∞ ∑
n4 r=1 6
1 1
limn→∞ 4 [ n2 (n + 1)2 +
n 12
1 1
n(n + 1)(2n + 1) + n(n + 1)
12 12
1 n+1 2
= limn→∞ ( ) +
12 n
1 1 1 1 1 1 1 1
⋅ (1 + ) (2 + ) + ⋅ (1 + ) =
12 n n n 12 n2 n 12
27. Ans (D)
Then find Mean deviation about median using the S. D. of a series unaltered if each item is raised
Σfi |xi −M| (reduced) by some scalar quantity. So S. D. is
formula =
Σfi
independent of change of origin.
25. Ans (B)
∑xi
∴ S. D. = 30
i.e. x‾ = and Mean deviation from the mean =
n 29. Ans (D)
∑|xi −x‾|
n variance = 42 = 16
26. Ans (A) Σxi2
∴ − ( mean )2 = 16
Now, Σ(x − 5)2 = 43 18
Σxi2
⇒ Σx 2 − Σ10x + Σ25 = 43 ⇒ = 16 + 72 = 65
18
⇒ Σx 2 − 10{Σ(x − 5 + 5)} + (18 × 25) = 43
⇒ Incorrect Σxi2 = 1170
⇒ Σx 2 − 10Σ(x − 5) − 10Σ5 + 450 = 43
∴ Correct Σxi2 = 1170 − 212 + 122 = 873
2
⇒ Σx − 10(3) − (10 × 18 × 5) + 450 = 43
Hence, correct variance
⇒ Σx 2 − 30 − 900 + 450 = 43
correct Σxi2
2
⇒ Σx = 523 and Σ(x − 5) = 3 = − ( correct mean )2
18
⇒ Σx − Σ5 = 3 873
= − (6.5)2 = 48.5 − 42.25 = 6.25
⇒ Σx = 3 + (5 × 18) = 93 18
∴ Correct S. D. = √ correct varience
Σx2 Σx 2
Now, σ = √ −( )
n n = √6.25 = 2.5
Statement-2 is true 72
= =9
8
1
Statement-1: Now, Variance = ∑xi2 − (x‾)2
n
x‾ = mean =
1+2+3+⋯+(2n+1) or 41.20 = 11.56 + 5.76 + 2.56 + x 2 + y 2 −
(2n+1)
8.8 × 13 + 38.72
∑(2n + 1)
= Therefore x 2 + y 2 = 97
(2n + 1)
(2n + 1)(2n + 2) But from (i), we have x 2 + y 2 + 2xy = 169
= =n+1
2(2n + 1) From (ii) and (iii), we have 2xy = 72 … (iv)
1 Correct mean =
950
= 38
= (n + 1)(2n + 1) 25
6
64. Ans (D)
58. Ans (B)
Cannot find Mean from only one data out of 8 data.
Sum of 50 observations = 50 36 = 1800
65. Ans (B)
30 and 42 observations are deleted
x 2 3 4 5 6 7
Now, sum of observations
f 4 4 2 3 0 P
= 1800 – 42 – 30= 1728
1728 1728 fx 8 12 8 15 0 7p
New mean = = = 36
50−2 48 ∑ fx 43+7p
Mean = =4=
59. Ans (D) ∑f 13+p
Use TSR (i.e., For each data if a constant is added Corrected ∑ x 2 = 170 − 20 + 30 = 180
or subtracted then variance remains same.) 3330 180 2
correct variance = −( ) = 78
15 15
83. Ans (C)
89. Ans (C)
If terms are in AP; 101
( )[1+1+100d]
2
n2 −1 Now, x̄ = = 1 + 50d
101
S.D=√ × (Common difference)
12 100
1
31, 32, 33…..47 are in A.P with (d) = 1 M. D = {∑|(1 + rd) − (1 + 50d)|
101
r=0
n2 − 1 (17)2 − 1 100
S. D = √ ×d=√ ×1 1
12 12 = ∑|r − 50| d
101
r=0
289−1
=√ = √24 = 2√6 2d 50 × 51 50 × 51
12 = . = d
101 2 101
Where, tn=47a + (n – 1) d
By data, M. D = 255
=31+ (n – 1) (1) = 47
50 × 51
=31 + n – 1 = 47 ∴ 255 = d
101
n = 47 – 30⇒n = 17 101 × 255
⇒d= = 10.1
84. Ans (C) 50 × 51
90. Ans (C)
1
S. D = √ (x1 − x̄ )2 σ If terms are in AP;
10
n2 −1
S.D=√ × (common difference)
12
1
S. D = √ (x1 − x̄ )2 = σ 2, 4, 6….2n, are in A.P with common difference, d
10
=2
85. Ans (A)
n2 −1 n2 −1
We know that when each observation is increased S. D = ×4=
12 3
n(n + 1)(2n + 1)
( ) 2n + 1
6 n(n+1)
= = ⇒ σ=√ d
n(n + 1) 3 3
( )
2
103. Ans (C)
99. Ans (D) n+1
Mean of 1st n- natural no. is
2
We have,
And in a moderately symmetric distribution,
x1 + x2 + ⋯ + xn−1 + xn
X= QD=MD=SD
n
104. Ans (A)
⇒ n X = x1 + x2 + ⋯ + xn−1 + xn
Let the mean of the remaining 4 observations Given series is 148, 146, 144, 142,... whose
be X1 . Then, first term and common difference is
Assuming original nine elements are Consider assumed mean A=25 (Somewhere near
xi di = |xi − Med|
20 30
33 17
39 11
40 10
∑ di 18
MD = = = 2.57 50 0
n 7
122. Ans(B) 53 3
1
MD = ∑ni=1 |xi − x‾| 59 9
n
123. Ans(A)
65 15
The lines of 5 bulbs are given by
1357,1090,1666,1494,1623 69 19
1357+1090+1666+1494+1623
∴ MEAN =
5 Total ∑di = 114
7230
⇒ x‾ = = 1446
5
∑ di 114
1357 89 ∴ MD = = = 12.67
n 9
1494 48
1623 177
∑ di 890
∴ MD = = = 178
n 5
124. Ans(C)
Marks obtained are 50, 69, 20, 33, 53, 39, 40, 65
and 59
2 2 ∑ (x2 2
∑x ∑x i +K +2xi K)
=√ i − ( i ) =√
N
− (m + K)2
n n
619 63 2 ∑ x2 ∑ K2 2 K ∑ xi
=√ − ( ) =√
N
i
+
N
+
N
− m2 − K 2 − 2m K
7 7
619 ∑ x2
=√ − (9)2 =√ i
+ K 2 + 2 Km − m2 − K 2 − 2m K
7 N
619−567 52 ∑ x2 ∑ xi
=√ − 81 = √ =√ i
− m2 [∵ = m]=S
7 7 N N
=K⋅S
∑ xi2 5000 2
5=√ −( ) 130. Ans(A)
100 100
Given that wi = xi + k, x‾i = 48,
∑ xi2
⇒ 25 = − 2500 SD(xi ) = 12,wi = 55 and SD (wi ) = 15
100
then w
‾ i = x‾i + k
∑ xi2
⇒ = 2500 + 25 ⇒ 55 = 48 + k
100
∑ xi2 ‾ i = mean of wi 's and x‾i is the mean of xi′ 's
w
⇒ = 2525
100 SD of wi = SD of xi
∴ ∑ xi2 = 2525 × 100 = 252500 15= l × 12
15
128. Ans(A) ⇒ l= = 1.25
12
Given observation are a, b, c, d and e
from eq. (i) and (ii) we have
a+b+c+d+e
∴ Mean = m = k=w
‾ i − x‾ i = 55 − 1.25 × 48
5
= 55 − 60 = −5
∴ ∑ xi = 5 m
131. Ans(D)
Now mean of the data,
n2 −1
We know that SD of first n natural numbers √
a + K, b + K, c + K, d + K and e + K is 12
a+K+b+K+c+K+d+K+e+K Here n = 10
=
5
(a + b + c + d + e) + 5 K (10)2 − 1
= ∴ SD = √
5 12
5m + 5 K
= = m+K 99
5 =√ = √8.25 = 2.87
12
∑ (xi +K)2 ∑ xi +K 2
∴ SD = √ −[ ] 132. Ans(D)
N N
∑ xi 2 52
∴ Standard deviation, σ = √ − (x‾)2 S. D = √
7
n
2. Ans(C)
330
=√ − (8)2 = √2 x̅1 = 52, ̅̅̅
x2 = 42
5
x‾ = 50
141. Ans(C)
n1 x‾1 + n2 ̅̅̅
x2
X = {−1,0,1, K} x‾ =
n1 + n2
k
X‾ = n1 52 + (100 − n1 )42
4 50 =
100
σ = √5 5000 = 52n1 + 4200 − 42n1
2
∑ xi2 ∑ xi 800 = 10n1
⇒ −( ) =5
n n
n1 = 80
2 2
2+k k
− =5 3. Ans(A)
4 16
Average of three subjects
8 + 4k 2 − k 2
=5 x1 + x2 + x3 75 + 80 + 85
16 = = 80
3 3
8 + 3k 2 = 80
x1 + x2 + x3 = 3 × 80
3k 2 = 72
One more subject x4
3k 2 = 72
will be added
k = √24
x1 + x2 + x3 + x4
Avg =
= 2√6 4
142. Ans (B) 3 × 80 + x4
=
4
n2 −1
S.D. = √ (n = 17) x4
12 = 60 + ( ) ⩾ 60%
4
172 − 1 4. Ans(B)
=√ = 2√6
12 Gives series 1,2, 22 , 23 , 24 , … … 2n
143. Ans (D) 1+2+22 +⋯…+2n
A.M of the series =
n+1
∑xi2
2525 = 2n+1 − 1
100 =
n+1
⇒ ∑xi2 = 252500
5. Ans(B)
144. Ans (D) 1(12 )+2(22 )+⋯+n(n2 )
mean (x‾) =
145. Ans (C) 12 +22 +⋯+n2
n (n + 1)2
2
3 n(n + 1)
= =
4n(n + 1)(2n + 1) 2 (2n + 1)
6
MOCK TEST SOLUTIONS 6. Ans(C)
1. Ans(A) x̅1 = 52, ̅̅̅
x2 = 42
X = {5,6,8,9,10,12,13} x‾ = 50
X ′ = X − 9 = {−4, −3, −1,0,1,3,4} n1 x‾1 + n2 ̅̅̅
x2
x‾ =
n1 + n2
V(X ′ ) = V(X − 9)
n1 52 + (100 − n1 )42
∑(xi′ )2
′)
∑xi 2 50 =
V(X = −( ) 100
n n
5000 = 52n1 + 4200 − 42n1
2(26) 52
= −0 = 800 = 10n1
7 7
MATHEMATICS Page | 14A. 16
STATISTICS
= np[qn−1 + n−1
C1 , qn−2 p + ⋯ … + pn−1 ] The change in the maximum and minimum values
= np[q + p]n−1 =np in the series does not affect the median of that
=
49×100−40−20−50+60+70+80 16. Ans(A)
100
Data : 2,3,4,6,7,7,8
50 × 100
= = 50 Median = 6
100
11. Ans(C) Data : 2, 3,4, 6, 9, 9, 9, 6
x = {x1 , x2 , … … xn } Mode = 9
6+9
μ(x) = μ(xi) = x‾, i = 1,2 … … n Average = = 14.5
2
x1 = {x1 + 1, x2 + 2, … … , xn + n} 17. Ans(C)
μ(x1 ) = μ(xi + i), When i = 1,2 … . . n
The G.M of x1 , x2 , … … xn is n√x1 ⋅ x2 ⋅ x3 … … xn
1 + 2 + ⋯…+ n 1
= μ(xi) +
n = (2 ⋅ 22 ⋅ 23 … … 2n )n
n+1 1
= x‾ + = [21+2+3+⋯……+n ]n
2
1
12. Ans(B) n(n+1) n n+1
= [2 2 ] =2 2
2n(2n + 1) mean
(2n + 1)a + d
= 2 = a + nd
2n + 1 3 + 5 + 11 + 13 + 17 + 23 + 29
X‾ =
∑ |xi − x‾| 8
M. D (X‾) = = 15
2n + 1
∑|x −x‾|
2[nd + (n − 1)d + ⋯ … + d] M.D (X‾) = i
= 8
2n + 1
12 + 10 + 4 + 2 + 2 + 4 + 8 + 14
2n(n + 1) =
= d 8
2(2n + 1)
56
30. Ans(D) = =7
8
X = {3,5,11,13,17,19,23,29}